Sie sind auf Seite 1von 168

CHAPTER 1: BASIC LOGIC CONCEPTS

Recognizing Arguments

What is ARGUMENT?
In logic, we use the term "argument" to mean a set of propositions in which some propositions--the
premises--are asserted as support or evidence for another--the conclusion.
Why do we need to recognize it?
 To understand the language of the passage.

Difficulty: Even with a thorough comprehension of the language, the identification of an argument can be
problematic.
Why?
1. Because of the peculiarities of its formulation.
2. We may be unsure (propositions, conclusion )
How then shall we proceed to our arguments?

A. Conclusion Indicators and Premise Indicators

Conclusion Indicator Method Premise Indicator Method:


-Words/phrases ( such as “therefore” or -Words/ phrases (like “because” and
“thus”) that serve to signal the appearance “since”) typically serve to mark that
of an argument’s conclusion or of its what follows it are the premises of an
premises and usually indicating that what argument.
follows it is the conclusion of that
argument.
 therefore  because
 hence  since
 thus  for
 so  as
 for these reasons  the reason is that
 it follows that  as indicated by
 we may infer  follows from
 I conclude that  as shown by
 which shows that  inasmuch as
 which entails that  in view of the fact that
 which implies that  may be inferred from

1|Page
 consequently
 proves that
 as a result
 for this reason

B. Arguments in Context
According to Copi and Cohen, sometimes it is just the meaning of the passage, or its setting, that indicates the
presence of an argument. Thus, these arguments need understanding through context. Statements (propositions)
without argument indicators will probably be used, but the context will indicate that certain statements are put
forth as reasons in support of the view. In legal logic, this is done by knowing the general rule, the facts and how
the facts apply to the general rule.

Example:
“Free speech will be abused. Every liberty is abused.” (Note that the argument is enthymematic)
Major premise- Every liberty is abused
Minor premise- Free speech is a kind of liberty (There is a need for the audience to fill in missing proposition/s)
Conclusion- Therefore, free speech will be abused

*The force of an argument can be appreciated only when one understands the context in which the argument is
presented.
Sometimes a premise is not given in declarative form, but as rhetorical question:
If a man say, ‘I love God,’ and he hateth his brother, he is a liar; for he that loves not his brother whom he hath
seen, how can he love God whom he hath not seen? (1 John 4:20)

Rhetorical question: An utterance used to make a statement, but which, because it is in interrogative form and is
therefore neither true nor false, does not literally assert anything.
In some arguments, the conclusion is given as a command or appears in the form of an imperative.
Which of course can be interpreted as:
1. Wisdom is the principal thing.
2. Therefore, one ought to acquire wisdom.

2|Page
C. Premises or Conclusion Not in Declarative Form
Many arguments assume certain premises that are not stated. Such arguments are called enthymemes.

He would not take the crown;


Therefore ’tis certain he was not ambitious. (Shakespeare’s Julius Caesar)

This argument depends on a missing premise, that “One who would not accept the crown must not have been
ambitious.”

Arguments and Explanations


Compare the following two passages:
1. Lay up for yourselves treasures in heaven, where neither moth nor rust consumes and where thieves do
not break in and steal. For where your treasure is, there will your heart be also. —Matt. 7:19
2. Therefore is the name of it [the tower] called Babel; because the Lord did there confound the language of
all the earth. —Gen. 11:19
The first passage is clearly an argument. Its conclusion, that one ought to lay up treasures in heaven, is supported
by the premise (here marked by the word “for”) that one’s heart will be where one’s treasure is laid up. The
second passage, which uses the word “therefore” quite appropriately, is not an argument. It explains why the
tower (whose construction is recounted in Genesis) is called Babel. The tower was given this name, we are told,
because it was the place where humankind, formerly speaking one language, became confounded by many
languages—the name is derived from a Hebrew word meaning “to confound.” The passage assumes that the
reader knows that the tower had that name; the intention is to explain why that name was given to it. The phrase,
“Therefore is the name of it called Babel,” is not a conclusion but a completion of the explanation of the naming.
In addition, the clause, “because the Lord did there confound the language of all the earth,” is not a premise; it
could not serve as a reason for believing that Babel was the name of the tower, because the fact that that was the
name is known by those to whom the passage is addressed. In this context, “because” indicates that what follows
will explain the giving of that name, Babel, to that tower.
Whether some passage is an argument or an explanation depends on the purpose to be served by it. If our aim is
to establish the truth of some proposition, Q, and we offer some evidence, P, in support of Q, we may
appropriately say “Q because P.” In this case we are giving an argument for Q, and P is our premise.
Alternatively, suppose that Q is known to be true. In that case we don’t have to give any reasons to support
its truth, but we may wish to give an account of why it is true. Here also we may say “Q because P”—but in
this case we are giving not an argument for Q, but an explanation of Q.

In an explanation, one must distinguish what is being explained from what the explanation is.

3|Page
Explanandum vs. Explanans
Explanandum Explanans
What is being Means of
explained explanation

Example: The sky appears blue from the earth because light rays from the sun are scattered by particles in the
atmosphere.
*This statement merely provides an explanation not an argument. An argument aims to prove.

Deductive and Inductive Arguments

A deductive argument is an argument in which it is thought that the premises provide a guarantee of the truth of
the conclusion. In a deductive argument, the premises are intended to provide support for the conclusion that is so
strong that, if the premises are true, it would be impossible for the conclusion to be false.
Here is a classic example:
1. Socrates was a man (premise)
2. All men are mortal (premise).
3. Socrates was mortal (conclusion)
If A = B, and B= C, then A = C.

An inductive argument is an argument in which it is thought that the premises provide reasons supporting the
probable truth of the conclusion. In an inductive argument, the premises are intended only to be so strong that, if
they are true, then it is unlikely that the conclusion is false.
Here is an example:
1. Socrates was Greek (premise).
1. Most Greeks eat fish (premise).
2. Socrates ate fish (conclusion).
In this example, even if both premises are true, it is still possible for the conclusion to be false (maybe Socrates
was allergic to fish, for example). Words which tend to mark an argument as inductive—and hence probabilistic
rather than necessary—include words like probably, likely, possibly and reasonably.

Deductive Arguments vs. Inductive Arguments


It may seem that inductive arguments are weaker than deductive arguments because in a deductive
argument there must always remain the possibility of premises arriving at false conclusions, but that is true only
to a certain point. With deductive arguments, our conclusions are already contained, even if implicitly, in our
premises. This means that a deductive argument offers no opportunity to arrive at new information or new ideas—
4|Page
at best, we are shown information which was obscured or unrecognized previously. Thus, the sure truth-
preserving nature of deductive arguments comes at the expense of creative thinking.
Inductive arguments, on the other hand, do provide us with new ideas and possibilities, and thus may expand our
knowledge about the world in a way that is impossible for deductive arguments to achieve.

NOTE: Inductive reasoning is logic of probabilities and generalities, not certainties. It yields workable rules,
but not proven truths. It is best never to apply enthymemes and rhetorical questions in inductive reasoning.

Validity and Truth

Validity is a characteristic of any deductive argument whose premises, if they were all true, would provide
conclusive grounds for the truth of its conclusion. Such an argument is said to be valid. Validity is a formal
characteristic; it applies only to arguments, as distinguished from truth, which applies to propositions.
A deductive argument is valid when it succeeds in linking, with logical necessity, the conclusion to its premises.
Its validity refers to the relation between its propositions—between the set of propositions that serve as the
premises and the one proposition that serves as the conclusion of that argument. If the conclusion follows with
logical necessity from the premises, we say that the argument is valid. Therefore, validity can never apply to any
single proposition by itself, because the needed relation cannot possibly be found within any one proposition.
Truth and falsehood, on the other hand, are attributes of individual propositions. A single statement that serves as a
premise in an argument may be true; the statement that serves as its conclusion may be false. This conclusion might
have been validly inferred, but to say that any conclusion (or any single premise) is itself valid or invalid makes no
sense.

TRUTH- attribute of a proposition that asserts what really is the case.

Note: An argument may be valid even when its conclusion and one or more of its premises are false.
There are many possible combinations of true and false premises and conclusions in both valid and invalid
arguments. Here follow seven illustrative arguments, each prefaced by the statement of the combination (of truth
and validity) that it represents.

True Premises, False Conclusion

0. Valid Impossible: no valid argument can have true premises and a false conclusion.

5|Page
Cats are mammals.
1. Invalid Dogs are mammals.
Therefore, dogs are cats.

True Premises, True Conclusion

Cats are mammals.


2. Valid Tigers are cats.
Therefore, tigers are mammals.

Cats are mammals.


3. Invalid Tigers are mammals.
Therefore, tigers are cats.

False Premises, False Conclusion

Dogs are cats.


4. Valid Cats are birds.
Therefore, dogs are birds.

Cats are birds.


5. Invalid Dogs are birds.
Therefore, dogs are cats.

False Premises, True Conclusion

Cats are birds.


6. Valid Birds are mammals.
Therefore, cats are mammals.

Cats are birds.


7. Invalid Tigers are birds.
Therefore, tigers are cats.

• It is clear that the truth or falsity of an argument’s conclusion does not by itself determine the validity or
invalidity of that argument. Moreover, the fact that an argument is valid does not guarantee the truth of its
conclusion.
Note:
• If an argument is valid and its premises are true, we may be certain that its conclusion true also.
• If an argument is valid and its conclusion is false, not all of its premises can be true.

6|Page
SOUND- when an argument is valid and all of its premises are true.
CHAPTER 2: ANALYZING ARGUMENTS

Paraphrase Arguments is by setting forth its propositions in clear language and in logical order. This
may require the reformulation of sentences, and therefore great care must be taken to ensure that the paraphrase
put forward captures correctly and completely the argument that was to be analyzed.
In going through this process, we must identify two processes: 1. identifying the premises and
conclusions, and 2. fitting these premises and conclusions into common argument patterns. This is for teaching
purposes; in real life, these two things aren’t so neatly divided. By having this slightly artificial, but more finely
grained account of the process, you can get a better sense of what aspects of your paraphrasing need work.
Sometimes paraphrasing can bring to the surface what was assumed in an argument but was not fully or
clearly stated.

Example:
Jose could not have been the late comer of the class, because he exercised before the class starts.

# Jose could not have been the late comer of the class,
# Because he exercised before the class.
# The person who exercised before the class starts become late comer- hidden premise

Example:
Jose didn't get much sleep last night, because he looks tired.

# Jose didn't get much sleep last night.


# Because he looks tired
# A person who looks tired cannot sleep at night- hidden premise

Paraphrase enables us to distinguish and examine the premises and process of reaching a conclusion about
the premises from known fact compressed into Hardy’s single sentence.

Diagramming Arguments is to represent the structure of an argument graphically; the flow of premises
and conclusions is displayed in a two-dimensional chart, or picture, on the page. A diagram is not needed for a
simple argument, even though drawing one can enhance our understanding. When an argument is complex, with
many premises twist together in various ways, a diagram can be exceedingly helpful.

7|Page
 Diagrams reveal dependence or independence of premises
 Diagrams reveal the ‘lines’ of reasoning
 Diagrams reveal where the ‘action’ is
• What is at the core of the argument
• What is/are the main line(s) of reasoning
• What is (considered) less important
 In short, diagrams provide an overall picture of the structure of the argument.
As such, potential problems or gaps in the argument quickly reveal themselves as well.

By bracketing the premises in the diagram of this argument, we show that its premises give support only
because they are joined.
Example:

(1) Jose didn't get much sleep last night. (2) He has dark circles under his eyes. (3) He looks tired.

The conclusion is the first sentence in the passage.

When some premises give direct support not to the conclusion but to other premises that support the
conclusion.

Examples:
1. Coal seams have been discovered in Antarctica. 2. This means that the climate there was once warmer than it is
now. 3. Thus, either the geographical location of the continent has shifted or the whole Earth was once warmer
than it is now.

(1) Questionable research practices are far more common than previously believed, (2) after all, the Acadia
Institute found that 44 percent of students and 50 percent of faculty from universities were aware of cases
of plagiarism or falsifying data

Premise's can also support more than conclusion.

8|Page
(1) Lenses function by refracting light at their surfaces. (2) Consequently, not only does their action depends
on the shape of the lens surfaces but also (3) it depends on the indices of refraction of the lens material
and the surrounding medium.

Aside from analyzing arguments, argument diagrams are also good for:
 Organizing arguments
 Communicating arguments

Argument diagrams have been used to organize complicated arguments in:


 academia (especially philosophy)
 court rooms
 industry (think listing ‘pros’ and ‘cons’ in the context of decision making)

Complex Argumentative Passages is when several arguments are interwoven, with some propositions
serving as both premises and subconclusions while other propositions serve only as premises, and still others are
repeated in different words, can be a challenge. (Arguments are exceedingly complicated)

The presence of an argument in a passage is discovered by understanding the author's intention of proving
a statement by offering reasons or evidence. Generally speaking, these reasons are presented as verbal reports
although they might not be initially presented in declarative sentences.
There are three main ways of judging the presence of an argument:

1. The author or writer explicitly states explicitly lists the reasons, evidence, justification, rationale, or
proof of a statement.

I conclude the dinosaurs probably had to cope with cancer. These are my reasons: (2) a beautiful bone
found in Colorado filled with agate has a hole in its center, (3) the outer layer was eroded all the way
through, and (4) this appearance closely matches metastatic bone tumors in humans.

2. The author uses argument indicators signifying the presence of an argument. (Common premises and
conclusion indicators are listed below in Section IV)

9|Page
Since the solution turned red when the indicator was added, (2) I conclude it is acidic, inasmuch as acidic
substances react with this indicator to form a red color.

3. The passage under question implicitly provides an answer to the sometimes irreverent question of
"What are you trying to prove?" The presence of an argument cannot be always known with certainty.
A charitable, conventional interpretation of the content and context of the passage is assumed.

(1) The types of sentences you use are quite varied. (2) I've noticed that your essays are quite
sophisticated. (3) You have been learning much more about sentence structure.

[The conclusion is statement (3)].

Complex Arguments is often helpful to reconstruct the argument backwards from the conclusion.
Consider the following argument.

(1) If students were environmentally aware, they would object to the endangering of any species of animal. (2)
The well-known Greenwood white squirrel has become endangered (3)as it has disappeared from the Lander
Campus (4) because the building of the library destroyed its native habitat. (5) No Lander students objected. (6)
Thus, Lander students are not environmentally aware.
The premises indicators suggest that (2) is a subconclusion of (3) since the indicator "as" connects them,
and (3), in turn, is a subconclusion of (4) since the indicator "because" connects those two statements.

Statement (6) is the final conclusion since it has the conclusion indicator "thus" and the import of the
paragraph indicates that this statement is the main point of the argument.
Intuitively, the structure of the first statement (1) together with statement (5) is a common argument
form:

If students were environmentally Aware, they would Object to the endangering of any species of animal.
No student Objected (to the endangering of the Greenwood white squirrel).
Hence the whole argument can now be pieced together as:

Logical analysis, paraphrase supported by diagrams, can expose such deficiencies. By exhibiting the
structure of a reasoning process, we can better see what its strengths and weaknesses may be.

Problems in Reasoning is advancing from premises known (or affirmed for the purpose) to conclusions.
We construct arguments of our own every day, in deciding how we shall act, in judging the conduct of others, in

10 | P a g e
defending our moral or political convictions, and so on. Skill in devising good arguments (and in deciding
whether a proffered argument is good) is of enormous value, and this skill can be improved with practice. Ancient
games of reasoning, such as chess and go, exercise that skill, and there are some widely known commercial games
(Clue and Mastermind are examples) that also have this merit. Problems may be contrived which are designed to
test and strengthen logical skills; some of these are presented in this section. Such problems are far neater than
those that arise in real life, of course. But solving them may require extended reasoning in patterns not very
different from those employed by a detective, a journalist, or a juror. Chains of inferences will be needed, in
which subconclusions are used as premises in subsequent arguments. Finding the solution may require the
creative recombination of information given earlier or discovered. Contrived problems can prove frustrating—but
solving them, like every successful application of reasoning, is quite satisfying. In addition to being models for
the employment of reason, logical games and puzzles are good fun. “The enjoyment of the doubtful,” wrote the
philosopher John Dewey, “is a mark of the educated mind.”

To solve problems we look first for a sphere in which we have enough information to reach some
conclusions going beyond what is given in the premises.

Example:
Question: Today is Monday. After 61 days, It will be:
1.Tuesday
2.Monday
3.Sunday
4.Saturday

Answer:
Saturday

why?: Each day of the week is repeated after 7 days. So, after 63 days, it will be Monday. After 61, it will be
Saturday.

Question: Pointing to a man, a woman said, “His mother is the only daughter of my mother”. How is the woman
related to the man?
1.Mother
2.Sister
3.Daughter
4.Grandmother

11 | P a g e
Answer:
Mother

Why?: Only daughter of my Mother – means herself. Thus, she is his mother.

Real problems are often identified, initially at least, only by the recognition of some inconsistency or the
occurrence of an unusual event, or perhaps just by the feeling that something is amiss, rather than by a well
formed question seeking a clearly defined answer. In spite of these differences, contrived problems and puzzles
are useful in strengthening our reasoning skills—and they are fun.

References:
http://philosophy.lander.edu/logic/diagram.html
http://community.middlebury.edu/~kkhalifa/Teaching/Guides/Paraphrasing%20Arguments.pdf
http://www.cogsci.rpi.edu/~heuveb/teaching/CriticalThinking/Web/Presentations/ArgumentDiagrams.pdf
Irving M. Copi, Carl Cohen, Kenneth McMahon-Introduction to Logic-Pearson Education (2013)
http://legacy.earlham.edu/~peters/courses/log/tru-val.htm
https://www.thoughtco.com/deductive-and-inductive-arguments-249754
Introduction to Logic by Irving M. Copi and Carl Cohen (13th and 14th Edition), pp. 2-32

12 | P a g e
CHAPTER 3: LANGUAGE AND DEFINITIONS
Language and Function

Meriam Webster Dictionary defines language as “A language is a system of communication which consists
of a set of sounds and written symbols which are used by the people of a particular country or region for talking or
writing.” Communication means when we relay our message to someone else mostly by verbal expression.

In relaying a message it does not mean only say one word to express but we it with group of words which
is called a sentence. Therefore, sentence is the structural foundation of our ability to express ourselves. Although
language is used to express ourselves there are also varieties in which language is used.

The three major function of language are:

1. Informative,

2. Expressive

3. Directive.

“The informative function of language involves an effort to communicate some content.”1 It means, it
informs. To further understand informative use of language that effort to communicate some content can be broken
into three kinds which are: first, “the informative function affirms or denies propositions, as in science or the
statement of a fact.”2 An example of this “the sun rises in the east.” Second, “the informative function is used to
describe the world or reason about it”3 an example is whether a state of affair has occurred or not or what might
have led to it. Third, “sentences in informative function have a truth value; that is, the sentences are either true or
false.”4

“An expressive function of language, on the other hand, intends only to vent some feeling, or perhaps to
evoke some feeling from other people.”5 It must be clear that the two main aspect of expressive function is to evoke
some feeling and to vent or express feeling. Therefore it can be assumed that it is a language of feeling wherein it
is something that is neither true nor false. Poetry and literature are among the best examples, but much of, perhaps
most of, ordinary language discourse is the expression of emotions, feelings or attitudes. Example, “If you are in
heaven and I’m in hell, I will look up to you with reverence, but if I’m in heaven and you are in hell, I will go down
in hell because heaven is not heaven without you!”

“Finally, directive uses of language aim to cause or to prevent some overt action by a human agent.”6 Since
it aims to cause or prevent something it is normally in the forms of request or commands. Thus, when it is a

1
Kemerling, Garth, Language and Logic. philosophypages.com. http://www.philosophypages.com/lg/e04.htm. (accesed:
February 6, 2018).

2
Introduction to Logic.http://philosophy.lander.edu.http://philosophy.lander.edu/logic/form_lang.html.(accesed: February
6, 2018)

3
Ibid.

4
Ibid.

5
Kemerling, Garth, Language and Logic. philosophypages.com. http://www.philosophypages.com/lg/e04.htm. (accesed:
February 6, 2018).

6
Ibid.
13 | P a g e
command or a request, like the expressive function of language, this form of language neither have true or false
proposition.

Aside from the three major function of language there are also less common type of functions are
ceremonial and performative. “The ceremonial is also ritual language use”7 wherein there is a combination of
expressive and other function. “In performative, language performs the words itreports”8 example of this are I do,
I accept, I apologize, I congratulate, I promise and etc.

Although functions of language was defined as like informing, giving command and expressing feeling, it
should not be confused with the forms of language such as the declarative, exclamatory or the imperative because
although a sentence is in exclamatory form it can be an informative, expressive or directive. Therefore, there is no
direct relation between the form and the function of the sentence. It can be understood better in the graph below:

Function Informative Expressive Directive


Sentence Type
assertion / The room is cool. I had a nice time. I would like some coffee.

declarative
question / But isn't this room 222A? Isn't that great? Don't you want to help me?

interrogative
command / Read pages 1-10 for the test. Have a nice day. Shut the windows.

imperative /
exclamation / The universe is bounded! I'm really glad! It's late!

exclamatory

The problem here really is not the distinction between the forms and functions of language. The problem is
the distinction between the function from other functions. As I explained in the preceding paragraph, there are
instances wherein there is a combination of functions. It should be rare in a discourse when there is only one
function. Thus one task is to identify what is the function of the sentence.

Another is that, “function in a particular instance often depends more on the specific context and tone of
voice than it does on the grammatical form or vocabulary of what is said.”9 The simple declarative sentence, "I'm
hungry," for example, could be used to report on a physiological condition, or to express a feeling, or implicitly to
request that someone feed me. In fact, uses of two or more varieties may be mixed together in a single utterance;
"Stop that," for example, usually involves both expressive and directive functions jointly. In many cases, however,
it is possible to identify a single use of language that is probably intended to be the primary function of a particular
linguistic unit.

7
Introduction to Logic.http://philosophy.lander.edu.http://philosophy.lander.edu/logic/form_lang.html.(accesed: February
6, 2018)

8
Ibid.

9
Kemerling, Garth, Language and Logic. philosophypages.com. http://www.philosophypages.com/lg/e04.htm. (accesed:
February 6, 2018).

14 | P a g e
Therefore “the importance of the differentiation of functions is shown by recognizing that the correct
evaluation of a passage requires knowledge of the functions relevant to the situational context.”10

Emotive Language, Neutral Language, and Disputes

Introduction:

Arguments are made up of language, so we cannot fully understand arguments without first understanding
language. Significantly, language is a shared convention which allows us to communicate with others (Armstrong
and Fogelin 2010).Language serves many purposes: to express feelings; to give commands; to ask questions; to
convey information. Often these purposes are intricately intertwined: ‘I love you’ may superficially seem like a
mere statement of fact, but when spoken it may convey feeling directly in the way in which it is spoken (Dean
2003).

However, according to Copi and Cohen (2009), language can also be “deceptive and manipulative” where
careless use of it can lead to misunderstandings and disputes (79). The way we convey our beliefs as a neutral fact
may have an impact on the attitude of the listeners (ibid 2009).

Euphemisms

Knowing that some words may be offensive or derogatory to others, euphemisms are devised in order to
counter the negative attitudes attached to a certain word. In short, euphemisms are “gentle words for harsh realities”
(Copi and Cohen 2009, 79).

Examples:

- “Maintenance Workers” instead of Janitors


- “Waitpersons” instead Waiters
- “Passed Away” instead of Died
- “Correctional facility” instead of Jail
- “Comfort woman” instead of Prostitute

Seven Dirty Words (The Seven Words You Can Never Say on Television by George Carlin)

“…That's what they told us they were, remember? "That's a bad word!" No bad words, bad thoughts, bad
intentions, and words!
You know the 7, don't you, that you can't say on television?
"Sh*t, p*ss, f*ck, c*nt, c*cks*cker, m*therf*cker, and t*ts."
Those are the heavy seven. Those are the ones that'll infect your soul, curve your spine, and keep the country from
winning the war..”

These words are not allowed to be used in the media because they have unacceptable emotive meanings
which are distinguishable from their literal meanings (Copi and Cohen 2009).

Emotive Language and Neutral Language

10
ibid
15 | P a g e
Words or phrases can exhibit the distinction between informative and expressive uses of language. Such
words or phrases which comprise language could have a literal or descriptive meaning or an emotive meaning. The
former refers to the way things are while the latter is merely an expression of a positive or negative feeling towards
the former (Kemerling 2017).

In our everyday communication, little do we notice that the language we use could either be neutral or
emotive depending on the situations we are in. First, emotive language is a “emotionally colored language” we use
to convey our feelings in addition to a certain claim (Copi and Cohen 2009). The use of the emotive language is
evident in poetry, broadcasting, advertising, campaigning, and other similar contexts where the words used are
usually metaphorical and persuasive.

Poetry is a literary work which is considered as an “art of rhythmical composition, written or spoken, for
exciting pleasure by beautiful, imaginative, or elevated thoughts” (dictionary.com). Clearly, if emotive language
will not be used in poetry, its “beauty”, “pleasure”, and “art” will be defeated.

In the broadcasting industry, one pertinent character of the Philippine news is it being “bombastic”. One
example is, instead of saying “Nagbanggaan ang dalawang sasakyan sa Marcos Highway”, the news structures its
language as “Nagsalpukan ang dalawang sasakyan.” The goal of the television and radio news is to give
information or warning, and in order to achieve this, they first have to capture their viewers and listeners attention
through the use of emotive language (Beltran 2016). Likewise, the advertising industry also needs to present a
“shock” value in selling their products and services to persuade the consumers (Goddard 1998). Copi and Cohen
(2009) also mentioned about the use of “rhetorical tricks” in political campaigns. This trickery is utilized to sell
their platforms and treat the voters as consumers.

However, the use of the emotive language is highly appropriate in some contexts when the use of the neutral
language would best fit the situation. An example which Copi and Cohen (2009) illustrated is in a survey research.
The responses to a survey will certainly depend in the good measure and choice of words used in asking a question.

In researches, ideas must be “operationalized”. This process is best defined as the conversion of the abstract
idea or notion into a measurable item. It involves taking something that is conceptual and making it observable, or
going from abstract to concrete. Thus, emotive language is not preferred because the goal of research is to prevent
errors by using objectivity (Dantzker and Hunter 2012).

Which is preferable?

The neutral language is preferred because the primary purpose of logic is establishing the truth—as we are
when assessing the logical merits of an argument—the use of words laden with emotive meaning can easily distract
us from our purpose (Kemerling 2017). Thus, from the words of Copi and Cohen (2009), “we generally strive for
language that is, so far as possible, free of the distortion that emotive meanings may introduce” (80).

Is it possible that emotive language can be eliminated completely?

No. It is nearly impossible to avoid some emotive element in the use of language especially when it comes
to issues concerning morality (Copi and Cohen 2009). An example would be the conflict posited by the morality of
abortion. There is a dichotomy between the classification of “pro-life” or “pro-choice” individuals.

The pro-life argument always posits the “right to life” and the morality which is attached to it. It argues that
it is always “morally wrong to kill an innocent being”. On the other hand, the pro-choice argument responds through
the use of “facts” and argues that the conditions of personhood is based from either the genetic code (which
16 | P a g e
determines biological species), brain activity (first detected around eight weeks), sentience (capacity to feel pain
and pleasure), viability (when the fetus can survive outside the womb), and rationality (and other related capacities)
(Armstrong and Fogelin 2010). In this issue, emotive language cannot be avoided.

If the emotive meaning cannot be completely avoided, Copi and Cohen (2009) suggests as a remedy is that
“whether we should avoid emotive language, or rely on it, depends on the purpose language is intended to serve in
the context” (79).

Disputes: Disagreement in Beliefs about the Facts, Disagreement in Attitudes about the Facts

“The achievement of human knowledge is often hampered by the use of words without fixed signification.” – John
Locke

The careless use of language, without considering its purpose and appropriating it in a proper context, lead
to needless misunderstandings and dispute. Differences arise when parties is in dispute. Such disputes may include
disagreements in beliefs about the facts and disagreement in attitudes about the facts (Copi and Cohen 2009).

X and Y may agree about certain facts but their attitudes toward such facts may be different and vice versa.
An example would be the legislation about death penalty, they may agree that it is the right of the state to execute
criminals or they may both believe that it is violative of the right to life. They may agree in facts but disagree in
attitude or disagree in facts but agree in attitude or disagree in both (ibid 2009).

How do we resolve these disputes?

To resolve these disputes in the factual and emotive meaning, it is fundamental to first determine what the
issue is about, then distinguish whether the issue involves a disagreement in belief or in attitude about the facts, and
lastly try to resolve it depending on the issue and the kind of disagreement present (ibid 2009).

Steps: (Copi and Cohen 2009)

 Determine the issue.


- For example, the issue is about the death penalty.
 Determine whether the issue involves a disagreement in belief or in attitude.
- In case there is a disagreement in belief but agreement in attitude - X and Y both believe that it is right
to punish a criminal. X thinks that it is just to execute criminals while Y thinks it is excessive and
unjust.
 Attempt to resolve the dispute in accordance with the kind of disagreement
- If there is a disagreement in belief like on the example cited above, then use the methods of finding the
facts in a mutually agreeable manner: authority, science, or observation.
- If there is a disagreement in attitude, then the methods of rhetoric and peruasion might be helpful

Other Examples:

Disagreement in Beliefs
“A bad peace is even worse than war” – Tacitus, Annals
“The most disadvantageous peace is better than the most just war” – Desiderious Erasmus, Adagia, 1539

“Absence makes the heart grow fonder”


“Out of sight, out of mind”
17 | P a g e
Disagreement in Attitudes
“For when the One Great Scorer comes
To write against your name.
He marks – not that you won or lost-
But how you played the game.” – Grantland Rice, journalist

“Winning isn’t everything. It’s the only thing.” Vince Lombardi, coach

Sources: (Chapter 3.2)

Armstrong, Walter and Robert Fogelin. 2010. Understanding Arguments: An Introduction to Informal Logic. 8th ed.
US: Wadsworth Cenage Learning.

Beltran, James. 2016. “Fundamentals of Broadcast Writing.” Lecture. College of Arts and Communication.
University of the Philippines Baguio.

Carlin, George. “The Seven Words You Can Never Say on Television” from the album Class Clown. Accessed
February 5, 2918 from genius.com/George-carlin-the-seven-words-you-can-never-say-on-television-
annotated

Copi, Irving and Carl Cohen. 2009. Introduction to Logic. 13th ed. Jurong, Singappore: Pearson Education, Inc.

Dantzker, Mark and Ronald Hunter. 2012. “Chapter 4: The Language of Research” in Research Methods: For
Criminology and Criminal Justice. 3rd ed. Ontario, Canada: Jones & Barlett Learning

Dean, Neville. 2003. Logic and Language. New York City, New York: Palgrave Macmillan

Goddard, Angela. 1998. The Language of Advertising. USA and Canada: Routledge.

Kemerling, Garth. 2017. “Logic: Arguments, Language, Meaning, and Fallacies”. Accessed February 5, 2018 from
brewminate.com/logic-arguments-language-meaning-and-fallacies.

Disputes and Ambiguity

“Many disputes, whether about beliefs or about attitudes, are genuine. However, some disputes are merely verbal,
arising only as a result of linguistic misunderstanding. The terminologies by the opposing parties may have more
than one meaning or ambiguity but they may not recognize it at once. As such, to resolve the dispute and
disagreements, ambiguities must be identified.”11

In accordance to what was stated, the first step therefore to resolve a dispute is to identify them which falls
into three categories. The First is the obviously genuine dispute, in this type of dispute the parties unambiguously
disagree either in belief or attitude. An example of this Brent believes that Atok is south of La Trinidad but jairus
denies it. The disagreement is of belief thus can be resolve by some facts.

11
Copi, Irving. Introduction to Logic. Philippines: Pearson education Inc. 2009

18 | P a g e
The second cathegory of dispute is merely verbal dispute which is “dispute in which the apparent conflict
is not genuine and can be resolved by coming to agreement about how some word or phrase is to be understood.”
An example of this is that suppose someone says: “John has a new computer.” To this, someone replies, “No, he
does not, his brother gave him his old computer when he upgraded, and that computer is at least three years old.”
So does John have a “new” computer or not? Well, it depends on how we define “new.” If we mean by “new,” “not
used by a previous owner” then he does not have a new computer; if we mean by “new,” “not previously in his
possession,” then he does have a new computer, that is, it is an old computer that is new to him. Here we have a
clear case in which the two disputants have no genuine disagreement. Therefore the ambiguity in merely verbal
dispute lies in the equivocality of the term. A term is equivocal is “a term used for a number of objects with entirely
different meanings”12 to further illustrate a simple illustration follows:

A 12foot Ruler

Ruler

A head of a state with certain power

The third category is the apparently verbal but really genuine which means “a misunderstanding about use
of term maybe involved but when that misunderstanding is cleared up there remains a disagreement that goes
beyond the meaning of the words.” An example of this is:

“A: Capital punishment is repulsive to me because it violates one of the Ten


Commandments: “Thou shall not kill.” The Bible prohibits killing for the simple
reason that it is cruel and inhuman for human beings to take the life of a fellow
human being.
B: Capital punishment is far from being repulsive, for it is an honorable way for
human beings to serve justice. It certainly does not violate one of the Ten
Commandments, for when the Bible says: “Thou shall not kill” it means only that
we should not murder. Capital punishment is certainly not murder, nor is it cruel
or inhuman. Indeed capital punishment is nobly human, for it shows the high value
that human beings put on justice.”13

In this example, it seems that the problem only is how A and B defines “kill” wherein one defines it as
taking away the life of a fellow while the other one defines “kill” as murder only. By fixing the definition of the
term “kill” suppose A would just admit that the Bible prohibits murder and not simply the taking of a human life
but would A agree that capital punishment is not murder?

“A might well believe that "murder" is primarily a moral term while B might well believe that "murder" is
primarily a legal term. In this case, A might agree that capital punishment is not murder in the strict legal sense, but
may insist that it is murder in the moral sense. It may turn out that A actually believes that any taking of human life
is the moral equivalent to murder.”14

12
Gualdo, Rosendo. Logic: The Basics of Correct Reasoning. Malabon City: Mutya Publishing House Inc. 2013
13
Hall, Ronald.Logic: A Brief Introduction. http://www.stetson.edu. http://www.stetson.edu/artsci/ philosophy/
media/Chapter%203.pdf.(accesed: February 7, 2018)
14
Ibid.
19 | P a g e
This clarification of terms, however, may show us that the dispute between the two is deep and genuine and
not merely verbal. It would be genuine if a clarification of terms did not achieve a resolution to the disagreement.

In conclusion if clarifying the meaning of the terms that are used equivocally (with more than one meaning)
in a controversy, resolves the dispute, then the dispute was merely verbal; if it does not, then the dispute may be
genuine.

Definitions and Their Uses

Introduction

Good definitions are very helpful in eliminating verbal disputes (Copi and Cohen 2009). Definitions help
in providing clarity and eliminating ambiguity to set out the meaning of symbols, words, and utterances (Moore
2009). The most common way of preventing or eliminating differences in the use of languages is by agreeing on
the definition of our terms (Kemerling 2017).

It is important to remember that that we define are always symbols and not objects, or a fact. The reason is
that only symbols have the meanings that definitions may explain (Copi and Cohen 2009). An example would be
a “chair”, we can say the word “chair” because it is a symbol which has a meaning we have accorded to it, but we
cannot define an “actual chair” because it is an object (ibid 2009)

Another example would be the “balance scales” which is a popular representation in law. The balance scales
as a symbol has a meaning which could be defined, but when an actual scale is presented to us, it would become a
mere object that need not be defined.

In discussing definitions, two terms are useful, which are the definiendum and the definiens. Specifically,
the definiendum is the symbol being defined. Definiens is the symbol or group of symbols used to explain the
meaning of the definiendum (Copi and Cohen 2009).

To illustrate:

DEFINITION
DEFINIENDUM DEFINIENS
(the symbol to be defined) (the symbol or group of symbols having
the same meaning as the definiendum)

In line with this, definitions, depending on how they are used have find kinds which are stipulative, lexical,
précising, theoretical, and persuasive.

Kinds of Definition

I. Stipulative Definitions

A stipulative definition is one which deliberately assigns to a new meaning to a certain symbol (Copi and Cohen
2009). It is a proposal or a request, rather than an assertion, that this term be understood as having a certain meaning
(Moore 2009). Thus, it can neither be true or false, neither accurate nor inaccurate because it is more directive than
informative and it does not assert but stipulates (Copi and Cohen 2009)

20 | P a g e
Introducing a term by stipulation is important to avoid conflicts and disagreements arising from a word or words
which may have different meanings in a certain message (ibid 2009)

An example would be legislative definitions in a statute where words are not defined by their literal meaning
but by the way the legislature had defined it. The word “children” is defined as a human being between the stages
of birth and puberty, while in the Section 3 (h) of RA 9262 or the Anti-Violence Against Women and Children Act,
the word “children” is stipulated as “those below eighteen (18) years of age or older but are incapable of taking care
of themselves as defined under Republic Act No. 7610. As used in this Act, it includes the biological children of
the victim and other children under her care.”

Another example would be in the case of Alawi vs Alauya, AM SDC-97-2-P, February 24, 1997, contrary to
the popular belief that an “Attorney” is anyone who practice law or appear as counsel, the Supreme Court held in
the said case that an “Attorney” is a full-fledged member of the Philippine Bar, “having obtained the necessary
degree in the study of law and successfully taken the Bar Examinations, have been admitted to the Integrated Bar
of the Philippines and remain members thereof in good standing.”

II. Lexical Definitions

If stipulative definitions involve the assigning meaning, lexical definitions reports the meaning a definiendum
already has (Copi and Cohen 2009). It is an assertion regarding how a term is actually spoken, written, or used in a
certain linguistic community (Moore 2009). Thus, the truth and falsity of lexical definitions can be identified based
from its established use and meaning. A bird for example can be defined as a “warm-blooded vertebrate with
feathers” and not as a “two-footed mammal” because we already have an established definiens on what a bird is
and is not (Copi and Cohen 2009). Thus, they are true or false depending on whether they do or do not accurately
report common usage

In law, the principle of Verba Legis can be an example where words that are free from ambiguity shall be given
its literal, common, and ordinary meaning. Like in lexical definitions, there is no need to stipulate or speculate any
meaning (Mondok n.d).

III. Precising Definitions

Precising definitions, on the other hand, have for its purpose the elimination of ambiguity and vagueness (Copi
and Cohen 2009). It can be related to stipulative definitions but it does not assign new meaning to a word, it just
narrows down what a word means but still based from its established usage. It goes beyond lexical definition in that
it incorporates additional attributes in the definiens so as to narrow the scope of the term’s meaning (Moore 2009).

Precising definitions is different from lexical definitions for there are words that cannot be defined just by
consulting the dictionary such as “freedom”, “democracy”, “general welfare”, “public interest” and the like. The
existence of such vague and ambiguous words is the reason why there is a need to refine its meaning through the
said kind of definition (Swartz 2010).

An example would be what a "public disturbance” is. No definition covers all the possible circumstances that
such a term might cover. The drafters of a law cannot be expected to be able to foresee all the ways it is possible to
disturb the peace. Courts of law constantly refine the meaning of the term as new cases occur, they reduce the
vagueness – and their decisions become precedents for future reference (ibid 2010).

Another would be the term “equal protection” which does not necessarily mean affording equality for all regardless
of class, the Supreme Court had already narrowed down the definition of the term by providing its requisites in the
case of People vs Cayat, GR No. L-45987, May 5, 1939 which (1) must rest on substantial distinctions; (2) must be

21 | P a g e
germane to the purposes of the law; (3) must not be limited to existing conditions only; and (4) must apply equally
to all members of the same class.

IV. Theoretical Definitions

Theoretical definitions, on the other hand, are those which serve as a compressed summary or recapitulation of
a theory. It is the comprehensive grasp of a theory in which the term is a key element. It is a definition which
encapsulates a larger understanding (Copi and Cohen 2009).

In theoretical definitions, take what we know about a given subject, concept, or thing, and attempt to define it
to the best of our current knowledge. A theoretical definition of “justice,” for example, is not simply an attempt to
point out what justice is or report on how people happen to use the word. Instead, it is an attempt to create a theory
which argues for a particular conception of justice. An example would be Socrates’ struggle in finding the correct
definition in Plato’s Republic (Cline 2017; Copi and Cohen 2009)

Theoretical definitions emphasizes that there are terms that cannot be defined by simply explaining it through
a set of synonymous words (ibid 2009). Unlike the other kinds of definitions where the meaning is assigned,
asserted, or narrowed, here, meanings are sought.

V. Persuasive Definitions

Lastly, persuasive definitions are those utilized to resolve disputes by influencing attitudes or stirring emotions
(Copi and Cohen 2009). They are very common on political arguments. It is like adding emotive color to a lexical
definition (Swartz 2010).

One example would be the definition given by Abraham Lincoln to “democracy” which is “a government of
the people, by the people, and for the people”. Evidently, such words are not the literal definition of democracy but
it was utilized as a persuasive meaning of it.

Other examples would be defining “capitalism" as "the system of economic distribution which places the
ownership of the means of production in the hands of an entrenched few" and "government" as "the subjugation of
the masses by bureaucrats" (Swartz 2010).

Sources: (Chapter 3.4)

Alawi vs Alauya, AM SDC-97-2-P, February 24, 1997. Accessed February 5, 2018 from
sc.judiciary.gov.ph/jurisprudence/1997/feb1997/am_sdc_97_2_p.htm

Copi, Irving and Carl Cohen. 2009. Introduction to Logic. 13th ed. Jurong, Singappore: Pearson Education, Inc.

Kemerling, Garth. 2017. “Logic: Arguments, Language, Meaning, and Fallacies”. Accessed February 5, 2018 from
brewminate.com/logic-arguments-language-meaning-and-fallacies/

Moore, James. “The Logic of Definition”, Defence R&D Canada-Toronto Technical Note 2009-082 (2009).
Accessed February 5, 2018 from cradpdf.drdc-rddc.gc.ca/PDFS/unc86/ p531786.pdf

Mondok, Nestor. (n.d). “Statutory Construction: A Compendium from the Books of Rodriguez, Martin, and
Agpalo.” College of Law, University of the Cordilleras

22 | P a g e
People vs Cayat, GR No. L-45987, May 5, 1939. Accessed February 5, 2018 from
lawphil.net/judjuris/juri1939/may1939/gr_l-45987_1939.html

Republic Act 9262, 2004, “An Act Defining Violence Against Women And Their Children, Providing For
Protective Measures For Victims, Prescribing Penalties Therefore, And For Other Purposes”. Accessed
February 5, 2018 from lawphil.net/statutes/repacts/ ra2004/ra_9262_2004.

Swartz, Norman. 2010. “Definitions, Dictionaries, and Meanings”. Accessed February 5, 2018 from
http://www.sfu.ca/~swartz/definitions.htm

The Structure of Definitions

To understand the meaning of terms, we should examine their fundamental qualities or properties which
are the extension and intention.

“Extentional meaning of a general term is the collection of the objects that constitutes the extension.”15 Let
us take the example of a plant. When we hear the word plant, we are able to apply it to flower, cabbage, tree, grass,
vegetables and etc. With that we the idea of a plant is extended to those words. To clarify that, understand the
diagram below:

Flower
Cabbage
Plant(idea) Tree all these are plants
Grass
Vegetables

Extension therefore is defined as “that property of a term by which such term or concept is applied or
extended to other things.”16 In other words it is the collection of things to which the concept is used or it refers to
all things which the term is applicable.

Intention on the other hand is “the set of attributes shared by all and only those objects to which a general
term refers.”17 For example, a set of characteristic is listed: four-footed, canine, alive, barking and wags his tail.
With those set, we can connect and associate them and come up with an idea that what those terms in set is referring
to a dog. In other words intention is the “sum total of the different characteristic notes by which an object is
identified as such.”18 Moreover an object is not identified by one characteristic only, there should be other criteria
related to one another characterizing a thing. Therefor if there is only one characteristic or attribute mentioned it is
incomprehensible.

Another explanation of it is that “Intension and extension, in logic, correlative words that indicate the
reference of a term or concept: “intension” indicates the internal content of a term or concept that constitutes its

15
Copi, Irving. Introduction to Logic. Philippines: Pearson education Inc. 2009

16
Gualdo, Rosendo. Logic: The Basics of Correct Reasoning. Malabon City: Mutya Publishing House Inc. 2013

17
Copi, Irving. Introduction to Logic. Philippines: Pearson education Inc. 2009

18
Gualdo, Rosendo. Logic: The Basics of Correct Reasoning. Malabon City: Mutya Publishing House Inc. 2013

23 | P a g e
formal definition; and “extension” indicates its range of applicability by naming the particular objects that it
denotes.”19

Although they are correlative words they are not reciprocal ones. While intension determines extension, the
reverse is not true. For example in the book which is the “equilateral triangle” (intention) and all those objects with
the shape of “equilateral triangle” must be in the same class, which is the extension.

Extension does not determine intension because two different intension can have the same extension. Also
when there is only one extension mentioned, if we are going to identify its intention alone, there would be too much
intention. Therefore the more extension mentioned, there would be lesser intention and vice versa. To understand
further an illustration below might help:

Intension Extension
Substance Spirits, Minerals, Plants, Animals, Men Etc.
Material Substance Minerals, Plants, Animals, Men Etc.
Living Material Substance Plants, Animals, Men
Sentient Living Material Substance Animals, Men
Rational Sentient Living ,Material Substance Men

“the clarity and preciseness of concept depends upon the sum total of notes included in its intension. The
more notes of a concept has, the clearer and definitive does it become and, therefore, the fewer inferiors will it
apply.”20

A. EXTENSION AND DENOTATIVE DEFINITIONS

“Denotative definition employs techniques that identify the extension of the term being defined.” Or it is
simply the listing of all the term that id applied to a general term. Since it is impossible to list all the terms applied
to a general term, an easier way to denote them is by pointing at them which is called ostensive definition. Ostensive
definition is done “defining a term by making a gesture at the object which can define it.”21 Meaning it is pointing
at the actual object that one is referring to. But there are limitations to this because “gestures have a geographic
limitation; one can only indicate what is visible.”22 Moreover there can be ambiguity to which what one is pointing
at. One can interpret the gesture as pointing at the colour, shape, and surface and so on.

The quasi-ostensive definition can resolve that ambiguity because “quasi-ostensive definition adds a
descriptive phrase to the difinens.” in other words, aside from the gesture it is done by providing sets of
characteristics, external features and properties of an objects one is pointing at.

B. Intension and Intensional Definition

19
The Editors of Encyclopædia Britannica, Intension and extension. www.britannica.com. https://www.
britannica.com/topic/intension. (accessed: February 7, 2018)
20
Gualdo, Rosendo. Logic: The Basics of Correct Reasoning. Malabon City: Mutya Publishing House Inc. 2013

21
Copi, Irving. Introduction to Logic. Philippines: Pearson education Inc. 2009

22
Ibid.
24 | P a g e
Although intension was already defined, it should still be distinguished. There are three different senses of
intension which are: subjective, objective and conventional.

Subjective intension refers to the speaker’s belief of the definition of a term. It differs to other individuals
own definition of a term and the individual may change their definition of a thing as time passes. Meaning,
subjective intension only depends from the personal opinion of each person.

“The objective definition is the total set of characteristic shared by all the object in the word’s extension.”23
It is simply finding what is common from the general terms. As Copi and Cohen stated, no one is omniscient,
therefore no one is able to understand all the attributes shared by the objects denotes by general terms, and therefore
objective intension cannot be the public meaning whose explanation we seek to give.

Since people communicate and most of the people understand each other, therefore there should be a
common intension that is available. This is conventional intension, “an intension which uses terms that already has
a stable meaning and which is commonly understood.”24 Meaning it is a generally accepted principle or term.

There are also techniques, using intension, for defining terms. One of these is the synonymous definition,
which depends greatly on the dictionary in defining terms. Synonymous definition is a way of defining a term by
giving other terms with the same meaning as the definiendum. In other words, a definiendum is defined by words
synonymous to it. But there are some limitations to it, one is that all words does not have exact synonym and
therefore it may mislead the true meaning of the term. Another limitation is that when a concept is not fully
understood finding its synonym would also be a problem.

“Operational definition defines the definiendum by tying it to particular set of action”25 and “defines
concepts and labels by the way they are measured.”26 For example, an operational definition of weight could be:
how much a spring stretches when you hang something from it, or how many pennies it takes to balance the weight
of something. In other words, it is like setting rules wherein those who satisfies those rules are the only those who
are included in that set.

The technique is definition by genus and difference wherein the next chapter is entirely devoted to it.

Definition by Genus and Difference

Introduction

A definition by genus and species is one which defines a term through their classes and subclasses. It is
devised to comprehend the complexity and analysability of some terms (Copi and Cohen 2009). The definition by

23
Ibid.

24
Ibid.

25
Ibid.

26
Operational Definitions. http://www.indiana.edu. http://www.indiana.edu/~p1013447/dictionary/ operat.htm.(accessed
February 7, 2018)
25 | P a g e
genus and difference is usually used to define general terms. It is a method where the meaning of a term is broken
down into constituent elements (Moore 2009).

Specifically, a genus is a class of membership having some common characteristic. It can also be divided
into subclasses, which are referred to as species. Being under the same subclass, the characteristic that serves to
distinguish a member of a specie from others is called specific difference (Copi and Cohen 2009). There are two
steps required in defining a term by genus and difference. First, the genus must be named. Second, the specific
difference of the member of the species from all other species in that genus must be identified (ibid 2009; Moore
2009).

For example, the genus “polygon” is composed of subclasses “triangle, quadrilateral, pentagon, hexagon,
and so on”. All the subclasses are polygon, however, each are different from each other as to their number of sides.
As an example, the specific difference of a “pentagon” from all the other members of the subclass is that it has five
sides. Thus, the definition of “pentagon” is “polygon with five sides”.

Another example, the genus “parts of a statute” is composed of subclasses “title, preamble, body,
interpretative clause, repealing clause, saving clause, and others”. Their common characteristic is that they are all
parts of a statute but the contents of one subclass are different from the others. Thus, a “preamble” is a part of a
statute which recites the “purpose, reason, or occasion for making the law” (Agpalo 2009, 10)

There are, however, two limitations of definitions by genus and difference. First, it does not apply to words
which possess attributes which are “unanalyzable”. An example would be “sensed qualities of specific shades of
color”. Second, it does not apply when the attributes of a term is considered “universal”. Some examples of this
term are “being”, “entity”, or “existent”. They cannot be defined in a broader genus or specific difference because
they belong to the universal class, the very highest class (Copi and Cohen 2009).

It is not enough that a term is defined by its genus and difference. The definition must be a good definition.
This requires a selection of the most appropriate genus and the identification of the most significant specific
difference for that term. To achieve this, Copi and Cohen (20009) enumerated five grounds or rules.

1. A definition should state the essential attributes of the species.


- Under this rule, the term must be defined based from its conventional intension, and not from its subjective
or objective intension. The conventional intension may concern the origin of the term, the relation of the
term with the other members of the class, or the meaning or use which the members of that class normally
put (ibid 2009).
- Note that what this rule entails is an “essential” attribute and not just any attribute. Thus, “governors” and
“senators” are not defined by their mental or physical characteristics but by their relations with the citizens.
A “shoe”, cannot be defined based from what it is made, but based from the use to which it is put, “as an
outer covering of the foot”

2. A definition must not be circular.


- A definiendum is defined through its definiens. However, if the definiendum itself appears in the definiens,
the definition will fail its purpose to explain the meaning of the definiendum. This is the characteristic of a
circular definition (Copi and Cohen 2009). Since a circular definition uses the term being defined as part
of its own definition, it cannot provide any useful information (Kemerling 2017)

26 | P a g e
- It is therefore wrong to define a “gambler” as “a person who gambles compulsively”; a “cordless phone”
as a “telephone without a cord”; or “stress” as a “specific morphological, biochemical, physiological, and/or
behavioural change experienced by an organism in response to a stressful event or stressor”.

3. A definition must neither be too broad or too narrow.


- Basically, this rule posits that the definiens must not define things more than what are denoted by the
definiendum, nor shall it define fewer things than which is denoted (Copi and Cohen 2009). Thus, a good
definition will apply to exactly the same things as the term being defined, no more and no less (Kemerling
2017).
- Copi and Cohen (2009) gave as an example Plato and his successors in the Academy of Athens on the
definition of “man” as a “featherless biped,” their critic Diogenes, plucked it and threw it over the wall of
the Academy. There was a featherless biped but no man! The definiens is too broad. The attribute of “having
broad nails” is added in the definition but made it too narrow.
- Kemerling (2017) cited as an example the definition of a “bird”. He said that it must not be defined as a
“warm-blooded animal” because it is too broad and such attribute also describes dogs, horses, and other
mammals. A bird cannot likewise be defined as a “feathered-egg laying animal” because it is too narrow
and such attribute excludes male birds.

4. Ambiguous, obscure, or figurative language must not be used in the definition.


- Ambiguity defeats the purpose of the definition to explain the definiendum. An obscure language, on the
other hand, uses language that does not help a person in learning a term. The terms used to define the
definiendum are words that only professionals are familiar with. Likewise, a figurative language can convey
a “feeling” about a word but it does not provide a clear explanation.
- An example is when Dr. Samuel Johnson, in his “Dictionary of the English Language (1755), defined “net”
as “anything reticulated or decussated equal distances with interstices between the intersections”.
- From Michel de Certeau’s “Walking in the City”, he defined “walkers” as “ordinary practitioners who live
down below the thresholds at which visibility begins.”
- For figurative language, an example is “happiness is a warm puppy”, it conveys a feeling but was not able
to explain the term well.

5. A definition should not be negative when it can be affirmative.


- A definition must be expressed based on what it means an not on what it does not mean (Copi and Cohen
2009)
- An “honest person” cannot be defined as “someone who rarely lies” but as “someone who always say the
truth”.
- However, Copi and Cohen (2009) also pointed out that there are some terms which are essentially negative
which requires negative definition. The word “bald” means “the state of having no hair on one’s head”. An
“orphan” means a “child who does not have parents.”

Sources: (Chapter 3.6)

Agpalo, Ruben. 2009. Statutory Construction. 6th ed. Manila: REX Printing Company Inc.

Copi, Irving and Carl Cohen. 2009. Introduction to Logic. 13th ed. Jurong, Singappore: Pearson Education, Inc.

27 | P a g e
De Certeau, Michel. “Walking in the City”. Translated by Steven Rendall Accessed February 6, 2018 from
faculty.georgetown.edu/irvinem/theory/DeCerteau-Practice-Excerpts.pdf

Kemerling, Garth. 2017. “Logic: Arguments, Language, Meaning, and Fallacies”. Accessed February 5, 2018 from
brewminate.com/logic-arguments-language-meaning-and-fallacies/

Moore, James. “The Logic of Definition”, Defence R&D Canada-Toronto Technical Note 2009-082 (2009).
Accessed February 5, 2018 from cradpdf.drdc-rddc.gc.ca/PDFS/unc86/ p531786.pdf

28 | P a g e
CHAPTER 4: FALLACIES

WHAT IS FALLACY?

Fallacy is derived from the term fallere which means to deceive. It is a type of argument that seems to be correct,
but contains a mistake in reasoning. The reasoning is incorrect which is illogical, deceptive, and misleading. Any
argument which is mistake is said to commit a fallacy.

Fallacies are defects in an argument, which cause an argument to be invalid, unsound or weak.

WHAT MAKES IT FALLACIOUS

An argument for it to be valid, the premise must be is true and the conclusion must also be true.

It will only become fallacious when the reasoning is incorrect when the premises fails to support the conclusion.
Fallacy undermines the conclusion when it is not relevant to the conclusion, when it undermines or does not support
the conclusion, when the premises makes the argument ineffective and weak, when the premises of the argument is
obscure or ambiguous.

IMPORTANCE

• To avoid thinking incorrectly and avoid being deceived by others

• Knowledge of the principles and laws of argumentation is clarified and strengthened by the awareness of
the various fallacies

• The ability to argue correctly is developed by the knowledge of incorrent ways of thinking

• The study of the fallacies enables the student of Logic to master the rules of categorical syllogism

• The ability to identify fallacies helps prevent one from confusion and ensures him to correct decision
making.

Types of Fallacies

Formal Fallacies:

A formal fallacy is a defect which can be identified merely by looking at the logical structure of an argument rather
than any specific statements. It is only found only in deductive arguments with identifiable forms. One of the things
which makes them appear reasonable is the fact that they look like and mimic valid logical arguments, but are in
fact invalid. Where the premises are correct but the conclusion is false.

Example of a valid formal fallacy:

1. It is raining outside, the street is wet

It is raining outside

=Therefore, the street is wet

Examples of an invalid formal fallacy:

1. All humans are mammals.

29 | P a g e
All cats are mammals
Therefore, All humans are cats
2. It is raining outside, the street is wet
The street is wet
Therefore, it is raining outside

Informal fallacies are defects which can be identified only through an analysis of the actual content of the argument.

Classification of informal Fallacy:

Fallacies of Relevance
In these fallacies, the premises of the argument are simply not relevant to the conclusion. Because the connection
is missing, the premises offered cannot possibly establish the truth of the conclusion.

Fallacies according to fallacies of relevance

1. The appeal to emotion


2. The appeal to the populace
3. The red herring
4. The straw man
5. The attack on the person
6. The appeal to force
7. Missing the point (irrelevant conclusion)

Appeal to Emotion
Variety under appeal to emotion:

• Appeal to Pity or ad misericordiam

- “merciful heart”. Pity is often admirable to human response.

Appeal to the populace or argumentum ad populum

– An informal fallacy commited when the support offered for some conclusion is an inappropriate appeal to
the multitude.

– This fallacy may often use by “demagogue” and propagandist.


– Instead using of evidence or rational arguments, speaker relies on to excite enthusiasm for or against some
cause.

• Bandwagon Fallacy
- Because that is everyone else is doing, you are well advised to join, buy, support, etc. “jump on
the bandwagon”.

30 | P a g e
The Red Herring
-An informal fallacy committed when some distraction used to mislead and confuse.
• Argument whose effectiveness lies in distraction;

• Attention is deflected; Deliberately misleading trail

The Straw Man Fallacy


- Committed when the position of one's opponent is misrepresented and that distorted position is now the object of
the attack.
- Is a variety of red herring, because it also introduces a distraction from the real disputes, however the distraction
is of a particular kind; “ it is an effort to shift the conflict from its original complexity into a different conflict, other
than those originally in dispute.

• Difference between red herring and strawman fallacy;


Example:

Straw Man: "So you're saying I evolved from a monkey? Then how come I don't throw feces and climb trees?"
Evolution doesn't make that claim, but by claiming that it does, the arguer is trying to beat down a much easier
argument.

Red Herring: "So you believe in evolution? I'll bet your parents would be disappointed in you." The arguer is
distracting from the issue by bringing his opponent's parents into this - they have no impact on whether or not the
theory is correct.

Argument against the Person or argument ad hominem


– An argument in which the thrust is directed, not at a conclusion, but at some person who defend the
conclusion in dispute.
– Rather attacking the substance of some position, one attacks the person of its advocate.
2 major forms of argument ad hominem:

• The abusive

• The circumstantial

The Abusive
- Attacking the person making the argument, rather than the argument itself, when the attack on the person is
completely irrelevant to the argument the person is making.

The Circumstantial
Tu quoque or “You too”, “look who's talking'

- when the circumstance of the speaker are used not merely as ground for attack – but used rather in a plainly
negative spirit.

– the substance f the fallacy is to contend that you (first party) are just bad as I am, just guilty of whatever it
is you complained about. It may be true that the first party is guilty of the conduct in question, but that does not
support the innocence of the second party, which is the issue in the argument at hand.

The Appeal to Force or argument ad baculum

- Establishing proposition as true by resorting to force.


31 | P a g e
- Threat or strong-arm methods to coerce one's opponent can hardly be considered arguments at all.

Missing the Point or Ignoratio Elenchi

- Committed when on refutes, not the thesis one's interlocutory is advancing, but some different thesis that one
mistakenly imputes to him or her.

- Is a mistake that is made in seeking to refute another's argument.

Fallacies of Defective Induction


- The mistake arises from the fact that the premises of the argument, although relevant to the conclusion , are so
weak and ineffective that relying on them is a blunder.

- Premises are relevant and yet are wholly inadequate

Fallacies under defective induction:

1. Argument from Ignorance

2. The appeal from inappropriate authority

3. False Cause

4. Hasty Generalization

Argument from Ignorance or argument ad ignorantiam


- The assumption of a conclusion or fact based primarily on lack of evidence to the contrary. Usually best described
by, “absence of evidence.”

- Conclusion is supported by an illegitimate appeal to ignorance, as when it is supposed that something is likely to
be true because we cannot prove that it is false

“X is true because you cannot prove that X is false.” ; “X is false because you cannot prove that X is true.”

Appeal to Inappropriate Authority or argument ad verecundiam


- Appeal to authority is illegitimate because the authority appealed to has no special claim to expertise on the matter
in question.

- Using an authority as evidence in your argument when the authority is not really an authority on the facts relevant
to the argument. As the audience, allowing an irrelevant authority to add credibility to the claim being made.

32 | P a g e
False Cause
- Argument non causa pro causa

- An informal fallacy in which the mistake arises from accepting as the cause of an event what is not really its cause

Hasty Generalization
- An informal fallacy in which a principle that is true of a particular case is applied, carelessly or deliberately, to the
great run of cases.

- We commit when we drawn conclusion about all the persons or things in a given class on the basis of our
knowledge about only one or few of members of that class

Why Fallaciuos argument consider as part of bad reasoning?

Logic may well have begun, historically, when people first noticed that there are patterns or approaches to reasoning
that cannot be trusted. In Plato’s dialogues, you often see references to people who “make the weaker argument
appear the stronger,” i.e., people who manipulate language so skillfully that they can convince others of things that
they should not really be convinced of.

More precisely, Logic is more concerned with the process or the pattern of reasoning than it is with the thing
believed: Logic is concerned with whether the reasoning is good or not, but a given belief might be better supported
by one argument than by another.
A fallacious argument may be deceptive by appearing to be better than it really is.

Fallacies of Presumption

In fallacious arguments of this kind the premises may indeed be relevant to the conclusion drawn, but that relevance
is likely to flow from the tacit supposition of what has not been given support and may even be unsupportable. The
presumption often goes unnoticed. To expose such a fallacy it is therefore usually sufficient to call attention to the
smuggled assumption, or supposition, and to its doubtfulness or its falsity.

Three common fallacies are included in this category.

P1. Accident Circumstances alter cases.

Accident is the fallacy that arises when we move carelessly or unjustifiably from a generalization to some particulars
that it does not in fact cover

For example, there is a general principle in law that hearsay evidence—statements made by a third party outside
court—may not be accepted as evidence in court; this is the “hearsay rule,” and it is a good rule. However, when
the person whose oral communications are reported is dead, or when the party reporting the hearsay in court does
so in conflict with his own best interest, that rule may not apply. Indeed, there is hardly any rule or general principle
that does not have plausible exceptions, and we are likely to argue fallaciously if we reason on the supposition that
some rule applies universally.

33 | P a g e
P2. Complex Question (Plurium Interrogationum)

One of the most common fallacies of presumption is to ask a question in such a way as to presuppose the truth of
some conclusion that is buried in the question.

Thus an essayist recently asked: With all of the hysteria, all of the fear, all of the phony science, could it be that
man-made global warming is the greatest hoax ever perpetrated on the American people?

Such a statement assumes that much of the evidence supporting global warming is unreliable or “phony.”

Or a homeowner might ask, regarding a proposed increase in the property tax, “How can you expect the majority
of the voters, who rent but don’t own property and don’t have to pay the tax, to care if the tax burden of others is
made even more unfair?”—assuming both that the burden of the proposed tax is unfair and that those who rent
rather than own their own homes are not affected by tax increases on property.

Because assumptions like these are not asserted openly, the questioners evade the need to defend them forthrightly.
The complex question is often a deceitful device. The speaker may pose some question, then answer it or strongly
suggest the answer with the truth of the premise that had been buried in the question simply assumed.

A letter writer asks, “If America’s booming economy depends on people’s using consumer credit beyond their
means, thus creating poverty, do we really have a healthy economy?”29 But the role and the results of consumer
credit remain to be addressed.

For example: Lawyer: The figures seem to indicate that your sales increased as a result of these misleading
advertisements. Is that correct? Witness: They did not! Lawyer: But you do admit, then, that your advertising was
misleading. How long have you been engaging in practices like these? When a question is complex, and all of its
presuppositions are to be denied, they must be denied individually. The denial of only one presupposition may lead
to the assumption of the truth of the other. In law, this has been called “the negative pregnant.” Here is an illustration
from a notorious murder trial: Q: Lizzie, did you not take an axe and whack your mother forty times, and then
whack your father forty-one times when faced with the prospect of cold mutton stew? A: Not true. We were to eat
Brussels sprouts fondue that day.

P3. Begging the Question (Petitio Principii or “circular argument”)

It is the mistake of assuming the truth of what one seeks to prove. The “question” in a formal debate is the issue
that is in dispute; to “beg” the question is to ask, or to suppose, that the very matter in controversy be conceded.
The Latin name of the fallacy, for which “begging the question” is the translation, is petitio principii, so each
instance of it is called a petitio.

The logical mistake arises because it is obscured, even from its author, by the language used. Logician Richard
Whately used this classic example of a deceptive petitio: To allow every man unbounded freedom of speech must
always be, on the whole, advantageous to the state; for it is highly conducive to the interests of the community that
each individual should enjoy a liberty, perfectly unlimited, of expressing his sentiments.

Another illustration, equally fallacious, is found in this claim by a sixteenth-century Chinese philosopher: There is
no such thing as knowledge which cannot be carried into practice, for such knowledge is really no knowledge at
all. This fallacy, like the fallacy of missing the point, is often a mistake that is not recognized by the author of the
passage.

They are relevant; indeed, they prove the conclusion, but they do so trivially—they end where they began. A petitio
principii is always technically valid, but always worthless.
34 | P a g e
Fallacies of Ambiguity

The meaning of words or phrases may shift as a result of inattention, or may be deliberately manipulated within the
course of an argument. A term may have one sense in a premise but quite a different sense in the conclusion. When
the inference drawn depends on such changes it is, of course, fallacious. Mistakes of this kind are called fallacies
of ambiguity or sometimes “sophisms.” The deliberate use of such devices is usually crude and readily detected—
but at times the ambiguity may be obscure, the error accidental, the fallacy subtle. Five varieties are distinguished
here.

A1. Equivocation

Most words have more than one literal meaning, and most of the time we have no difficulty keeping those meanings
separate by noting the context and using our good sense when reading and listening. Yet when we confuse the
several meanings of a word or phrase—accidentally or deliberately—we are using the word equivocally.

Carroll’s account of the adventures of Alice in Through the Looking-Glass is replete with clever and amusing
equivocations. One of them goes like this: ”Who did you pass on the road?” the King went on, holding his hand out
to the messenger for some hay. ”Nobody,” said the messenger. ”Quite right,” said the King; “this young lady saw
him too. So of course Nobody walks slower than you.” The equivocation in this passage is rather subtle. As it is
first used here, the word “nobody” simply means “no person.” Reference is then made using a pronoun (“him”), as
though that word (“nobody”) had named a person. When subsequently the same word is capitalized and plainly
used as a name (“Nobody”), it putatively names a person having a characteristic (being passed on the road) derived
from the first use of the word. Equivocation is sometimes the tool of wit— and Lewis Carroll was a very witty
logician.

* Equivocal arguments are always fallacious, but they are not always silly or comical, as in the example discussed
in the following excerpt: There is an ambiguity in the phrase “have faith in” that helps to make faith look respectable.
When a man says that he has faith in the president he is assuming that it is obvious and known to everybody that
there is a president, that the president exists, and he is asserting his confidence that the president will do good work
on the whole.

Thus the phrase “to have faith in x” sometimes means to be confident that good work will be done by x, who is
assumed or known to exist, but at other times means to believe that x exists. Which does it mean in the phrase “have
faith in God”? It means ambiguously both; and the self evidence of what it means in the one sense recommends
what it means in the other sense. If there is a perfectly powerful and good god it is self-evidently reasonable to
believe that he will do good. In this sense “have faith in God” is a reasonable exhortation. But it insinuates the other
sense, namely “believe that there is a perfectly powerful and good god, no matter what the evidence.” Thus the
reasonableness of trusting God if he exists is used to make it seem also reasonable to believe that he exists.

One kind of equivocation deserves special mention. This is the mistake that arises from the misuse of “relative”
terms, which have different meanings in different contexts.

*This passage very probably inspired David Powers, who formally changed his name to Absolutely Nobody and
ran as an independent candidate for lieutenant governor of the state of Oregon. His campaign slogan was “Hi, I’m
Absolutely Nobody. Vote for me.” In the general election of 1992, he drew 7 percent of the vote.

The fallacy is one of equivocation with respect to the relative term “small.” Not all equivocation on relative terms
is so obvious, however. The word “good” is a relative term and is frequently equivocated on when it is argued, for
example, that so-and-so is a good general and would therefore be a good president, or that someone is a good scholar
and is therefore likely to be a good teacher.
35 | P a g e
A2. Amphiboly

The fallacy of amphiboly occurs when one is arguing from premises whose formulations are ambiguous because
of their grammatical construction. The word “amphiboly” is derived from the Greek, its meaning in essence being
“two in a lump,” or the “doubleness” of a lump. A statement is amphibolous when its meaning is indeterminate
because of the loose or awkward way in which its words are combined. An amphibolous statement may be true in
one interpretation and false in another. When it is stated as premise with the interpretation that makes it true, and a
conclusion is drawn from it on the interpretation that makes it false, then the fallacy of amphiboly has been
committed. In guiding electoral politics, amphiboly can mislead as well as confuse. During the 1990s, while he sat
in the U.S. House of Representatives as a Democrat from California, Tony Coelho is reported to have said: “Women
prefer Democrats to men.” Amphibolous statements make dangerous premises—but they are seldom encountered
in serious discourse. What grammarians call “dangling” participles and phrases often present amphiboly of a striking
sort, as in “The farmer blew out his brains after taking affectionate farewell of his family with a shotgun.” Some
tidbits in The New Yorker make acid fun of writers and editors who overlook careless amphiboly: Dr. Salick
donated, along with his wife, Gloria, $4.5 million to Queens College for the center. Gloria is tax-deductible.

A3. Accent

A fallacy of ambiguity that occurs when an argument contains a premise that relies on one possible emphasis of
certain words, but the conclusion relies on a different emphasis that gives those same words a different meaning

. In English today there are not very many cases in which changing the accent in a word changes the meaning of
the word. Three of the most common are increase and increase, insult and insult, record and record. These pairs of
words accented differently mean different parts of speech—one member of each pair is a noun, the other a verb—
and thus it is unlikely that fallacious argument would now arise from those differently accented words.

Consider, as an illustration, the different meanings that can be given to the statement We should not speak ill of our
friends. When the sentence is read without any special stress on one of its words, this injunction is surely one with
which we would all agree. But, if the sentence is read with stress on the word “friends,” we might understand it to
suggest that speaking ill of those who are not our friends is not precluded.

Fallacy of accent may be construed broadly to include the distortion produced by pulling a quoted passage out of
its context, putting it in another context, and there drawing a conclusion that could never have been drawn in the
original context. Quoting out of context is sometimes done with deliberate craftiness. In the presidential election
campaign of 1996 the Democratic vice presidential candidate, Al Gore, was quoted by a Republican press aide as
having said that “there is no proven link between smoking and lung cancer.” Those were indeed Mr. Gore’s exact
words, uttered during a television interview in 1992. But they were only part of a sentence. In that interview, Mr.
Gore’s full statement was that some tobacco company scientists “will claim with a straight face that there is no
proven link between smoking and lung cancer. . . . But the weight of the evidence accepted by the overwhelming
preponderance of scientists is, yes, smoking does cause lung cancer.”37 Fallacies 147 Fallacies The omission of the
words “will claim with a straight face”—and of Gore’s express conviction that cancer is caused by smoking—
unfairly reversed the sense of the passage from which the quotation was pulled. The argument suggested by the
abbreviated quotation, having the apparent conclusion that Mr. Gore seriously doubts the causal link between
smoking and cancer, is an egregious example of the fallacy of accent. Deliberate distortion of this kind is not rare.

Even the literal truth can be used, by manipulating its placement, so as to deceive with accent. Disgusted with his
first mate, who was repeatedly inebriated while on duty, the captain of a ship noted in the ship’s log, almost every
day, “The mate was drunk today.” The angry mate took his revenge. Keeping the log himself on a day when the
captain was ill, the mate recorded, “The captain was sober today.”

36 | P a g e
A4. Composition The term fallacy of composition is applied to both of two closely related types of mistaken
argument. The first may be described as reasoning fallaciously from the attributes of the parts of a whole to the
attributes of the whole itself. A flagrant example is to argue that, because every part of a certain machine is light in
weight, the machine “as a whole” is light in weight. The error here is manifest when we recognize that a very heavy
machine may consist of a very large number of lightweight parts. Not all examples of fallacious composition are so
obvious, however. Some are misleading. One may hear it seriously argued that, because each scene of a certain play
is a model of artistic perfection, the play as a whole is artistically perfect. This is as much a fallacy of composition
as to argue that, because every ship is ready for battle, the whole fleet must be ready for battle. The other type of
composition fallacy is strictly parallel to that just described. Here, the fallacy is reasoning from attributes of the
individual elements or members of a collection to attributes of the collection or totality of those elements. For
example, it would be fallacious to argue that because a bus uses more gasoline than an automobile, all buses use
more gasoline than all automobiles. This version of the fallacy of composition turns on a confusion between the
“distributive” and the “collective” use of general terms

A5. Division

The first kind of division consists of arguing fallaciously that what is true of a whole must also be true of its parts.
To argue that, because a certain corporation is very important and Mr. Doe is an official of that corporation, therefore
Mr. Doe is very important, is to commit the fallacy of division. This first variety of the division fallacy is committed
in any such argument, as in moving from the premise that a certain machine is heavy, or complicated, or valuable,
to the conclusion that this or any other part of the machine must be heavy, or complicated, or valuable. To argue
that a student must have a large room because the room is located in a large dormitory would be still another instance
of the first kind of fallacy of division.

The second type of division fallacy is committed when one argues from the attributes of a collection of elements to
the attributes of the elements themselves. To argue that, because university students study medicine, law,
engineering, dentistry, and architecture, therefore each, or even any, university student studies medicine, law,
engineering, dentistry, and architecture is to commit the second kind of division fallacy. It is true that university
students, collectively, study all these various subjects, but it is false that university students, distributive, do so.

37 | P a g e
CHAPTER 5: CATEGORICAL PROPOSITIONS

The Theory of Deduction

“Deduction" refers to the process of advancing a deductive argument, or going through a process of reasoning
that can be reconstructed as a deductive argument.

Because deductive arguments are those in which the truth of the conclusion is thought to be completely
guaranteed and not just made probable by the truth of the premises, if the argument is a sound one, the truth of the
conclusion is "contained within" the truth of the premises; i.e., the conclusion does not go beyond what the truth
of the premises implicitly requires. For this reason, deductive arguments are usually limited to inferences that
follow from definitions, mathematics and rules of formal logic.

Deductive reasoning is a basic form of valid reasoning. Deductive reasoning, or deduction, starts out with a
general statement, or hypothesis, and examines the possibilities to reach a specific, logical conclusion. The
scientific method uses deduction to test hypotheses and theories. "In deductive inference, we hold a theory and
based on it we make a prediction of its consequences. That is, we predict what the observations should be if the
theory were correct.

Deductive reasoning usually follows steps. First, there is a premise, then a second premise, and finally an
inference. A common form of deductive reasoning is the syllogism, in which two statements — a major premise
and a minor premise — reach a logical conclusion. For example, the premise "Every A is B" could be followed by
another premise, "This C is A." Those statements would lead to the conclusion "This C is B." Syllogisms are
considered a good way to test deductive reasoning to make sure the argument is valid.

DEDUCTIVE VS INDUCTIVE
1. The premise provide a guarantee of the 2. The premises provide reasons supporting
truth of the conclusion. probable truth of the conclusion.
3. Form general to specific. 4. From specific to general

Classes and Categorical Propositions

*Categorical propositions are the building blocks of categorical logic, which goes back to Aristotle’s fundamental
work in the 4th century BC. Aristotle developed his logic as a foundation for science. His system of logic was
based on classification: what is the relationship between one class of objects and another? Thus, Classical logic
deals mainly with arguments based on the relations of classes of objects to one another.

*Everyone can see immediately that two classes can be related in at least the following three ways:

1. All of one class may be included in all another class. Thus, the class of all dogs is wholly included or
wholly contained) in the class of all mammals.
2. Some, but not all, of the members of one class may be included in another class. Thus, the class of all
athletes is partially included (or partially contained) in the class of all females.
3. Two classes may have no members in common. Thus, the class of all triangles and the class of all circles
may be said to exclude one another.

38 | P a g e
These three relations may be applied to classes, or categories, of every sort. In a deductive argument we present
propositions that state the relations between one category and some other category. The propositions with which
such arguments are formulated are therefore called categorical propositions.

A proposition that relates two classes, or categories, is called a categorical proposition. By a class we mean a
collection of all objects that have some specified characteristics in common. The two classes in question are
denoted by the subject term and the predicate term. Categorical propositions assert that either all or part of the
class denoted by the subject term is included in or excluded from the class denoted by the predicate term.

Thus,

All stand-up comedians are witty persons.

No stand-up comedians are witty persons.

Some stand-up comedians are witty persons.

Some stand-up comedians are not witty persons.

There are exactly four types of categorical propositions.

Those that assert:

1) The whole subject class is included in the predicate class;

2) The whole subject class is excluded from the predicate class;

3) Part of the subject class is included in the predicate class;

4) Part of the subject class is excluded from the predicate class.

Thus there are four standard forms of categorical propositions:

1. All S are P (A proposition)

2. No S are P (E proposition)

3. Some S are P (I proposition)

4. Some S are not P (O proposition)

'S' and 'P' represent the subject and predicate terms, respectively. Note that this is different from the grammatical
subject and predicate. The grammatical subject contains the quantifier (All), while the subject term does not. 'No',
'All', and 'Some' are quantifiers. 'Are' and 'are not' are copulas.

39 | P a g e
*Simply put, a categorical proposition is defined as any proposition that can be interpreted as asserting a relation
of inclusion or exclusion, complete or partial, between two classes.

A class is defined as a collection of all objects which have some specified characteristic in common.
Understanding this definition is no more complicated than observing that the class of “light bulbs” all have the
common characteristic of “being a light bulb.”

The Four Kinds of Categorical Propositions

The critical first step in developing a theory of deduction based on classes, therefore, is to identify the kinds of
categorical propositions and to explore the relations among them.

There are four and only four kinds of standard-form categorical propositions. Here are examples of each of the
four kinds:

1. All politicians are liars


2. No politicians are liars
3. Some politicians are liars
4. Some politicians are not liars
We will examine each of these kinds in turn.

1. Universal affirmative propositions. In these we assert that the whole of one class is included or contained in
another class. “All politicians are liars” is an example; it asserts that every member of another class, the class
of liars. Any universal affirmative propositions can be written schematically as.

All S is P

Where the letters S and P represent the subject and the predicate terms, respectively. Such a proposition
affirms that the relation of class inclusion holds between the two classes and says that the inclusion is
complete, or universal. All members of S are said to be also members of P. Propositions in this standard form
are called universal affirmative propositions. They are also called A propositions.

Categorical propositions are often represented with diagrams, using two interlocking circles to stand for the
two classes involved. These are called Venn Diagrams, named after the English logician and mathematically,
John Venn (1824-1923), who invented them. later we will explore these diagrams more fully, and we will find
that such diagrams are exceedingly helpful in appraising the validity of deductive arguments, For the present
we use these diagrams only to exhibit graphically the sense of each categorical proposition.

We label one circle S, for subject class, and the other circle P, for predicate class. The diagram for the A
proposition, which asserts that all S is P, shows that portion of S which is outside of P shaded out, indicating
that there are no members of S that are not members of P. So, the A proposition is diagrammed thus:

40 | P a g e
2. Universal negative propositions. The second example above, “No politicians are liars”, is a proposition in
which it is denied, universally, that any member of the class of politicians is a member of the class of liars. It
asserts that the subject class, S, is wholly excluded from the predicate class, P. Schematically, categorical
propositions of this kind can be written as

No is P

Where again S and P represent the subject and predicate terms. This kind of propositions denied the relation of
inclusion between the two terms, and denies it universally. It tells us that no members of S are members of
Propositions in this standard form are called universal negative propositions. They are also called E propositions.

The diagram for the E propositions will exhibit this mutual exclusion by having the overlapping portion of the
two circles representing the classes S and P shaded out. So, E propositions is diagrammed thus:

3. Particular Affirmative propositions. The third example above, “Some politicians are liars”, affirms that
some members of the class of al politicians are members of the class of all liars. But it does not affirm this
politician universally. Only some politician or politicians are said to be liars. This proposition does not affirm
or deny anything about the class of all politicians; it makes no pronouncements about the entire class. Nor
does it say that some politicians are not liars, although in some contexts it may be taken to suggest that. The
literal and exact interpretation of this propositions is the assertion that the class of politicians and the class of
liars have some member or members in common. That is what we understand this standard form proposition
to mean.

41 | P a g e
“Some” is an indefinite term. Does it mean “at least one “, or “at least two”, or “at least several”? Or how many?
Context might affect our understanding of the terms as it is used in everyday speech, but logicians, for the sake of
definiteness, interpret “some” to mean “At least one”. A particular affirmative proposition may be written
schematically as

Some S is P.

Which says that at least one member of the class designated by the subject term S is also a member of the class
designated by the predicate term P. The proposition affirms that the relation of class inclusion holds, but does not
affirm it of the first class universally but only partially, that is, it is affirmed of some member, or members, of the
first class. Propositions in this standard form are called particular affirmative propositions. They are also called I
propositions.

The diagram for the I proposition indicates that there is at least one members of S that is also a member of P by
placing an x in the region in which the two circles overlap. So, the I proposition is diagrammed thus:

4. Particular Negative Propositions. The fourth example above, “Some politicians are not liars”, like the third,
does not refer to politicians universally, but only to some member or members of the class; it is. Unlike the
third example, however, it does not affirm the inclusion of some member or members of the first class in the
second class; this is precisely what is denied. It is written schematically as

Some S is not P

Which says that at least one member of the class designated by the subject term S is excluded from the whole of
the class designated by the predicate term P. The denial is not universal. Propositions in this standard form are
called particular negative propositions. They are also called O propositions.

The diagram for the O proposition indicates that there is at least one member of S that is not a member of P by
placing an x in the region of S that is outside of P. So, the O proposition is diagrammed thus:

42 | P a g e
The examples we have used in this section employ classes that are simply named: politicians, liars, vegetarians,
athletes, and so on. But subject and predicate terms in standard-form propositions can be more complicated. Thus,
for example, the proposition “All candidates for the position are persons of honor and integrity”” has the phrase
“candidates for the position” as its subject term. Subject and predicate terms can become more intricate still, but
in each of the four standard forms a relation is expressed between a subject class and a predicate class. These
four---- A, E, I, O propositions--- are the building blocks of deductive arguments.

The analysis of categorical propositions appears to be simple and straightforward, but the discovery of the
fundamental role of theses propositions, and the exhibition of their relations to one another, was a great step in the
systematic development of logic. It was one of Aristotle’s permanent contributions to human knowledge. It
apparent simplicity is deceptive. On this foundation—classes of objects and the relations among those classes---
logicians have erected, over the course of centuries, a highly sophisticated system for the analysis of deductive
argument. This system, whose subtlety and penetration mark it as one of the greatest of intellectual achievements.

STANDARD-FORM CATEGORICAL PROPOSITIONS


PROPOSITION FORM NAME AND TYPE EXAMPLE
All S is P A – Universal Affirmative All lawyers are wealthy people
No S is P E – Universal Negative No lawyers are wealthy people
Some S is P I – Particular Affirmative Some lawyers are wealthy
people
Some S is not P O – Particular Negative Some lawyers are not wealthy
people

Quality, Quantity, and Distribution

A. The QUALITY of a proposition is determined by whether the asserted class relation is one of inclusion or
exclusion. It is either affirmative or negative.

Affirmative – if the proposition affirms some class inclusion, whether complete or partial.

Example: A-proposition (All S is P)

All human beings are mortal

I-proposition (Some S is P)

43 | P a g e
Some logicians are philosophers

Negative – if the proposition denies class inclusion, whether complete or partial.

Example: E-proposition (No S is P)

No Africans are rich

O-proposition (Some S is not P)

Some politicians are not corrupt

B. The QUANTITY of a proposition is determined by whether or not it refers to all members of its subject class.
It is either universal or particular.

Universal – if the proposition refers to all members of the class designated by its subject term.

Example: A-proposition (All S is P)

All idiots are slow learners

E-proposition (No S is P)

No man is immortal

Particular – if the proposition refers only to some members of the class designated by its subject term.

Example: I-proposition (Some S is P)

Some green things are grass

O-proposition (Some S is not P)

Some roses are not red

C. DISTRIBUTION is if the proposition applies to everything described by the term.


Distributed – is a term of a categorical proposition that is used with reference to every member of a class.

Undistributed – if the term is not being used to refer to each and every member of the class.

Example: A-proposition (All S is P) – All senators are citizens

E-proposition (No S is P) – No athletes are vegetarians

I-proposition (Some S is P) – Some soldiers are cowards

O-proposition (Some S is not P) – Some horses are not thoroughbreds

The kinds of distribution of subject and predicate terms in the above listed propositions are explained here:

44 | P a g e
Proposition Subject Predicate Distribution
Refers to all senators. Does not refer to every The subject term is distributed,
member. but the predicate term is not
A
distributed.

Refers to all athletes by Refers to all vegetarians Distribute both their subject and
indicating that they are by indicating that they are predicate terms.
E
not part of the predicate not part of the subject
class. class.
Refers only to some Refers only to some Both subject and predicate terms
I soldiers. cowards, those which are are not distributed.
soldiers.
Refers only to some Refers to all members of The subject term is not
horses, not all of them. the class, not one of them distributed, but the predicate
O
is in the class referred to term is distributed.
by "some horses".

D. General Schema of Standard-Form Categorical Propositions

 Every propositions has matter and form.


- The subject and predicate term are called the matter. The thought-content of the proposition, because they
are the materials out of which the proposition is made.
- The copula is the form. The structure of a proposition, because it is the unifying principle that maintains
the structure of the proposition and imparts to its materials the nature of a proposition. (Piñon, 1979)

 Verbs which serves to connect the subject and predicate terms and is called the copula. But not only the
words “is”, “is not”, “are”, and “are not” are appropriate to use, but depending on the context, other forms of
the verb may also be appropriate.

 The general skeleton of a standard-form categorical proposition always consists of just four parts:
First, the quantifier
Second, the subject term
Third, the copula
Fourth, the predicate term

 The schema may be written as:


Quantifier (subject term) copula (predicate term)

Example:

Some form of the verb “were” and “will not be”

All who attended the party were drunk

Some soldiers will not be heroes.

45 | P a g e
The Traditional Square of Opposition

A. The Square of Opposition


Inference

- Is the act or process of deriving a conclusion based solely on what one already knows. Inference has two
types: Deductive Inference and Inductive Inference. They are deductive, when we move from the general
to the particular and inductive where the conclusion is wider in extent than the premises. In intelligence
testing, mostly deductive inference ability is judged. Inference is studied within several different fields:
 Human inference (i.e. how humans draw conclusions) is traditionally studied within the field of cognitive
psychology.
 Logic studies the laws of valid inference.
 Statisticians have developed formal rules for inference (statistical inference) from quantitative data.
 Artificial intelligence researchers develop automated inference systems.

Deductive Inference

- May be further classified as (i) Immediate Inference (ii) Mediate Inference. Mediate inference is when
more than one premise is relied on (as in a syllogism), because the conclusion is drawn from the first
premise through the mediation of the second. In immediate inference there is only one premise and from
this sole premise conclusion is drawn. Immediate inference has two types: Square of Opposition and
Eduction. Here we will study about Square of Opposition.

The Traditional Square of Opposition is a diagram specifying logical relations among four types of categorical
propositions. (Tahirima, 2014)

There are four ways in which propositions may be “observed” –as Contradictories, Contraries, Subcontraries and
Subalternation. These are represented with an important and widely used diagram called the Square of
Opposition.

46 | P a g e
B. Contradictories
Two propositions are contradictory if one of them is denial of another. They cannot both be true, and cannot both
be false. Two standard-form categorical propositions that have the same subject and predicate terms but differ
from each other in both quantity and quality are contradictories.

A-proposition is contradictory of O-proposition, and E-proposition is contradictory of I-proposition

Example:

A-proposition, “All judges are lawyers”, and the O-proposition “Some judges are not lawyers”, are
clearly contradictories.

E-proposition, “No politicians are idealists”, and the I-proposition, “Some politicians are idealists”, and
they are too contradictories.

 Of the pair, both are opposed in quality (one affirms, the other denies) and quantity (one refers to all and the
other to some). They cannot both be true, and cannot both be false.

C. Contraries
Two propositions are contrary if they cannot both be true, but they might both be false.

A-proposition is contrary of E-proposition

If both the A and the E could be true at the same time, then the subject class would be empty. In the traditional
square, we assume that the subject of the proposition refers to something that exists.

Example:

A-proposition – Texas will win the coming game with Oklahoma

E-proposition – Oklahoma will win the coming game with Texas

 If either of these propositions (referring to the same game) is true, then the other must be false. But these two
propositions are not contradictories, because can game be a draw and then both would be false. Contraries
cannot be both true, but, unlike contradictories, they can both be false.

D. Subcontraries
Two propositions are subcontrary if they cannot both be false, but they might both be true.

I-proposition is subcontrary of O-proposition

Example:

I proposition – Some diamonds are precious stones

O proposition – Some diamonds are not precious stones


47 | P a g e
 Both the propositions could be true that some diamonds are precious stones and some diamonds are not
precious stones. But they could not both be false and therefore must be regarded as subcontraries.

E. Subalternation
A proposition is a subaltern (particular proposition) of another if it must be true, if its superaltern (universal
proposition) is true, and the superaltern must be false if the subaltern is false. When two propositions have the
same subject and the same predicate terms, and agree in quality (both affirming or both denying) but differ in
quantity (one universal, the other particular), they are called corresponding propositions.

I-proposition is subaltern of A-proposition, and O-proposition is subaltern of E-proposition

Example:

A-proposition – All spiders are eight-legged animals Corresponds

I-proposition – Some spiders are eight-legged animals to each other

E-proposition – No whales are fishes Corresponds

O-proposition – Some whales are not fishes to each other

F. Inferences from Square of Opposition:


A number of very useful immediate inferences may be readily drawn from the information embedded in the
traditional square of opposition. Given in the truth, or the falsehood, of any one of anyone of the four standards
form categorical proposition, it will be seen that the truth or falsehood of some or all of the others can be inferred
immediately.

A is given as True: E is false; I is true; O is false.

E is given as True: A is false; I is false; O is true.

I is given as True: E is false; A and O are undetermined.

O is given as True: A is false; E and I are undetermined.

A is given as False: O is true, E and I are undetermined.

E is given as False: I is true; A and O are undetermined.

I is given as False: A is false; E is true; O is true.

O is given as False: A is true; E is false; I is true.

Note: A proposition is undetermined if its truth or falsity is not determined – fixed – by the truth or falsity of any
other proposition. In another sense, a proposition is undetermined if one does not know that it is true and one also
does not know that it is false.

48 | P a g e
Example:

What can you infer about the truth or falsity of the following if we assume that “All successful executives are
intelligent people” is true?

No successful executives are intelligent people. - False

Some successful executives are intelligent people. - True

Some successful executives are not intelligent people. – False

What can you infer about the truth or falsity of the following if we assume that “No animals with horns are
carnivores” is false?

Some animals with horns are carnivores. - True

Some animals with horns are not carnivores. - Undetermined

All animals with horns are carnivores. - Undetermined

Further Immediate Inferences

There are three other important kinds of immediate inferences which were discovered to be not directly
related to the square of opposition. These are: Conversion, Obversion, and Contraposition. (Copi & Cohen, 2009)

Before explaining each of the three other immediate inferences, it is essential to define immediate inferencing.
“Immediate Inference” is a phrase that denotes something that follows from something else without anything
additional. “To mediate” means to be in-between in some fashion. “Im-” is a negative prefix, so “immediate” means
“without mediation,” or “directly.” (Rizvi, 2014)

A. CONVERSION

Conversion is an inference that proceeds by interchanging the subject and predicate terms of the proposition. It
interchanges the subject and predicate to form the converse from the original proposition called convertend. (Copi
& Cohen, 2009)

This important kind of immediate inference is valid because the two propositions, the convertend and converse,
may be validly inferred from one another. They are logically equivalent and doing the process of conversion does
not change the meaning of the convertend or original proposition. (Copi & Cohen, 2009)

However, it shall be noted that conversion, when applied to the four kinds of categorical propositions, different
results yield.

Conversion is perfectly valid for E and I propositions.

Examples:

E propostion:
49 | P a g e
Convertend: No idealists are politicians

Converse: No politicians are idealists

I proposition:

Convertend: Some women are writers

Converse: Some writers are men

The conversion of an O proposition is not, generally valid. An O proposition and its converse are not logically
equivalent.

Example:

O proposition:

Convertend: Some animals are not dogs

Converse: Some dogs are not animals (Not valid)

The A proposition presents a special problem. The converse of an A proposition is not logically equivalent
with its convertend. However, by the process of conversion by limitation or conversion per accidens, wherein the
subject and predicate are interchanged and the quantity of the proposition from universal to particular.

Example:

A proposition:

Convertend: All dogs are animals

*by conversion by limitation, it can be inferred: Some dogs are animals (by subalternation)

Converse: Some animals are dogs

VALID CONVERSIONS
Convertend Converse
A: All S is P A: Some P is S (by limitation)
E: No S is P E: No P is S
I: Some S is P I: Some P is S
O: Some S is not P O: conversion not valid

B. CLASSES AND CLASS COMPLEMENTS

To grasp the other two important kinds of immediate inference, obversion and contraposition, it is essential to
examine more closely the concept of a “class” and the “complement of a class.” Further, the application of these
two concepts shall be differentiated from that of the “contraries” (Copi & Cohen, 2009)

A class is a collection of all things that have a certain common attribute or class-defining characteristic. A class,
therefore can be any collection that is separated from other class because members of such class share a defining

50 | P a g e
characteristic. For purposes of understanding the concept, a class shall be designated as the term S. A class is also
the class complement of its own complement. (Copi & Cohen, 2009)

Every class, is associated with a complement. To define, a complement or complementary class is the collection
of all the things that do not belong to the original class. Every complement has also a complement-defining
characteristic which can easily be said as the negative attribute of the define-characteristic of the original class. The
complement of a class shall be designated as term non-S. (Copi & Cohen, 2009)

The term complement is used in two senses: the complement of a class and the complement of a term. These
are very different but closely related. One term is the term complement of another just in case the first term
designates the class complement of the class designated by the second term. (Copi & Cohen, 2009)

Contraries, not being directly involved with the concepts of a class and complement of a class, shall be discusses
to avoid confusion. Contraries are antonyms if used in English Grammar, wherein one word is the exact opposite
of another, While one may claim that a complement and complement of a class are antonyms in that sense, this is
not true for not everyone or everything, need be one or the other at a certain point. (Copi & Cohen, 2009)

EXAMPLES:

Class: humans

Class-defining characteristic: being human

Complement of the original class: nonhumans

Complement-defining characteristic: not being human

Contraries: Hero and Coward

Class and Complement of a Class: Hero and Nonhero

Explanation: Coward and hero are contraries because no person can be both a coward and a hero. To designate
the compelement of the class “hero,” it shall be non-voter and not coward because not everyone, and certainly not
everything, need be one or another.

C. OBVERSION

Obversion is the inference in which the quality of the proposition is changed and the predicate is
interchanged with its complement; however, the subject and quantity remains.The obverse is logically
equivalent to the original proposition or obvertend. (Copi & Cohen, 2009)

This immediate inference is valid because the original proposition or obvertend is logically equivalent with
that of its obverse and either may be validly inferred from the other. (Copi & Cohen, 2009)

Obversion is valid when applied to any of the four standard-form categorical propositions.

Examples:

E propostion:

51 | P a g e
Obvertend: No umpires are partisans

Obverse: All umpires are nonpartisans

I proposition:

Obvertend: Some metals are conductors

Obverse: Some metals are nonconductors

O proposition:

Obvertend: Some nations were not belligerent

Obverse: Some nations were nonbelligerents

A proposition:

Obvertend: All residents are voters

Obverse: No residents are non-voters

OBVERSIONS
Obvertend Obverse
A: All S is P A: No S is non-P
E: No S is P E: All S is non-P
I: Some S is P O: Some S is not non-P
O: Some S is not P I: Some S is non-P

D. CONTRAPOSITION

Contraposition is the inference in which the subject is interchanged with the complement of the predicate and
the predicate is interchanged with the complement of the subject. In modern logic it is only valid for the A and O
propositions. The valid contrapositive is logically equivalent to the original proposition. In traditional logic, the E
proposition has a contrapositive by limitation which is the subaltern of the invalid E-contrapositive; i.e., the
corresponding O proposition. The contrapositive by limitation is implied by the original but is not (usually)
equivalent to it. (Copi & Cohen, 2009)

The original proposition is the premise while the conclusion is known as the contrapositive.

EXAMPLES:

E propostion: contrapostive does not follow from the original

Premise: No wrestlers are weaklings

Contrapositive: No nonweaklings are nonwrestlers

I proposition: generally not valid

Premise: Some citizens are nonlegislators

Contrapositive: Some legislators are noncitizens

52 | P a g e
O proposition:

Premise: Some students are not idealists

Contrapositive: Some nonidealists are not nonstudents

A proposition:

Premise: All members are voters

Contrapositive: All non-voters are non-members

CONTRAPOSITION
Premise Contrapositive
A: All S is P A: All non-P is non-S
E: No S is P O: Some non-P is not non-S (by limitation_
I: Some S is P Contraposition not valid
O: Some S is not-P Some non-P is non-S

Existential Import and the Interpretation of Categorical Propositions

A proposition is said to have existential import if the truth of the proposition requires a belief in the
existence of members of the subject class. I and O propositions have existential import; they assert that the classes
designated by their subject terms are not empty. But in Aristotelian logic, I and O propositions follow validly from
A and E propositions by sub-alternation. (Copi & Cohen, 2009) As a result, Aristotelian logic requires A and E
propositions to have existential import, because a proposition with existential import cannot be derived from a
proposition without existential import. Thus, for Aristotle, if we believe it is that "All unicorns have one horn" then
we are committed to believing that unicorns exist. For the modern logician or mathematician, this is an unacceptable
result because modern mathematics and logic often deal with empty or null sets or with imaginary objects. A
modern mathematician might, for example, wish to make a true claim about all irrational prime numbers. Since
there are no irrational prime numbers, Aristotle would say that any claim about them is necessarily false. (cstl-
cla.semo, n.d.)

George Boole developed an interpretation of categorical propositions solves the dilemma by denying that
universal propositions have existential import. Modern logic accepts the Boolean interpretation of categorical
propositions. This interpretation has the following consequences: (Copi & Cohen, 2009)

 Because A and E propositions have no existential import, sub-alternation is not valid.

 Because A and E propositions have no existential import, super-alternation is not valid.

 Contraries are eliminated because A and E propositions can now both be true when the subject class is
empty.

 Sub-contraries, on the other hand, are retained because I and O propositions always have existential import.

 Some immediate inferences are preserved: conversion for E and I propositions, contraposition
for A and O propositions, and obversion for any proposition.
53 | P a g e
 Any argument that relies on the mistaken assumption of existential import commits the existential fallacy.

Why Reject Existential Import?

Modern logic rejects existential import for a number of reasons. The most significant for such purposes
have to do with the nature of universal claims and the understanding of what it means to say of a proposition
that it is false. (Copi & Cohen, 2009) Starting with the latter, ask yourself what would have to be the case about
the world for you to claim that an A type proposition is false. Consider the claim "All swans are white". In
order for that claim to be false, one needs to know that there is at least one non-white swan. Imagine how one
might argue with someone who insists that it is true that "A'' swans are white". One would produce as evidence
for the falsity of the claim the existence of a non-white swan. "No," you might argue, "not all swans are white,
for here is a swan that is brown." But now suppose for a minute that there were no swans at all. What sort of
evidence could one produce, in the total absence of any swans, against the claim that all swans are
white? Obviously one could not produce a non-white one because there are not any swans at all. (cstl-cla.semo,
n.d.)

In the absence of any evidence for a falsifying instance to the universal claim, one should accept the
claim. But now extend that reasoning to universal claims about empty classes and non-existent
objects. Universal claims about empty sets are all true, because there are no falsifying instances. (cstl-cla.semo,
n.d.)

NOTE: Claims about empty sets are trivially true. Sure, "all irrational prime numbers are odd" because there
are no irrational prime numbers, but it is equally true that "all irrational prime numbers are even".

Universal Conditionals

Another reason for adopting the modern approach toward existential import, related to the previous one,
has to do with the interpretation of a universal claim. Instead of seeing a universal claim as one about the
members of 2 sets, modern logic sees A and E type propositions as universally quantified conditionals. "All
S's are P's" is understood by the modern logician as saying "For any object x, if x has property S, then x has
property P". Suppose there are no S's in the universe. Then, no matter what value we assign to x, "x has
property S" will be false. But a conditional with a false antecedent is true, so a universal conditional whose
subject class is empty is true. (cstl-cla.semo, n.d.)

Symbolism and Diagrams for Categorical Propositions

Categorical proposition is the base for the Classical Logic. They are called categorical propositions because
they are about categories or classes. Such propositions affirm or deny that some class S is included in some other
class p, completely or partially. (Copi & Cohen, 2009)

Boolean interpretation of categorical propositions depends heavily on the notation of an empty class; it is
convenient to have a symbol to represent it. The symbol zero 0, is used for this purpose. The term S has no members,
we write an equal sign between S and 0. (Copi & Cohen, 2009)

54 | P a g e
Thus the equation S = 0 says that there are no S’s, or that S has no members. To deny that S is empty is to
say that S does have members. We symbolize that denial by drawing a slanting line through the equals sign. The
inequality S ≠ 0 says that there are S’s, by denying that S is empty. (Copi & Cohen, 2009)

1. To symbolize A proposition: The A proposition, “All S is P”, says that all members of class S are also
member of the class P. That is, there are no members of the class S that that are not members of P or “No
S is non- P”

2. To symbolize E propositions: The E propositions, “No S is P”, says that no members of the class S are
the members of class P. This can be rephrased by saying that the product of the two classes is empty which
is symbolized by the equation SP = 0.

3. To symbolize I proposition: The I proposition “Some S is P”, says that at least one member of S is also a
member of P. This means that the product of the classes S and P is not empty. It is symbolized by the
inequality SP ≠ 0.

4. To symbolize O proposition: The O proposition, “Some S is not P”, obverts to the logically equivalent to
I propositions, “Some S is non-P”. It is symbolized by the inequality SP ≠ 0.

The relationships between these propositions contradict each other in several ways, as can be illustrated here.

This diagram shows:

1. CONTRADICTORIES

Contradictory means they have opposite truth values. A and O propositions are contradictory as are E and
I proposition. They are opposite of each other in both Quantity and Quality therefore, have opposite truth
values. When any Categorical statement is true, its partner across the diagonal is false. When false its
contradictory must be true. (Copi & Cohen, 2009)

55 | P a g e
Example: if “all rubies are red stones” (A) is true, then “some rubies are not red stones” (O) must be false.
Similarly if “no mammals are aquatic” (E) is false, then “some mammals are aquatic” (I) must be true.

2. CONTRARIES
A and E propositions are contrary. Propositions are contrary when they cannot both be true. (Copi & Cohen,
2009)

Example: An A proposition e.g. “all giraffes have long necks” cannot be true at the same time as the
corresponding E proposition: “no giraffes have long necks”. They are opposite in Quality only (both are
universals). however that corresponding A and E proposition while contrary are not contradictory.

3. SUBCONTRARIES
I and O propositions are Sub Contrary. Propositions are Sub Contrary when it is impossible for both to
false. Example: “some lunches are free” is false, “some lunches are not free” must be true. However that is
possible for corresponding I and O both to be true. (Copi & Cohen, 2009)

Example: “some nations are democratic” and “some nations are not democratic”. Again I and O
propositions are sub contrary, but not contrary or contradictory.

4. SUBALTERNATION
Subalternations are same in Quality but different in Quantity. From A to I (if A is true then, the I is true).
From E to O (if E is true then, O is true).

Now for Falsity: From I to A (if the I is false, then A is false). From O to E (if O is false then, E is false).

The Venn diagrams constitute an iconic representation of the standard form categorical propositions, in
which spatial inclusions and exclusions correspond to the non-spatial inclusions and exclusions of classes. They
provide an exceptionally clear method of notation. (Rizvi, 2014)

56 | P a g e
References:

Copi & Cohen , www.lanecc.edu, faculty.bsc.edu

I. M. Copi and Carl Cohen, Introduction to Logic, New York: Macmillan, 1998 (10th edition)

Retrieved from: http://philosophy.lander.edu/logic/prop.html

I. M. Copi and Keith Burgess-Jackson, Informal Logic, Upper Saddle River, New Jersey: Prentice Hall, 1996
Retrieved from: http://philosophy.lander.edu/logic/prop.html

James A. Gould, editor, Classic Philosophic Questions, Englewood Cliffs, NJ: Prentice-Hall, 1998
Retrieved from: http://philosophy.lander.edu/logic/prop.html

Kathleen Dean Moore, Inductive Arguments: Developing Critical Thinking Skills, Dubuque, Iowa: Kendall/Hunt,
1995)
Retrieved from: http://philosophy.lander.edu/logic/prop.html

John Hospers, Human Conduct: Problems of Ethics, New York: Harcourt Brace, 1995
Retrieved from: http://philosophy.lander.edu/logic/prop.html

Hermann Hesse. Siddhartha. San Francisco: New Directions, 1951


Retrieved from: http://philosophy.lander.edu/logic/prop.html

John M. Koller. Oriental Philosophies. New York: Scribners’, 1992


Retrieved from: http://philosophy.lander.edu/logic/prop.html

Walter Kaufmann, Existentialism From Dostoevsky to Sartre, New American Library, Meridian, 1975
Retrieved from: http://philosophy.lander.edu/logic/prop.html

William Barrett, Irrational Man: A Study in Existential Philosophy, Doubleday Anchor, 1962
Retrieved from: http://philosophy.lander.edu/logic/prop.html

Sungla K., 2015. Categorical Propositions- Logic


Retrieved from: http://www.easybib.com/reference/guide/apa/website

Plato.stanford.edu, Aug 8, 1997. The Traditional Square of Opposition


Retrieved from: https://plato.stanford.edu/entries/square/

Tahirimanov19, February, 2015. Traditional Square of Opposition


Retrieved from: https://blogfallacies.wordpress.com/2015/02/26/traditional-square-of-opposition/
academic.csuohio.edu. (n.d., n.d. n.d.). Conversion, Obversion, and Contraposition. Retrieved February 18, 2017,
from http://academic.csuohio.edu: http://academic.csuohio.edu/polen/LC9_Help/4/immediate.htm

Copi, I. M., & Cohen, C. (2009). Introduction to Logic 13th Edition. Jurong, Singapore: Pearson Education Inc.

cstl-cla.semo. (n.d., n.d. n.d.). EXISTENTIAL IMPORT. Retrieved February 18, 2018, from cstl-cla: http://cstl-
cla.semo.edu/hhill/pl120/notes/existential%20import.htm

57 | P a g e
Rizvi, S. (2014, June 6). in. Retrieved February 18, 2018, from slideshare.net:
https://www.slideshare.net/hadiqazehra92/symbolism-and-diagram-for-categorical-proposition

58 | P a g e
CHAPTER 6: CATEGORICAL SYLLOGISM

Standard-Form Categorical Syllogisms

Syllogism- refers to any deductive argument in which a conclusion is inferred from two premises .It is an argument
that relies on the forms of proposition that have commonly two categorical propositions as premises and one
categorical proposition as conclusion (Copi and Cohen 2014).

Categorical Syllogisms is a deductive argument consisting of three categorical propositions that contain exactly
three terms, each of which occurs in exactly two of the propositions. Syllogisms dealt in here are called categorical
because the arguments are based on the relations of classes, or categories- relations that are expressed by the
categorical propositions.

An argument needs to be in its Standard form for us to be able to understand the argument correctly. According to
Copi and Cohen 2014, the argument is in its standard form:

1. When its premises and conclusions are all in standard-form categorical propositions.
Standard-form categorical propositions are as follows:

FORM PROPOSITION
A Universal Affirmative ALL is P
E Universal Negative NO S is P
I Particular Affirmative SOME S is P
O Particular Negative SOME S is not P

2. The categorical propositions are arranged in standard order. The standard order follows the Major Premise,
Minor Premise, and the Conclusion.
The 1) logical names of the premises of the syllogism and the 2) names of the terms of syllogism is needed to
explain the order of the premises to be able to put a syllogism into its standard form.

A. The terms of the Syllogism


1. Name of Terms
A syllogism consists of a Major Term, Minor Term, and Middle Term.
a) Major Term- occurs as the predicate term of the conclusion in a standard-form categorical
syllogism.
b) Minor Term- occurs as the subject of the conclusion in a standard-form categorical syllogisms
c) Middle Term- appears in both premises but does not appear in the conclusion
To identify the terms by name, we refer to the conclusion of the syllogism.

Example :

No heroes are cowards.

Some soldiers are cowards.

Therefore, some soldiers are not heroes.

In this example, the conclusion “Some soldiers are not heroes”, the predicate is “heroes”, hence “heroes” here is
the major term. The subject in the example are the “soldiers”, hence “soldiers” here is the minor term.
59 | P a g e
As for the “cowards”, it appears to be the middle term as it existed in both premises but is nowhere to be found in
the conclusion.

The major and minor term must occur in a different premise. It cannot be that both major and minor term exists in
one premise. These premises of syllogism also have names.

2. Names of the premises


General Rule: A premise is named after the term that appears both in it and in the conclusion.

a) Major Premise- contains the major term, and usually states a general rule. In legal
arguments, it is the statement of law.
Example:

No heroes are cowards.

Some soldiers are cowards.

Therefore, some soldiers are not heroes.

In this example, “heroes” is the major term, and “heroes” appear in the premise “No heroes are cowards”. It is said
that premises containing the major term is the major premise. Hence, the premise “No heroes are cowards” is the
major premise.

The major premise is not the major premise because it appears first but because it is the premise that contains the
major term.

b) Minor Premise- contains the minor term. It makes a factual assertion about a particular
person or thing or a group of persons or things. In legal arguments, it is the statement of
facts.
Example:
No heroes are cowards.
Some soldiers are cowards.
Therefore, some soldiers are not heroes.
The premise “Some soldiers are cowards” contains the minor term “soldiers”. The premise containing the minor
term is called minor premise. Thus, the premise “Some soldiers are cowards” is called the minor premise.

It is the minor premise because it contains the minor term and not because of its position.

A syllogism is in a standard-form when its premises are arranged in a specified order. In a standard-form syllogism,
the major premise is always stated first, the minor premise is stated second, and the conclusion is stated last.

B. Mood of the Syllogism


Mood is a characterization of a categorical syllogisms. It is determined by the types of standard-form categorical
proposition it contains. It is represented by three letters, and those three letters are given in three standard-form
order.
First letter - names the type of the syllogism’s major premise
Second letter- names the type of the syllogism’s minor premise
Third letter- names the type of the syllogism’s conclusion.
Example:
No heroes are cowards. - The major premise is an E proposition.

60 | P a g e
Some soldiers are cowards.- The minor premise is an I proposition.
Therefore, some soldiers are not heroes.- The conclusion is an O propostion.
* The mood of the proposition is EIO.
There are 64 moods by the three letters of its constituent propositions, AAA, AAI, AAE, and so on to OOO.

C. The Figure of Syllogism


To describe the form of a syllogism we must state its mood and its figure.
Figure refers to the position of the middle term in the premises of a standard-form categorical syllogism.
Syllogisms can only have four,and only four possible different figures:
1. Middle term may be the subject term of the Major Premise and the predicate term of the Minor Premise;
or
2. Middle term may be the predicate of both premises; or
3. Middle term may be the subject term of both premises; or
4. Middle term may be the predicate term of the Major Premise and the subject term of the minor premise.
Every syllogism must have one or another of these four figures.
The characters of these figures can be readily visualized readily when schematized as in the following array :
Figure
First M-P
S-M
. · . S-P
Second P-M
S-M
. · . S-P
Third M-P
M-S
. · . S-P
Fourth P-M
M-S
. · . S-P

Any standard form syllogism is completely described when we specify its mood and figure.
Example 1:
No heroes are cowards. - E
Some soldiers are cowards.- I
Therefore, some soldiers are not heroes.- O
The mood is EIO, and it is in the second figure. The name of this form EIO-2, is Festino.

Example 2:
No M is P.- E
All S is M.-A
. · . No S is P.-E
The mood is EAE, and it is on the first figure. We can characterize this as EAE-1 a
form whose unique name is Celarent.

Celarent and Festino are known to be valid forms, we may conclude that whenever we encounter one argument in
one of these forms, it is valid too.

61 | P a g e
The Formal Nature of Syllogistic Argument

The constituent propositions of a syllogism are all contingent- that is, that one of those propositions is necessarily
ture, or necessarily false. Thus, the validity or invalidity of any syllogism depends entirely on its form.
Validity and invalidity are completely independent of the specigic content of the argument or its subject matter.
Thus any syllogism of the form AAA-1 is valid, regalrdless of the subject matter.
Example:
All M is P.
All S is M.
. · . All S is P.

This example is valid regardless of its subject matter. The name of this syllogism’s form is Barbara.
A valid syllogism is valid in virtue of its form alone, and so we call it formally valid. We assume throughout that
its constituent propositions are neither logically true nor logically false. That is, if it contained either a logically
false premise or a logically true conclusion, then the argument would be valid regardless of its syllogistic form-
valid in that it would be logically impossible for its premises to be true and its conclusion false.

If any syllogism is valid in virtue of its form alone, any other syllogism having that same form will also be valid;
and if a syllogism is invalid, any other syllogism having thtform will also be invalid.
Example:
All liberants are proponents of national health insurance.
Some members of the administration are proponents of national health insurance.
Therefore, some members of the administration are liberants.
Regardless of the truth or falsehood of its constituent propositions, the argument is invalid. To expose its fallacious
character,we are to construct another argument that has exactly the same form but whose validity is immediately
apparent. You might as well argue that:
All rabbits are very fast runners.
Some horses are very fast runners.
Therefore, some horses are rabbits.
This one is invalid, so the argument above is invalid.
This method of arguing is referred as logical argument. Logical agrument is one of the excellent method in arguing
and is one of the most powerful tool in debate.

Venn Diagram Technique for Testing Syllogisms

To test a categorical syllogism using a Venn Diagram, both of its premises must be represented first in one
diagram. It requires drawing three overlapping circles (Figure 1), the two premises of a standard- form syllogism
contain three different term namely: minor term, major term, and middle term, which is abbreviated as S, P, and M
respectively and label it in a clock-wise manner. One circle labeled S diagrammed the class S and 𝑆̅, while the two
overlapping circles labeled S and P diagrammed four classes (SP, S𝑃̅, and 𝑆̅𝑃̅) and the three overlapping circles,
labeled S, P and M diagrammed eight classes (S𝑃̅𝑀 ̅ , SP𝑀̅ , 𝑆̅P𝑀
̅ , S𝑃̅M, SPM, 𝑆̅PM, 𝑆̅𝑃̅M, 𝑆̅𝑃̅𝑀
̅ ), these represented
the eight parts into which the three circles divided the plane.

62 | P a g e
S S𝑃̅𝑀
̅ 𝑆 ̅ P𝑀
̅ P
̅
SP𝑀

SPM

S𝑃̅M 𝑆̅PM
Figure 1
𝑆̅𝑃̅𝑀
̅
𝑆̅𝑃̅M
M

To explain it further Copi and Cohen said that this figure can be interpreted, in terms of the various different
classes determined by the class of all Swedes (S), the class of all Peasants (P), and the class of all Musicians (M).

SPM It is the product of these three classes (Swedish Peasant Musicians)


̅
SP𝑀 It is the product of S and P and the compliment of M (Swedish Peasant who are not
Musicians)
S𝑃̅M It is the product of S and M and the compliment of P (Swedish Musicians who are
not peasants)
S𝑃̅𝑀
̅ It is the product of S and the compliment of P and M (Swedes who are neither
Peasants nor Musicians)
𝑆̅PM It is the product of P and M and the compliment of S (Peasant Musicians who are not
Swedes)
𝑆̅P𝑀
̅ It is the product of P and the compliment of S and M (Peasants who are neither
Swedes nor Musicians)
𝑆̅𝑃̅M It is the product of M and the compliment of S and P (Musicians who are neither
Swedes nor Peasants)
𝑆̅𝑃̅𝑀
̅ It is the product of S, P, and M (The class of all things that are neither Swedes nor
Peasants nor Musicians)

If we focus our attention on just the two circles labeled P and M, by simply shading out we can diagram
any standard- form categorical propositions whose two terms are P and M.

Example: All M is P

63 | P a g e
 We just simply shade out all of M that is not contained in P.

S P

If we focus our attention on just the two circles labelled S and M, by simply shading out we can diagram
any standard- form categorical propositions whose two terms are S and M.

Example: All S is M

 We just simply shade out all of S which is not contained in M.

S P

The advantage of using three overlapping circles allows us to diagram two propositions together.

Example: All M is P

All S is M

 All S is P

64 | P a g e
 Copi and Cohen said that this syllogism is valid if the two premises imply or entail the conclusion.
Diagramming the premises of a valid argument should also suffice to diagram its conclusion. As we can see,
the diagram that represents the two premises, also diagrams the conclusion and from this we can conclude that
this example is a valid syllogism.

Applying a Venn diagram test to an invalid syllogism:

Example:

All dogs are mammals CATS DOGS

All cats are mammals

All cats are dogs

MAMMALS

In this diagram, the class of all cats is equal to S, the class of all dogs is equal to P, and the class of all
mammals is equal to M. The conclusion in this example has not been diagrammed, because the part of S𝑃̅M was
not shaded, to diagram the conclusion both S𝑃̅𝑀 ̅ 𝑎𝑛𝑑 S𝑃̅M must be shaded. An argument whose premises do not
imply its conclusion is invalid. Hence, as this diagram does not imply its conclusion the given syllogism is invalid.

When we use a Venn diagram to test a syllogism with one universal premise and one particular premise,
we must diagram the universal premise first.

Example:

All artists are egotist (universal premise)

Some artists are paupers (particular premise)

 Some paupers are egotists

65 | P a g e
We should diagram the universal premise first before inserting x to diagram the particular premise.

paupers egotists

artists

Copi and Cohen said that if we diagram the particular premise first we would not have known where to
insert the x, whether in SPM part or S𝑃̅M or in both. We can now examine the figure to see if the conclusion has
been diagrammed. If the conclusion, “Some paupers are egotists”, has been diagrammed, there will be an x
somewhere in the overlapping part of the circles paupers and egotist. The x must be places in the region SPM for it
to be considered valid. As we can see the conclusion of the syllogism has already been diagrammed, therefore the
syllogism is valid.

Using a Venn diagram in testing the argument:


professional great
Example: athletes scientist
All great scientist are college graduates x

Some professional athletes are college graduates college


graduates
Therefore, some professional athletes are great scientists

In this syllogism it is a bit confusing as to where are we going to place the symbol x in diagramming the
particular premise. By placing x in each of the region will go beyond what the premise assert. But as Copi and
Cohen discussed, placing x on the line that divides the overlapping region of SM in to two-part SPM and S𝑃̅M we
can diagram what the second premise exactly assert without adding anything to it. Placing an x on the line between
the two regions states that there is something that belongs in one of them but does not indicate which one.

Inspecting this diagram of the premises to see whether the conclusion of the syllogism has been diagrammed
in it, we find that it is not. To diagram the conclusion, “Some professional athletes are great scientist”, an x must
appear in the overlapping part of the two upper- circle either in SP𝑀̅ or in SPM. The first of these is shaded out and
certainly contains no x. The diagram does not show an x in SPM either. True, there must be a member of either
SPM or S𝑃̅M, but Copi and Cohen said that the diagram does not tell us that it is in the former rather than the latter.

66 | P a g e
We do not know that the conclusion is false, only that it is not asserted or implied by the premises. However, to let
us know that the argument is invalid. The diagram suffices to show not only that the given syllogism is invalid, but
that all syllogisms of the example are invalid.

General technique of using Venn Diagrams to test the validity

Label the three- circle Venn diagram with the


syllogisms three terms

Diagram both premises (universal premise and the particular premise)

Inspect the diagram to see whether the diagram of the premises contains a
diagram of the conclusion

(If it does, the syllogism is valid)

(If it does not, then the syllogism is invalid)

THEORETRICAL RATIONALE:

In our previous discussion, we said that one legitimate test of the validity or invalidity of a syllogism is to
establish the validity or invalidity of a different syllogism that has exactly the same form. Copi sates that, a syllogism
will be about classes of objects that are not all present. The relations of inclusion or exclusion among such classes
may be reasoned about and may be discovered in the course of scientific investigation, but they certainly are not
open to direct inspection, because not all members of the classes involved are present at one time to be inspected.
But according to Copi and Cohen, we can examine situations on our own, in which the only classes concerned
contain by their very definitions only things that are present and open to direct inspection, and to further discuss
here are some examples:

All successful people are people who are keenly interested in their work.
No people who are keenly interested in their work are people whose attention is easily distracted when they
are working.
 No people whose attention is easily distracted when they are working are successful people.

67 | P a g e
S P Successful people

People whose attention


is easily distrusted
when they are working

M People who are


keenly interested in
work

As this syllogism concerns large classes of remote objects, how could we know that it is a valid or invalid
syllogism? According to Copi and Cohen we can construct a syllogism of the same form that involves objects that
are immediately present and directly available for our inspection, and these objects are the points within the
unshaded portions of the circles labelled S, P, and M in our Venn diagram.

Our created syllogism:


All points within the unshaded part of the circle labelled P are points within the unshaded part of the circle labelled
M.
No points within the unshaded part of the circle labelled M are points within the unshaded part of the circle labelled
S.
 No points within the unshaded part of the circle labelled S are points within the unshaded part of the
circle labelled P.

 In this case we can now see all the possibilities, and know that because all the points of P are also points of M,
and because M and S have no points in common, S and P cannot possibly have any points in common. The
new syllogism refers only to classes of points in the diagram; it can be seen to be valid by looking at the things
it talks about. The original syllogism has exactly the same form as this second one; we are now assured by the
formal nature of syllogistic argument that the original syllogism is also valid.

Syllogistic Rules and Syllogistic Fallacies

A violation of one of these rules renders the syllogism invalid. If it is a mistake of that special kind it is called
fallacy, but if it is a mistake in the form or argument, it is a formal fallacy.
One must avoid fallacies. Each of these formal fallacies has a traditional name explained below.

Rule 1. Avoid four terms.


A valid standard-form categorical syllogism must contain exactly three terms, each of which used in the same sense
throughout the argument.
The use of exactly three categorical terms is part of the definition of a categorical syllogism, and we saw earlier that
the use of an ambiguous term in more than one of its senses amounts to the use of two distinct terms. In categorical
syllogisms, using more than three terms commits the fallacy of four terms.

68 | P a g e
Fallacy of four terms is the formal fallacy that is committed when a syllogism is constructed with more than three
terms.
Example:

Power tends to corrupt


Knowledge is power
Knowledge tends to corrupt

This syllogism appears to have only three terms, but there are really four since one of them, the middle term “power”
is used in different senses in the two premises. To reveal the argument’s invalidity we need only note that the word
“power” in the first premise means “ the possession of control or command over people,” whereas the word “power”
in the second premise means “the ability to control things.
Rule 2. Distribute the middle term in at least one premise.
A term is “distributed” in a proposition when the proposition refers to all members of the class designated by that
term. If the middle term is not distributed in at least one premise, the connection required by the conclusion cannot
be made.
According to to Historian Barbara Tuchman, many early critics of anarchism relied on the following “unconscious
syllogism” :
All Russians are revolutionists.
All anarchists were revolutionists.
Therefore, all anarchists were Russians.

This syllogism is planly valid. It asserts a connection between anarchists and Russians by relying on the
links between each of those classes and the class of revolutionists- but revolutionists is an undistributed term in
both of the premises.
The first premise does not refer to all revolutionists, and neither does the second. The first premise does not
refer to all revolutionists, and neither does the second. If the middle term in the argument is not distributed in at
least one premise of a syllogism, that syllogism cannot be valid. This fallacy commited is called fallacy of the
undistributed middle.
Fallacy of the undistributed middle is the formal fallacy that is committed when the middle term of a syllogism is
not distributed in at least one premise.Under this rule is the need to link the minor and the major terms.
Example:

All sharks are fish


All salmon are fish
All salmon are sharks

The middle term is what connects the major and the minor term. If the middle term is never distributed, then the
major and minor terms might be related to different parts of the M class, thus giving no common ground to relate S
and P.
Rule 3. Any term distributed in the conclusion must be distributed in the premise.
A premise that refers only to some members of the class designated by the major or minor term of a syllogism
cannot be used to support a conclusion that claims to tell us about every menber of that class. Depending which of

69 | P a g e
the terms is misused in this way, syllogisms in violation commit either of the two different forms of illicit process
which are the fallacy of the illicit major and the fallacy of the illicit minor.
Fallacy of illicit process is the formal fallacy that is committed when a term that is distributed in the conclusionis
not distributed in the corresponding premise.
Ilustrating an illicit process of the major term:
All dogs are mammals.
No cats are dogs.
Therefore, no cats are mammals.
The mistake is in the conclusion’s assertion about all mammals, saying that all of them fall outside the class
of the cats. Take note that A proposition distributes its subject term but does not distribute its predicate term. The
conclusion illicitly goes beyond what the premises assert.Because “mammals” is the major term in this syllogism,
the fallacy here is an illicit major.
Example of an Illicit process of the minor term:
All traditionally religious people are fundamentalists.
All traditionally religious people are opponents of abortion.
Therefore, all opponents of abortion are fundamentalists.
The mistake is in the conclusion making an assertion “all opponents of abortion”, but the premise make no
such assertion; they say nothing about all abortion opponents. So the conclusion here goes illicitly beyond what the
premises warrant. In this case “opponents of abortion” is the minor term, so the fallacy is that of an illicit minor.

Rule 4. Avoid two negative premises.


Any negative proposition (E or O) denies class inclusion; it asserts that some or all of one class is excluded cannot
yield the linkage that the conclusion assers, and therefoe cannot yield a valid argument. The mistake is named the
fallacy of exclusive premises.
Fallacy of exclusive premises is the formal fallacy that is committed when both the premises in a syllogism are
negative propositions (E or O).
Example:

No fish are mammals


Some dogs are not fish
Some dogs are not mammals

If the premises are both negative, then the relationship between S and P is denied. The conclusion cannot, therefore,
say anything in a positive fashion. That information goes beyond what is contained in the premises.
Rule 5. If either premise is negative, the conclusion must be negative.
An affirmative proposition asserts that one class is included in some way in another class, but a negative proposition
that asserts exclusion cannot imply anything about inclusion. For this reason an argument with a negative
proposition cannot have an affirmative conclusion. An argument that violates this rule is said to commit the fallacy
of drawing an affirmative conclusion from a negative premise.
If an affirmative conclusion requires two affirmative premises, as has just been shown, we can know with certainty
that if either of the premises is negative, the conclusion must also be negative, or the argument is not valid.
Example:
No poets are accountants.
70 | P a g e
Some artists are poets.
Therefore, some artists are accountants.
It will be seen that the exclusion of poets and accountants, asserted by the first premise of this syllogism
cannot justify any valid inference regarding the inclusion of artists and accountants.

Rule 6. From two universal premises no particular conclusion may be drawn.


Because we do not assume the existential import of universal propositions, they cannot be used as premises to
establish the existential import that is part of any particular proposition. The existential fallacy violates this rule.
Although it is possible to identify additional features shared by all valid categorical syllogisms (none of them, for
example, have two particular premises), these six rules are jointly sufficient to distinguish between valid and invalid
syllogisms. (Boolean interpretation of categorical propositions)
Existential fallacy is the formal fallacy that is committed when, in a standard-form categorical syllogism, a particular
conclusion is inferred from two universal premises.

Example:

All disgruntled creatures are nihilists

All pterodactyls are disgruntled creatures

Some pterodactyls are nihilists

On the modern interpretation, universal statements make no claims about existence while particular ones do. Thus,
if the syllogism has universal premises, they necessarily say nothing about existence. Yet if the conclusion is
particular, then it does say something about existence. In which case, the conclusion contains more information
than the premises do, thereby making it invalid.
VI. Rules and Fallacies for Categorical Syllogisms
A syllogism may fail to establish its conclusion in many different ways. So to help the reasoner in
avoiding common errors Copi and Cohen gave six rules to guide the reasoner

Rule-1. A valid categorical syllogism will have three and only three unambiguous categorical terms. (Avoid
four terms rule)
The use of exactly three categorical terms is part of the definition of a categorical syllogism, and we saw earlier
that the use of an ambiguous term in more than one of its senses amounts to the use of two distinct terms. In
categorical syllogisms, using more than three terms commits the fallacy of four terms.

Fallacy: Four terms

Example:

Power tends to corrupt


Knowledge is power
71 | P a g e
Knowledge tends to corrupt

Justification: This syllogism appears to have only three terms, but there are really four since one of them, the
middle term “power” is used in different senses in the two premises. To reveal the argument’s invalidity we need
only note that the word “power” in the first premise means “ the possession of control or command over people,”
whereas the word “power” in the second premise means “the ability to control things.

Rule-2. In a valid categorical syllogism the middle term must be distributed in at least one of the premises.
In order to effectively establish the presence of a genuine connection between the major and minor terms, the
premises of a syllogism must provide some information about the entire class designated by the middle term. If
the middle term were undistributed in both premises, then the two portions of the designated class of which they
speak might be completely unrelated to each other. Syllogisms that violate this rule are said to commit the fallacy
of the undistributed middle.
Fallacy: Undistributed middle
Example:

All sharks are fish


All salmon are fish
All salmon are sharks

Justification: The middle term is what connects the major and the minor term. If the middle term is never
distributed, then the major and minor terms might be related to different parts of the M class, thus giving no
common ground to relate S and P.

Rule-3. In a valid categorical syllogism if a term is distributed in the conclusion, it must be distributed in
the premises.
A premise that refers only to some members of the class designated by the major or minor term of a syllogism
cannot be used to support a conclusion that claims to tell us about every menber of that class. Depending which of
the terms is misused in this way, syllogisms in violation commit either the fallacy of the illicit major or the fallacy
of the illicit minor.
Fallacy: Illicit major; illicit minor; or illicit process

Examples:

All horses are animals


Some dogs are not horses
Some dogs are not animals

And:

72 | P a g e
All tigers are mammals
All mammals are animals
All animals are tigers

Justification: When a term is distributed in the conclusion, let’s say that P is distributed, then that term is saying
something about every member of the P class. If that same term is NOT distributed in the major premise, then the
major premise is saying something about only some members of the P class. Remember that the minor premise
says nothing about the P class. Therefore, the conclusion contains information that is not contained in the
premises, making the argument invalid.

Rule-4. A valid categorical syllogism may not have two negative premises.
The purpose of the middle term in an argument is to tie the major and minor terms together in such a way that an
inference can be drawn, but negative propositions state that the terms of the propositions are exclusive of one
another. In an argument consisting of two negative propositions the middle term is excluded from both the major
term and the minor term, and thus there is no connection between the two and no inference can be drawn. A
violation of this rule is called the fallacy of exclusive premises.
Fallacy: Exclusive premises
Example:

No fish are mammals


Some dogs are not fish
Some dogs are not mammals

Justification: If the premises are both negative, then the relationship between S and P is denied. The conclusion
cannot, therefore, say anything in a positive fashion. That information goes beyond what is contained in the
premises.

Rule-5. If either premise of a valid categorical syllogism is negative, the conclusion must be negative.

An affirmative proposition asserts that one class is included in some way in another class, but a negative
proposition that asserts exclusion cannot imply anything about inclusion. For this reason an argument with a
negative proposition cannot have an affirmative conclusion. An argument that violates this rule is said to commit
the fallacy of drawing an affirmative conclusion from a negative premise.
Fallacy: Drawing an affirmative conclusion from a negative premise, or drawing a negative conclusion from an
affirmative premise.
Example:

All crows are birds


Some wolves are not crows

73 | P a g e
Some wolves are birds

Justification: Two directions, here. Take a positive conclusion from one negative premise. The conclusion states
that the S class is either wholly or partially contained in the P class. The only way that this can happen is if the S
class is either partially or fully contained in the M class (remember, the middle term relates the two) and the M
class fully contained in the P class. Negative statements cannot establish this relationship, so a valid conclusion
cannot follow. Take a negative conclusion. It asserts that the S class is separated in whole or in part from the P
class. If both premises are affirmative, no separation can be established, only connections. Thus, a negative
conclusion cannot follow from positive premises. Note: These first four rules working together indicate that any
syllogism with two particular premises is invalid.

Rule-6. In valid categorical syllogisms particular propositions cannot be drawn properly from universal
premises.
Because we do not assume the existential import of universal propositions, they cannot be used as premises to
establish the existential import that is part of any particular proposition. The existential fallacy violates this rule.
Although it is possible to identify additional features shared by all valid categorical syllogisms (none of them, for
example, have two particular premises), these six rules are jointly sufficient to distinguish between valid and
invalid syllogisms.
Fallacy: Existential fallacy

Example:

All mammals are animals


All tigers are mammals
Some tigers are animals

Justification: On the Boolean model, Universal statements make no claims about existence while particular ones
do. Thus, if the syllogism has universal premises, they necessarily say nothing about existence. Yet if the
conclusion is particular, then it does say something about existence. In which case, the conclusion contains more
information than the premises do, thereby making it invalid.

74 | P a g e
FLOWCHART FOR APPLYING THE SIX SYLLOGISTOC RULES

75 | P a g e
76 | P a g e
Expositions of the Fifteen Valid Forms of the Categorical Syllogism

Deduction of the Fifteen Valid Forms of the Categorical Syllogism

I. OBJECTIVES

At the end of the lesson, you are expected to:

a. Identify the moods and figures of valid forms of categorical syllogism


b. Explain the Traditional Latin Terms of Valid forms of categorical syllogism
c. To be able to eliminate all other forms that violates the basic rules of the syllogism by
d. Understand the importance of categorical syllogism

II. OVERVIEW AND SUMMARY

A. Form, Figure, Moods, Terms and Premise

The variety of structures in which a standard categorical syllogism may occur can be labeled by
stating its form.

The form of a categorical syllogism is the combination of its figure and mood.

The mood simply refers to the types of categorical propositions (A, E, I, or O) used in the syllogism,
listed in the order in which they appear in standard form.

The figure of a syllogism is its logical shape, as determined by the position of the term in its
premises.

If we let S represent the minor term, P the major term, and M the middle term, and leave out the
quantifiers and copulas, the four figures may be illustrated as follows:

77 | P a g e
Figure 1: the middle term is the subject term of the major premise and the predicate term of the
minor premise

Figure 2: the middle term is the predicate term of both premises

Figure 3: the middle term is the subject term of both premises

Figure 4: the middle term appears as the predicate term of the major premise and the subject term
of the minor premise

EXAMPLE:

All amphibians are cold-blooded vertebrates.

All frogs are amphibians.

Therefore, all frogs are cold-blooded vertebrates.

The terms used in our example are amphibians, cold-blooded vertebrates, and frogs. In a
categorical syllogism, the predicate term of the conclusion is called the major term (usually
represented by P) while the subject term is the minor term (S). In our example therefore, the major
term is cold-blooded vertebrates whereas the minor term is frogs.

The third term in the syllogism does not appear in the conclusion, but is employed in both premises.
It is the term that connects, relates, or mediates the two other terms, hence called middle term (M).

Aside from the conclusion, we have two premises in a categorical syllogism. One of them contains
the major term and is thus called the major premise. The other premise, which contains the minor
term, is referred to as the minor premise. “All amphibians are cold-blooded vertebrates” is the major
premise in our example.

In the above example, its logical form (mood and figure) is therefore AAA-1.

Considering all possible moods and figures, there are exactly 256 distinct forms of categorical
syllogism (four types of major premise multiplied by four kinds of minor premise multiplied by four
kinds of conclusion multiplied by four possible figures).

B. Names and structures of unconditionally valid syllogisms

A categorical syllogism is valid if it conforms to certain rules .

Meaning to say, if an argument violates at least one of these rules, it is invalid. On the other hand, if none
of the rules is broken, the syllogism is valid. For every rule that is violated, a specific formal fallacy is
committed

78 | P a g e
1. Boolean Standpoint

“If both premises are universal, the conclusion cannot be particular.”

This rule originated from the standpoint of British mathematician and logician George
Boole(1815-1864). Boole submits that universal statements do not contain existential import, i.e.
they do not claim whether or not the classes they refer to exist.

2. Validity by Analogy

The principles discussed (figure, mood, distribution, validity, rules, etc.) imply that the
validity or invalidity of a categorical syllogism can be identified by mere looking at its logical form
(mood and figure). Of the 256 distinct syllogistic forms, some are necessarily valid and some are
not, no matter what their contents happen to be. Every argument of the form AAA-1 is valid, for
example, while all syllogisms of the formOEE-3 are invalid.

We have therefore a clear-cut and practical method of demonstrating the validity (and
invalidity) of any syllogism by "logical analogy." Since logicians had already listed for us the
syllogistic forms which are valid, all we have to do is identify the mood and figure of a specific
argument and check it against the list. Of all the possible syllogistic forms, there are exactly 15
forms that are unconditionally valid. Thus, if an argument’s logical form exemplifies any of these
15 forms, then it is valid. If not, then it is invalid.

79 | P a g e
The following are the tables for valid forms and their assigned names.

EXAMPLES:

As the 15 forms are necessarily valid, it may be worthwhile to note them by name. And since these
forms serve as barometers or standard through which we determine whether a syllogism is valid or not, let
us provide here the structure or template of each forms. The letters P, S, and M refer to major term, minor
term, and middle term respectively.

80 | P a g e
1. AAA-1 is called Barbara. 5. AEE-2 is called Camestres. 9. AII-3 is called Datisi.
All M are P. All P are M. All M are P.
All S are M. No S are M. Some M are S.
Therefore, All S are P. Therefore, No S are P. Therefore, Some S are P.

2. AII-1 is called Darii. 6. AOO-2 is called Baroco. 10. IAI-3 is called Disamis.
All M are P. All P are M. Some M are P.
Some S are M. Some S are not M. All M are S.
Therefore, Some S are P. Therefore, Some S are not P. Therefore, Some S are P.

3. EAE-1 is called Celarent. 7. EAE-2 is called Cesare. 11. OAO-3 is Bocardo.


No M are P. No P are M. Some M are not P.
All S are M. All S are M. All M are S.
Therefore, No S are P. Therefore, No S are P. Therefore, Some S are not P.

4. EIO-1 is called Ferio. 8. EIO-2 is called Festino. 12. EIO-3 is Ferison.


No M are P. No P are M. No M are P.
Some S are M. Some S are M. Some M are S.
Therefore, Some S are not P. Therefore, Some S are not P. Therefore, Some S are not P.

81 | P a g e
13. AEE-4 is Camenes. 14. IAI-4 is Dimaris. 15. EIO-4 is Fresison.
All P are M. Some P are M. No P are M.
No M are S. All M are S. Some M are S.
Therefore, No S are P. Therefore, Some S are P. Therefore, Some S are not P.

III. SIGNIFICANCE AND APPLICATION

One needs the necessary knowledge in order to make the correct decisions. Likewise, one needs the
necessary logic skills, in order to make a correct reasoning. Stephen Rice in his writing entitled – “False Persuasion”
said,

“Readers and listeners are sometimes predisposed to accept arguments they read and hear simply because
they take certain logical forms, even where those forms are incontrovertibly illogical and cannot support
their purported

conclusion.

If psychological science has suggested that people are psychologically predisposed to accept
arguments that have logically invalid forms, then it seems that lawyers should be particularly attentive to
logical form. They should at least learn what logical form is .They should, at a minimum, arm themselves
with the capacity to recognize logically invalid arguments, lest they too find themselves (or their audience)
accepting a false argument due to psychological predisposition, rather than reasoned examination.”

Categorical syllogism, for one thing, provides us a conclusive yardstick in distinguishing correct from
incorrect reasoning.

Moreover, the valid forms of categorical syllogism can serve as your outline, if you wish to make a
persuasive essay or piece.

Sources:

Introduction to Logic- Copi & Cohen

http://ourhappyschool.com/philosophy/categorical-syllogism

https://digitalcommons.pace.edu/cgi/viewcontent.cgi?referer=https://www.google.com.ph/&httpsredir=1&article
=1848&context=plr

82 | P a g e
CHAPTER 7: SYLLOGISMS IN ORDINARY LANGUAGE

Syllogistic Arguments:
According to Copi and Cohen, syllogistic argument refers to any argument that either is a standard-form categorical
syllogism or can be reformulated as a standard-form categorical syllogism without any loss or change of meaning.

Purpose of reformulation:
There are arguments which are not in their proper or standard form categorical syllogism. We want to be able to
test the validity of these syllogistic arguments. If they are fallacious or misleading. According to Immanuel Kant
they can be easily detected if they are set out in correct syllogistic form. Reformulation is therefore important
because we cannot apply directly Venn diagrams and the rules of for categorical syllogisms if they are not in
their standard form.
 Reduction to standard form: The translation of syllogistic arguments in any form into the standard form
in which they can be tested for validity; also called translation to standard form.
 Standard-form translation,is the resulting argument when we reformulate a loosely put argument that
appears in ordinary language into a classical syllogism.
There are different ways in which a syllogistic argument in ordinary language may deviate from a standard-form
categorical argument.
 First deviation. The premises and conclusion of an argument in ordinary language may appear in an order
that is not the order of the standard-form syllogism. This can be remedied by reordering the premises: first
the major premise, second the minor premise, and third the conclusion. As discussed in chapter 6.
 Second deviation. A standard-form categorical syllogism always has exactly three terms. But the premises
may contain four or more terms which if we can reduced it to three terms without losing its original
meaning.
 Third deviation. The component proposition of the syllogistic argument in ordinary language may not all
be standard-from propositions.
Reducing the Number of Terms to Three
As discussed in chapter 6 a valid syllogism must have exactly three terms. If more than three terms seem to be
involved in an argument of apparently syllogistic form, it may be possible to translate the argument into a standard-
form categorical syllogism that is equivalent to it but that contains only three terms and is perfectly valid.
One way to reduced such terms is by Eliminating Synonyms. A synonym of one of the terms in the syllogism is
not really a fourth term, but only another way of referring to one of the three class involved. By using this method
we can use only one term rather than two different words which have the same meaning causing difficulty in
evaluating the premises (Copi and Cohen)
Example: Example:

83 | P a g e
No wealthy persons are vagrants.
All lawyers are rich people.
Therefore no attorneys are tramps.
The terms “wealthy” and “rich” are synonyms, as are “lawyer” and “attorney,” and also “vagrant” and “tramp.” If
the synonyms are eliminated, the argument becomes:
No wealthy persons are vagrants.
All lawyers are wealthy persons.
Therefore no lawyers are vagrants.
Thus, this argument in standard form, EAE-1 (Celarent), is plainly valid.
Another method is Second way is by Eliminating Class Complements. As discussed in chapter 5 a
complementary class or complement is the collection of all things that do not belong to the original class. When
complementary classes are present in the original argument, often we can reduce the number of terms by using
Immediate Interferences Conversion, Obversion, and Contraposition also discussed in chapter 5.
Example:
All mammals are warm-blooded animals.
No lizards are warm-blooded animals.
Therefore all lizards are nonmammals.
The terms “mammals” and “nonmammals” are complements of one another. So, by obverting the conclusion, the
argument becomes:
All mammals are warm-blooded animals.
No lizards are warm-blooded animals.
Therefore no lizards are mammals.
Thus, this argument in standard form, AEE-2 (Camestres), is known to be valid.
OR:
We take the contrapositive of the first premise, and we obvert the second premise. The argument becomes:
All non(warm-blooded animals) are nonmammals.
All lizards are non(warm-blooded animals).
Therefore all lizards are nonmammals.
This is also a valid translation; its form is AAA-1 (Barbara).
Also it is possible for us to reduced syllogistic arguments having more than four terms or as many as six terms
by using the immediate interferences.
More than one immediate inference may be needed to reduce the argument to standard form.
Consider this example:
No nonresidents are citizens.
All noncitizens are nonvoters.
Therefore all voters are residents.

84 | P a g e
The argument has six terms, but it is valid, and that can be shown by reducing it to standard form, which can be
done in more than one way. Perhaps the most natural reduction is to convert and then obvert the first premise. This
yields “All citizens are residents.” Then take the contrapositive of the second premise, which yields “All voters are
citizens.”
The argument is then in standard form:
All citizens are residents.
All voters are citizens.
Therefore all voters are residents.
The middle term (“citizens”) is the subject term of the major premise and the predicate term of the minor premise,
so the syllogism is in the first figure. Its three propositions are universal affirmatives. This is a syllogism in Barbara,
AAA–1, and it is plainly valid.
Translating Categorical Proposition into Standard Form

The four categorical propositions A, E, I, and O propositions are clearly somewhat stiff, and many
syllogistic arguments in everyday life contain nonstandard-form propositions. To reduce these arguments to
standard form requires that their constituent propositions be translated into standard form.
Copi and Cohen set guidelines rather than rules in which nonstandard-form propositions of certain describable kinds
can be reformulated into standard-form propositions that may serve as constituents of syllogistic arguments. But
we must keep in mind that in rem reformulating propositions we should not loss or change its original meaning.

The following are the nine methods given by Copi and Cohen:
1. Singular proposition: A proposition that asserts that a particular individual has (or does not have) some
specified attribute. Such propositions do not affirm or deny the inclusion of one class in another (as
standard-form propositions do)
Examples:
Socrates is a philosopher.
This table is an antique.
The above propositions do not affirm or deny the inclusion of one class in another, but we can nevertheless interpret
a singular proposition as a proposition dealing with classes and their interrelations.

Unit class. A class with only one member. Thus singular proposition may be translated into a universal
proposition as : “s is P” as if it were already expressed as the logically equivalent A proposition, “All S is P,” and
we similarly understand any negative singular proposition, “s is not P,” as an alternative formulation of the logically
equivalent E proposition, “No S is P”—in each case understanding S to designate the unit class whose only member
is the object s. As supported by Kant by saying that “in the employment of judgments in syllogisms, singular
judgments can be treated like those that are universal.”

85 | P a g e
In some cases, obviously valid two-premise arguments containing singular propositions translate into valid
categorical syllogisms, such as when

All H is M goes into the obviously valid All H is M.


S is an H. AAA-1 categorical syllogism in Barbara All S is H.
Therefore s is an M. Therefore All S is M.

In other cases, however, obviously valid two-premise arguments containing singular propositions translate into
categorical syllogisms that are invalid, such as when
s is M. goes into the invalid All S is M.
s is H. AAI-3 categorical syllogism All S in H.
Therefore Some H is M. Therefore All H is M.
which commits a existential fallacy, violation Rule 6.

On the other hand, if we translate singular propositions into particular propositions, there is the same kind of
difficulty. In some cases, obviously valid two-premise arguments containing singular propositions translate into
valid categorical syllogisms, such as when
All H is M. goes into the obviously valid All H is M.
s is an H. AII-1 categorical syllogism is Darii Some S is H.
Therefore s is an M. Therefore Some S is M.
In other cases, however, obviously valid two-premise arguments containing singular propositions translate into
categorical syllogisms that are invalid, such as when
s is M. goes into the invalid Some S is M.
s is H. III-3 categorical syllogism Some S is H.
Therefore Some H is M. Therefore Some H is M.
which commits the fallacy of the undistributed middle, violating Rule 2.

The difficulty arises from the fact that a singular proposition contains more information that is contained in any
single one of four standard-form categorical propositions. The solution to the difficulty is to construe singular
propositions as conjunctions of standard-from categorical propositions. And we can evaluate its validity by using
Venn diagram or by applying the syllogistic rules.
In other books, singular proposition A proposition that asserts something about a specific person, place, or thing.
Singular propositions can be translated into universals by use of what are called parameters (phrases that affect the
form but not the meaning of the statement).
Examples of parameters include: persons identical to, places identical to, things identical to, cases identical to,
times identical to

86 | P a g e
For instance,
“Meghan is a computer programmer”
should say
“All people identical to Meghan are computer programmers.”

NOW we have a statement of the form “All S are P” where both S and P are CLASSES of things. “People identical
to Meghan” represents a category that things can be a member of. As it turns out, the only thing that belongs in the
category of “people identical to Meghan” is Meghan herself (one SINGLE person)—so there is only ONE thing in
this class.

Note: This might all seem a little confusing right now, but realize it’s all about getting each proposition into the
correct FORM, so that, later, we can construct valid arguments out of them.

2. Categorical Propositions That Have Adjectives or Adjectival Phrases as Predicates, Rather than
Substantives or Class Terms.
When a categorical proposition is in standard form except that it has an adjectival predicate instead of a
predicate term, the translation into standard form is made by replacing the adjectival predicate with a term
designating the class of all objects of which the adjective may truly be predicated.
Example :
“Some flowers are beautiful” and “No warships are available for active duty” should be translated to “Some
flowers are beautiful things” and “No warships are things available for active duty.”

III. Categorical Propositions Who’s Main Verbs Are Other than the Standard-Form Copula “To Be.”
Examples:
All people seek recognition.
Some people drink Greek wine.
the words are and are not are called copulas because they connect the subject and predicate (same root as copulate).
No other words can be used as copulas. The copula is needed to connect the subject and predicate terms, The words
can be replaced by a term designating the class determined by that class-defining characteristic and may be linked
to the subject with a standard copula. Thus, those two statements above can be translated into the standard-form
categorical propositions.
All people are seekers of recognition.
Some people are Greek-wine drinker.

87 | P a g e
IV. Statements in Which the Standard-Form Ingredients Are All Present But Not Arranged in Standard-
Form Order.
Examples:
Racehorses are all thoroughbreds.
All is well that ends ends well.
In those cases, they have both all the terms and ingredients but not arranged in standard form. We first decide which
the subject term is and then rearrange the words to express a standard-form categorical proposition. Such
translations are usually quite straightforward. It is clear that the two preceding statements translate into the A
propositions
All racehorses are thoroughbreds.
All things that end well are things that are well.

VI. Categorical Propositions Whose Quantities Are Indicated by Words Other than the Standard-
Form Quantifiers “All,” “No,” and “Some.”

The words all, some and no are called quantifiers because they determine the quantity (how much) of the subject
class is or is not in the predicate class. No other words can be used as quantifiers in standard form propositions.
Propositions that uses other words other than Standard-Form Quantifiers can be translated by considering its
context.
Example: “Every dog has its day” is translated to “All dogs are creatures that have their days”
Note that we changed the subject and predicate into plural.
 The grammatical particles “a” and “an” may also serve to indicate quantity, but whether they are being used
to mean “all” or “some” depends largely on the context.
 The particle “the” may be used to refer either to a particular individual or to all members of a class.

VII. Exclusive Propositions

Propositions that assert that the predicate applies exclusively to the subject named. Propositions beginning
with “only” or “none but” usually translate into A propositions using this general rule: Reverse the subject and the
predicate, and replace the “only” with “all.” Thus “Only S is P” and “None but S’s are P’s” are usually
understood to express “All P is S.”
Example
“None but red trucks are fire engines” translates to “All fire engines are red things.”
“Only citizens can vote” translates to “All those who can votes are citizens.”

88 | P a g e
VII. Categorical Propositions That Contain No Words to Indicate Quantity.

Since every categorical proposition involves a quantifier (universal or particular), it is important to rewrite the
quantifier in standard-form. We may be able to determine its meaning only by examining the context in which it
occurs.
Example:
“Dogs are carnivorous” translates to “All dogs are carnivores.” Because it is probable that it refers to all dogs.
VIII. Propositions That Do Not Resemble Standard-Form Categorical Propositions But Can Be
Translated into Standard Form.

Examples:
Not all children believe in Santa Claus.
There are white elephants.
There are no pink elephants.
Nothing is both round and square.
On reflection, the above propositions will be seen to be logically equivalent to, and therefore to translate into, the
following standard-form propositions:
Some children are not believers in Santa Claus.
Some elephants are white things.
No elephants are pinks things.
No round objects are square objects.

IX. Exceptive Propositions

A proposition that asserts that all members of some class, with the exception of the members of one of its
subclasses, are members of some other class. Exceptive propositions are in reality compound, because they
assert both a relation of class inclusion, and a relation of class exclusion.
Example: “All persons except employees are eligible” is an exceptive proposition in which it is asserted both
that “All nonemployees are eligible” and that “No employees are eligible.
How an argument containing an exceptive proposition should be tested depends on the exceptive proposition’s
position in the argument. If it is a premise, then the argument may have to be given two separate tests.
For example, consider the argument:
Everyone who saw the game was at the dance.
Not quite all the students were at the dance.
So some students didn’t see the game.

89 | P a g e
To translate this we must first identify which is the exceptive proposition and first test the syllogism composed
of the first premise of the given argument, the first half of its second premise, and its conclusion. In standard
form, we have
All persons who saw the game are persons who were at the dance.
Some students are persons who were at the dance.
Therefore some students are not persons who saw the game.
The standard-form categorical syllogism is of form AIO–2 and commits the fallacy of the undistributed middle,
violating Rule 2.
We now have to test the categorical syllogism composed of the first premise and the conclusion of the original
argument together with the second half of the second premise. In standard form we then get a very different
argument:
All persons who saw the game are persons who were at the dance.
Some students are not persons who were at the dance.
Therefore some students are not persons who saw the game.
This is a standard-form categorical syllogism in Baroko, AOO–2, and it is easily shown to be valid. Hence the
original argument is valid

Uniform Translation

According to the book of Irving Copi and Carl Cohen entitled Introduction to Logic (13th edition), in order
to achieve the uniform translation of all three propositions contained in a categorical syllogism, it is sometimes
useful to modify each of the terms employed in an ordinary-language by stating it in terms of a parameter defined
as an auxillary symbol that aids in reformulating an assertion into standard form. The goal as always, is to represent
the intended meaning of each of the offered propositions, while at the same time bringing it into conformity with
the others, making it possible to restate the whole as a standard-form syllogism.

Example
Soiled paper plates are scattered only where careless people have picknicked.
There are soiled paper plates scattered about here.
Therefore careless people must have been picknicking here

Solution
All places where soiled paper plates are scattered are places where careless people have picknicked.
This place is a place where soiled paper plates are scattered.
Therefore this place is place where careless people have picknicked.

90 | P a g e
This standard-form categorical syllogism is in Barbara with mood form AAA-1 and has already been proved valid.

Enthymemes
It is established that enthymeme is an argument containing an unstated proposition. And an incompletely stated
argument is characterized as being enthymematic.

Example
Jone is a native-born American.
Therefore Jones is a citizen.
Missing premise
All native-born Americans are citizens.

Traditionally, enthymemes have been divided into different orders according to which part of the syllogism is left
unexpressed. A first-order enthymeme is one in which the syllogism’s major premise is not stated. A second-order
enthymeme is one in which only the major premise and the conclusion are stated, the minor premise being
suppressed.
Example
All students are opposed to the new regulations.
Therefore all sophomores are opposed to the new regulations.
Missing minor premise
All sophomores are students.

A third-order enthymeme is one which both premises are stated, but the conclusion is left unexpressed.

Example
No true Christian is vain, but some churchgoers are vain.
Infer the conclusion
Some churchgoers are not true Christians.

Sorites
It is observable that sometimes, a single categorical proposition will not suffice for drawing a desired
conclusion from a group of premises. The evidence for a conclusion consists of more than two propositions. The
inference is not a syllogism In such cases but a series of syllogisms.
Example

91 | P a g e
All diplomats are tactful.
Some government officials are diplomats.
All governtment officials are people in public life.
Solution
All diplomats are tactful individuals.
Some government officials are diplomats.
Therefore some government officials are tactful individuals.
All government officials are people in public life.

The argument is a chain of syllogisms in which the conclusion of one becomes a premise of another.
However, the conclusions of all the syllogisms except the last remain unexpressed. Thus, a sorite is a chain of
syllogisms in which the conclusion of one is a premise in another, in which all the conclusions except the last one
are unexpressed, and in which the premises are so arranged that any two successive ones contain a common term.

Chapter 7.7 Disjunctive and Hypothetical Syllogisms

Propositions are considered categorical when they affirm or deny the inclusion or exclusion of categories
or classes. Syllogisms, which is an argument that consists of two premises and a conclusion, are called categorical
when the propositions they contain are categorical. A syllogism may contain propositions that are not categorical.
Such cases are not called categorical syllogisms but are instead named on the basis of the kind of propositions they
contain. (Copi & Cohen, 2009)
Categorical propositions can be considered as simple propositions for they have a single component which
affirms or denies some class relation. In contrast, some propositions are compound statements, containing more
than one component.
Consider first the Disjunctive (or Alternative) Proposition as compound proposition. An example is “She
was driven either by stupidity or arrogance.” Its two components are “she was driven either by stupidity” and “she
was driven by arrogance.” The disjunctive proposition contains those two component propositions, which are called
its disjuncts. The disjunctive proposition does not categorically affirm the truth of either one of its disjuncts, but
says that at least one of them is true, allowing for the possibility that both may be true. (Copi & Cohen, 2009)
If we have a disjunction as one premise, and as another premise the denial or contradictory of one of its two
disjuncts, then we can validly infer that the other disjunct in that disjunction is true. Any argument of this form is a
valid disjunctive syllogism. But not every disjunctive syllogism is valid. The argument
She was either arrogant or stupid.
She was arrogant.

92 | P a g e
Therefore she was not stupid.
The foregoing is an example of what may be called as an invalid disjunctive syllogism. We readily see that even if
the premise were true, she may have been arrogant and stupid. The truth of one disjunct of a disjunction does not
imply the falsehood of the other disjunct, because both disjuncts of a disjunction can be true. We have a valid
disjunctive syllogism, therefore, only where the categorical premise contradicts one disjunct of the disjunctive
premise and the conclusion affirms the other disjunct of the disjunctive premise.
Some might put forth an argument like this, to rebut the above point:
Either Smith is in New York or Smith is in Paris.
Smith is in New York.
Therefore, Smith is not in Paris.
Here, the categorical premise affirms one disjunct of the stated disjunction, and the conclusion contradicts
the other disjunct, yet the conclusion seems to follow validly. Notice, however, that the disjunctive doesn't actually
play a role in the argument. The argument, in fact, relies upon an unstated enthymeme: "Smith cannot be both in
New York and in Paris", which can be stated in disjunctive form as "Either Smith is not in New York or Smith is
not in Paris" If we replace P1 with this statement, then the above disjunctive syllogism follows the proper form and
affirms the validity of proper form.
In Elements of Logic translated by Ewan Macpherson, the author defines disjunctive syllogism as a
syllogism which has for its major a disjunctive proposition, which not merely alleges an incompatibility, but implies
an alternative admitting no middle term. Accordingly, the disjunctive syllogism is governed by the following two
rules:
(1) The disjunction laid down in the major must be complete.
(2) When the minor affirms one of the members of the disjunction, the remaining member or members must be
denied in the conclusion, and vice versa.
Example: Every free act is morally good or bad. Now such or such an act (e. g., an oath) is not morally bad; therefore
it is morally good. Or, it is bad; therefore it is not good. Or, it is good; therefore it is not bad. Or, it is not good;
therefore it is bad (Mercier, 1912).

The second kind of compound proposition we consider is the Conditional (or Hypothetical) proposition,
an example of which is “If the first native is a politician. Then the first native lies.” A conditional proposition
contains two component propositions: the one following the "If" part is the antecedent, and the part following the
"Then" is the consequent. A syllogism that contains conditional statements exclusively is called a pure hypothetical
syllogism.
For example,
If the first native is a politician, then he lies.
If he lies, then he denies being a politician.
Therefore if the first native is a politician, then he denies being a politician.

93 | P a g e
In this argument, it can be observed that the first premise and the conclusion have the same antecedent, that the
second premise and the conclusion have the same consequent, and that the consequent of the first premise is the
same as the antecedent of the second premise. It should be clear that any pure hypothetical syllogism whose
premises and conclusion have their component parts so related is a valid argument.
A syllogism that has one conditional premise and one categorical premise is called a mixed hypothetical
syllogism. Two valid forms of the mixed hypothetical syllogism have been given special names. The first is
illustrated by:
If the second native told the truth, then only one native is a politician.
The second native told the truth.
Therefore only one native is a politician.
Here the categorical premise affirms the antecedent of the conditional premise, and the conclusion affirms its
consequent. Any argument of this form is valid and is said to be in the affirmative mood or modus ponens (from the
Latin ponere meaning “to affirm”). One must not confuse the valid form modus ponens with the clearly invalid
form displayed by the following argument:
If Bacon wrote Hamlet, then Bacon was a great writer.
Bacon was a great writer.
Therefore Bacon wrote Hamlet.
This argument differs from modus ponens in that its categorical premise affirms the consequent, rather than the
antecedent, of the conditional premise. Any argument of this form is said to commit the fallacy of affirming the
consequent.
The other valid form of mixed hypothetical syllogism is illustrated by:
If the one-eyed prisoner saw two red hats, then he could tell the color of the hat on his own head.
The one-eyed prisoner could not tell the color of the hat on his own head.
Therefore the one-eyed prisoner did not see two red hats.
Here the categorical premise denies the consequent of the conditional premise, and the conclusion denies its
antecedent. Any argument of this form is valid and is said to be in the form modus tollens (from the Latin tollere,
meaning “to deny”). One must not confuse the valid form modus tollens with the clearly invalid form displayed by
the following argument:
If Carl embezzled the college funds, then Carl is guilty of a felony.
Carl did not embezzle the college funds.
Therefore Carl is not guilty of a felony.
This argument differs from modus tollens in that its categorical premise denies the antecedent, rather than the
consequent, of the conditional premise. Any argument of this form is said to commit the fallacy of denying the
antecedent.
To simply say, the whole interest of the conditional syllogism, then, is in the major, which is equivalent to an
absolute affirmative proposition. The proposition, "If the soul is simple, it is imperishable," is equivalent to, "Every

94 | P a g e
simple thing is imperishable." Now a universal affirmative is not convertible. From this observation we deduce the
rules of the conditional syllogism:
(1) Affirm the condition, or antecedent, and you must affirm the conditioned proposition, or consequent. E. g.: If
you are from Brussels, you are a Belgian. But you are from Brussels. Therefore you are a Belgian.
(2) Deny the conditioned proposition, or consequent, and you must deny the condition, or antecedent. E. g.: If you
are from Brussels, you are a Belgian. But you are not a Belgian. Therefore you are not from Brussels (Mercier,
1912).

Chapter 7.8 The Dilemma


The dilemma is a common form of argument in ordinary language. It is, in essence, an argumentative device
in which syllogisms on the same topic are combined, sometimes with devastating effect. Each of the constituent
syllogisms may be quite ordinary, and therefore the dilemma is not of special importance from a strictly logical
point of view. But the premises of the syllogisms so combined are formulated disjunctively, and devised in a way
designed to trap the opponent by forcing him to accept one or the other of the disjuncts. Thus the opponent is forced
to accept the truth of the conclusion of one or the other of the syllogisms combined. When this is done successfully,
the dilemma can prove to be a powerful instrument of persuasion.
In other reference the dilemma is defined as the combination of a disjunctive proposition, serving as major,
with two or more conditional propositions forming a minor. First, partial conclusions exclude the members of the
disjunction one after another; then it is concluded in a general manner that the disjunctive proposition taken as a
whole is inadmissible. This method of arguing is lively and cogent. An alternative is presented to one's opponent:
he is left the choice between two positions; then it is proved that in either case he is wrong. The validity of the
dilemma requires a punctual observance of the rules of the disjunctive and of the conditional syllogisms which are
as follows:
First rule: The disjunction of the major admits of no intermediary proposition, but must be complete.
Second rule: Each of the two conditional syllogisms which together form the minor of the dilemma must
be conclusive, and must lead to the same conclusion.
Example (from Père Félix) : "If we supposed that Jesus Christ, in spite of His own assertions, is not God, we should
be led to one of these two insulting conclusions: that He is a madman; or that He is an impostor. Now, supposing
Jesus Christ to be insane, how can we reconcile with insanity the lofty wisdom manifested in His life and doctrine?
Supposing Him an impostor, how make His humility and abnegation agree with such ambitious designs? Both these
hypotheses, therefore, are equally inadmissible: Jesus Christ is the Christ. the Son of the living God.”
It is easy to show that the syllogisms are fundamentally reducible to the categorical syllogism. Furthermore The
dilemma must not be confounded with reasoning "by successive parts", which consists in enumerating all the species
of a genus, to take them up afterwards one by one and finally enunciate of all the conclusion which is valid for each
of the parts (Mercier, 1912).

95 | P a g e
A person is “in” a dilemma or (“impailed on the horns of a dilemma”) when that person must choose
between two alternatives, both of which are bad or unpleasant. The dilemma is a form of argument intended to put
one’s opponent in just that kind of position. In debate, one uses a dilemma to offer alternative positions to one’s
adversary. From which a choice must be made, and then to prove that no matter which choice is made, the adversary
is committed to an unacceptable conclusion.
A physicist named Richard Feynman was caustic in his criticism of mismanagement by administrators in
the National Aeronautics and Space Administration. He said:
Every time we talked to higher level managers, they kept saying they didn’t know anything about the problems
below them. Either the group at the top didn’t know, in which case they should have known, or they did know, in
which case they were lying to us.
An attack of this kind is designed to push the adversaries (in this case the NASA administrators) into a corner and
there annihilate them. The only explicitly stated premise of the argument is a disjunction but one of the disjuncts
must obviously be true; Either they knew or they didn’t know about the problems below them. And whichever
disjunct is chosen, the result for the adversary is very bad. The conclusion of a dilemma can itself be a disjunction
(for example, “Either the NASA administrators did not know what they should have known, or they lied”) in which
case the dilemma is called complex dilemma. But the conclusion may also be a categorical proposition, in which
case we call it simple dilemma.
A dilemma need not always have an unpleasant conclusion. An example of one with a happy conclusion is
provided by the following simple dilemma:
If the blest in heaven have no desires, they will be perfectly content; so they will be also if their desires are fully
gratified; but either they will have no desires, or have them fully gratified; therefore they will be perfectly content.
The premises of a dilemma need not be stated in any special order; the disjunctive premise that offers the
alternatives may either precede or follow the other. And the consequences of those alternatives may be stated in a
conjunctive proposition or in two separate propositions. An argument in dilemma form is often expressed
enthymematically; that is; its conclusion generally is thought to be so obvious that it secretly needs to be spelled
out. This is well illustrated in a passage from a letter of President Abraham Lincoln, defending the Emancipation
Proclamation that freed slaves of the Confederacy:
But the proclamation, as law, either is valid, or is not valid. If it is not valid, it needs no retraction, If it is valid, it
cannot be retracted, any more than the dead can be brought to life.

Forms of dilemma
A. CONSTRUCTIVE DILEMMA
Having affirmed a conjunction of two conditional statements and a disjunction of their antecedents, a disjunction
of their respective consequents logically follows. For example, given that the following statements are true,

96 | P a g e
If you study your lessons well, then you will pass all your subjects, and if you get high grades, then you will graduate
with honors. Either you study your lessons well or you get high grades.
A conclusion can be made such that,
Either you will pass your subjects or you will graduate with honors.
Constructive Dilemma can be symbolically seen as:
1. (p → q) & (r → s) - Conjunction of two Conditionals
2. (p / r) - states the disjunction of the two conditions
∴ (q / s) - the new derived statement from the two statements

B. DESTRUCTIVE DILEMMA
Having affirmed a conjunction of two conditional statements and a disjunction of the denial of their
consequents, a disjunction of the denial of their respective antecedents logically follows.
For example, given that the following statements are true,
If you are conscious of your health, then you will stay fit, and if you exercise every day, you will become
stronger. Either you are not fit or you are not conscious.
A conclusion can be made such that,
Either you are not conscious of your health or you don’t exercise every day.
Desdtructive Dilemma can be symbolically seen as:
1. (p → q) & (r → s) - Conjunction of two Conditionals
2. (~q/~s) - states the disjunction of the two conditions
∴ (~p/~r) - the new derived statements

Three possible patterns of rhetorical response


The three possible ways of defeating a dilemma are known as “going (or escaping between the horns,”
“Taking (or grasping) it by the horns.” and “rebutting it by means of a counterdilemma.” Note that these ways are
not ways to prove the dilemma invalid; rather, they are ways in which one seeks to avoid its conclusion without
challenging the formal validity of the argument.
One escapes between the horns of a dilemma by rejecting its disjunctive premise. This method is often the
easiest way to evade the conclusion of a dilemma, for unless one half of the disjunction is the explicit contradictory
of the other, the disjunction may very well be false. For example, one justification sometimes offered for giving
grades to students is that recognizing good work will simulate the students to study harder. Students may criticize
this theory using the following dilemma:
If students are fond of learning, they need no stimulus, and if they dislike learning, no stimulus will be of any avail.
But any student is either fond of learning or dislikes it. Therefore a stimulus is either needless or of no avail.
This argument is formally valid, but we can evade its conclusion by going between the horns. The disjunctive
premise is false, for students have all kinds of attitudes toward learning: Some may be fond of it, may dislike it, and

97 | P a g e
may be indifferent. For that third group, a stimulus may be both needed and of some avail. Going between the horns
does not prove the conclusion to be false but shows merely that the argument does not provide adequate grounds
for accepting the conclusion.
When the disjunctive premise is unassailable, as when the alternatives exhaust the possibilities, it is
impossible to escape between the horns. Another method of evading the conclusion must be sought. One such
method is to grasp the dilemma by the horns, which involves rejecting the premise that is a conjunction. To deny a
conjunction, we need only deny one of its parts. When we grasp the dilemma by the horns, we attempt to show that
at least one of the conditionals is false. The dilemma just above, attacking the use of grades in school, relies on the
conditional “If students are fond of learning, they need no stimulus.” The proponent of grading may grasp this
dilemma by the horns and argue that even students who are fond of learning may sometimes need stimulus, and that
the additional stimulus provided by grades promotes careful study by even the most diligent students. There may
be good response to this, of course—but the original dilemma has been grasped firmly by the horns.
Rebutting a dilemma by means of a counterdilemma is the most ingenious method of all, but it is seldom
cogent, for reasons that will appear presently. To rebut a given dilemma in this way, one constructs another dilemma
whose conclusion is opposed to the conclusion of the original. Any counterdilemma may be used in rebuttal, but
ideally it should be built up out of the same ingredients (categorical propositions) that the original dilemma
contained.
A classical example of this elegant kind of rebuttal concerns the legendary argument of an Athenian mother
attempting to persuade her son not to enter politics:
If you say what is just, men will hate you; and if you say what is unjust, the gods will hate you; but you must either
say the one or the other; therefore you will be hated.
Her son rebutted that dilemma with the following one:
If I say what is just, the gods will love me; and if say what is unjust, men will love me. I must say either the one or
the other. Therefore I shall be loved!
Where the dilemma is one of the strongest weapons of controversy, the use of a rebuttal of this kind, which
derives an opposite conclusion from almost the same premises, is a mark of great rhetorical skill. But if we examine
the dilemma and rebutting counterdilemma more closely, we see that their conclusions are not as opposed as they
might at first have seemed.
The conclusion of the first dilemma is that the son will be hated (by men or by the gods), whereas that of
the rebutting dilemma is that the son will be loved (by the gods or by men). But these two conclusions are perfectly
compatible. The rebutting counterdilemma serves merely to establish a conclusion different from that of the original.
Both conclusions may very well be true together, so no refutation has been accomplished. But in the heat of
controversy analysis is unwelcome, and if such a rebuttal occurred in a public debate, the average audience might
agree that the rebuttal was an effective reply to the original argument.
This sort of rebuttal does not refute the argument but only directs attention to a different aspect of the same
situation is perhaps more clearly shown in the case of the following dilemma, advanced by an “optimist”:

98 | P a g e
If I work, I earn money, and if I am idle, I enjoy myself. Either I work or I am idle. Therefore either I earn money
or I enjoy myself.
A “pessimist” might offer the following counterdilemma:
If I work, I don’t enjoy myself, and if I am idle, I don’t earn money. Either I work or I am idle. Therefore either I
don’t earn money or I don’t enjoy myself.
These conclusions represent merely different ways of viewing the same facts; they do not constitute a disagreement
over what the facts are.
The discussion of dilemma would not be completed unless it mentioned the celebrated lawsuit between
Protagoras and Euathlus. Protagoras is a teacher who lived in Greece during the fifth century B.C., specialized in
teaching the art of pleading before juries. Euathlus wanted to become a lawyer, but not being able to pay the required
tuition, he made an arrangement according to which Protagoras would teach him but not receive payment until
Euathlus won his first case. When Euathlus finished course of study, he delayed going into practice. Tired of waiting
for his money, Protagoras brought suit against his former pupil for the tuition money that was owed. Euathlus
decided to plead his own case. When the trial began, Protagoras presented his side of the case in a crushing dilemma:
If Euathlus loses this case, then he must pay me (by the judgment of the court); if he wins this case, then he must
pay me (by the terms of the contract). He must either lose or win this case. Therefore Euathlus must pay me.
The situation looked bad for Euathlus, but he had learned well the art of rhetoric. He offered the court the following
counterdilemma in rebuttal:
If I win this case, I shall not have to pay Protagoras (by the judgment of the court); if I lose this case, I shall not
have to pay Protagoras (by the terms of the contract, for then I shall not yet have won my first case). I must either
win or lose this case. Therefore I do not have to pay Protagoras!
Note that the conclusion of Euathlus’s rebutting dilemma is not compatible with the conclusion of Protagoras’s
original dilemma. One conclusion is the explicit denial of the other. However, it is rare that a counterdilemma stands
in this relation to the dilemma against which it is directed. When it does, the premises involved are themselves
inconsistent, and it is this implicit contradiction that the two dilemmas make explicit. (Copi & Cohen, 2009)

References:
Copi, I. M., & Cohen, C. (2009). Introduction to Logic 13th Edition. Jurong, Singapore: Pearson Education Inc.
Copi, I. M., & Cohen, C. (2014). Introduction to Logic 14th Edition. Jurong, Singapore: Pearson Education Inc.
Mercier, C. (1912). Elements of Logic. New York: The Manhattanville Press.
Logic-6. (n.d., n.d. n.d.). Retrieved March 28, 2018, from http://bgillette.com/wp-content/uploads/2014/08/Logic-
6.pdf
Stan,B.. (n.d). Logic Second Edition in. Retrieved March 28, 2018,
http://global.oup.com/us/companion.websites/9780199846313/student/chapter5/guide/
Mathhew,B.(February 13 2015). Translating Ordinary Language into Categorical Form

99 | P a g e
in. Retrieved April 3, 2018, from https://prezi.com/zhhhgehwvxp4/translating-ordinary-language-into-categorical-
form/
Ronald, H.(n.d). Predicate Logic Retrieved in April 3, 2018, from
http://www.stetson.edu/artsci/philosophy/media/Chapter%2010.pdf
(n.d) Contructing Categorical Proposition . Retrieved in April 3 2018 from
http://rintintin.colorado.edu/~vancecd/phil1440/catprop2.pdf

100 | P a g e
CHAPTER 8: SYMBOLIC LOGIC

In studying logic, we look for ways how to fully understand deductive reasoning. In search for these ways,
two logical theories were made. The first is the classical or Aristotelian logic – which we had studied on the first
chapters – and the second is the modern or Symbolic Logic (Copi and Cohen 2010, p. 315).
We note that classical and modern logic are two different theories. While in traditional logic, we deal mostly
on the study of syllogism, and categorical propositions, modern logic does not (Cothran 2012). Modern logic uses
different techniques and concepts as those used by the classical logic though they are of the same objective (Copi
and Cohen 2010, p. 315). Therefore, in studying the modern or symbolic logic, we shall have to temporarily forget
and start fresh whatever we have learned in the classical logic.
As it is called “Symbolic logic”, symbols will take a great part in studying this great body of logic. Symbol,
according to the Merriam-Webster Dictionary (2018), is “something that stands or suggests something else by
reason of relationship, association, convention, or accidental resemblance”. We use and encounter symbols in our
daily lives. They make a lot of things more understandable. This, too, applies to Symbolic logic. Symbols (or
artificial language) are used in Symbolic logic to avoid the difficulties of vagueness, equivocation, amphiboly, and
confusion from emotive significance of our natural language (philosophy.lander.edu 2004).
Symbols greatly affect our way of thinking and understanding arguments. They help us know what the real
meaning of the argument is and separate what is important from what is not. Further, symbols help us perform easier
some logical operations which originally may demand great effort (Copi and Cohen 2009, p. 316).
As we begin a new chapter about the system of modern logic, we now open the broader possibilities of
understanding forms of deductive arguments which traditional logicians may hardly address. Symbolic logic, with
the use of its more versatile symbolic language, will allow us to get to the heart of deductive arguments and
understand them deeply. With the use of the fundamental logical connectives – symbol or word used to connect two
or more sentences (New World Encyclopedia 2014) – we can achieve the principal objective of deductive logic
which is to distinguish valid arguments from invalid ones.

THE SYMBOLS FOR CONJUNCTION, NEGATION, AND DISJUNCTION


Before we discuss about these symbols, we must keep in our minds that one of the differences of modern
logic from traditional logic is that in traditional logic, the proposition “All dinosaurs are funny creatures” is
symbolized as “All S are P” while in modern logic, this proposition is symbolized as “D” which means the letter
represents and entire proposition. (teachpilosophy 2013). Hence the explanation that in modern logic, we are not
dealing with classes but with the internal structure of propositions and arguments (Copi and Cohen 2009, p. 316).
The letters representing the propositions are either true or false which means that the proposition “All
dinosaurs are funny creatures”, for example, may either be true or false. However, there are some statements which
do not make truth claims like “Who are you?” or “Stop!”. These sentences cannot be translated into modern logic
(teachphilosophy 2013).

101 | P a g e
In Modern logic, the arguments generally have at least one compound statement. To better understand these
arguments, we divide all statements into two general categories – simple and compound statements. A simple
statement does not have any other statement as a component (Copi and Cohen 2009, p. 317). For example, the
sentence “She is beautiful” is a simple statement. On the other hand, compound statements are those which have
two simple statements as components. Therefore, the sentence “She is beautiful and she is kind” is a compound
statement because such statement may be separated to create two simple statements and each statement may
represent a different truth claim.
However, a component must not be understood simply as “a part of a statement which is in itself a
statement”. This is so because a statement may be a part of a larger statement but is still not considered as a
component (Copi and Cohen 2009, p. 317). For example, in the sentence “The one who saved Harry was a wizard”,
the words “Harry was a wizard” are a part of the larger statement and may also either be true or false. But still, it is
not a component. This is so because according to Copi and Cohen (2009, p. 317) in order for a statement to be
component, there are two requisites: 1) the part must be a statement in its own right; and 2) if the part is replaced in
the larger statement by another statement, the result of that replacement must be meaningful – it must make sense.

A. Conjunction
The first type of compound statement is the conjunction. Conjunction statements are two statements
connected by the word “and” between them and these two statements are called conjuncts (Copi and Cohen 2009,
p. 318). For example, the statement “She is beautiful and she is kind” is a compound statement where the first
conjunct is “She is beautiful” and the second conjunct is “She is kind”. In symbolic logic, the conjunction is
represented in symbol as the dot “ “ (Intodution to Logic 2004). To explain further, the conjuncts “She is beautiful”
and “She is kind” may be represented by “B” and “K”, respectively. And to illustrate their conjunction, we write
“B K”.
Since every statement is either true or false, each statement, therefore, has a truth value – true for true
statements and false for false statements. With this, compound statements may be divided into two categories.
Where the truth value of a compound statement is determined completely by the individual truth values of the
component statements of the compound, such compound statement is truth-functional (Truth Functional
Connectives n.d.). This is the case for conjunction statements. For example, in the conjunction “B K”, if the
conjuncts “B” and “K” are true, then we can automatically say that the conjunction is true, otherwise, it is false.
Truth-functional compounds are those which by simply looking at the truth value of each of the components, we
can already determine the truth value of the whole compound statement. Thus, the statement is called a truth-
functional compound statement and the components as truth-functional components (Copi and Cohen 2009, p. 318).
However, not all compound statements are truth-functional.

102 | P a g e
A conjunction, being a truth-functional connective, having two statements B and K, there can only be four
possible sets of truth value which they can have. These are:
Where B is true and K is true, B K is true.
Where B is true and K is false, B K is false.
Where B is false and K is true, B K is false.
Where B is false and K is false, B K is false.

If we put it in a truth table with “true” and “false” represented by “T” and “F”:

To simply write conjunctions, we may abbreviate “She is beautiful and she is kind” B K. Conjunctions
which have similar subject terms may be written by placing “and” between the predicate terms, as the example
above may be written “She is beautiful and kind.” On the other hand, compound statements which have the same
predicate terms may be written simpler by placing “and” between the subject terms (Copi and Cohen 2009, p. 319-
320). For example, the compound statement “Mathematics is a difficult subject and Logic is a difficult subject”
may be written as “Mathematics and Logic are difficult subjects.
Other words which may also be used aside from the word “and” for conjunctive statements are “but”, “yet”,
“also”, “still”, “although”, “however”, “moreover”, “nevertheless” and so on. The comma and semi-colon may also
be used (Copi and Cohen 2009, p.320).

B K B•K

T T T
T F F
F T F
F F F

B. Negation
The negation (denial or contradictory) of a statement means the denial of the original statement (Slaney
2014). It is formed by placing the word “not” prior to the original statement (Copi and Cohen 2009, p. 320). For
example, the sentence “Bob is happy” has the negation “Bob is not happy”. However, though the word “unhappy”
may be synonymous to “not happy”, we cannot attach the prefix “un” to from the negation as “Bob is unhappy”
because it is not really the negation of “Bob is happy” for Bob may neither be happy nor unhappy but is only neutral
in respect of happiness (Slaney 2014). The negation statement is formed by the symbol “ ~ “ or tilde, or in some
books, “ – “ may also be used. Negation turns truths into falsehoods and falsehoods into truths (Slaney 2014).
Negation is likewise truth-functional but since it is not a connective for it does noy connect two statements, logicians

103 | P a g e
refer to it as truth-functional operator (Copi and Cohen 2009, p. 320). Therefore, if a statement M is true, then we
know that ~M is false. And if M is false, ~M is true. To illustrate it in a truth table:

C. Disjunction
Disjunction (alternation) is likewise a truth-functional connective and is formed when two statements are
connected by the word “or” between them (Truth Functional Connectives n.d.). The components in a disjunction
are called disjuncts (or alternatives). The symbol for disjunction is “ ∨” (Copi and Cohen 2009 p. 321). The example
“You will go to Boracay or you will go to Tagaytay” may therefore be written as “B ∨
T”. M ~M

In English, the word “or” may be used in two different ways which have two T F
different meanings or senses – the exclusive sense or inclusive sense. An example of its F T
exclusive sense is:
“Would you like pesto or Bolognese?”
Simply, for its exclusive use, you cannot choose both disjuncts. Thus, choosing one excludes the other.
On the other hand, an example of it inclusive use is:
“Would you like sugar or cream in your coffee?”
For its inclusive sense, you can choose either or both of the disjuncts, thus, choosing one does not exclude the other
(Truth Functional Connectives n.d.).
We understand that in an inclusive disjunction, at least one of the disjuncts is true and in an exclusive
disjunction, at least one of the disjuncts is true but they cannot be both true (Copi and Cohen 2009, p. 321). In
logic, we use the inclusive sense or inclusive disjunction. An inclusive disjunction shall be false only when both of
the disjuncts are false. Thus, if we show it in a truth table:

B T B˅T

T T T
T F T
F T T
F F F

Though it is presumed that the inclusive sense is used in a disjunction, we can say that a disjunction is
exclusive if it is explicitly stated that the disjunction is intended to be exclusive such as adding the phrase “but not
both” (Copi and Cohen 2009, p. 322). For example, “Either you eat the cake or the chocolate, but not both”.

104 | P a g e
Like for conjunctions, when the two disjuncts have the same subject terms, we can simplify it by placing
the “or” between the predicate terms. On the other hand, when the disjuncts have the same predicate terms, the word
“or” may be placed between the subject terms (Copi and Cohen 2009, p. 323).
The word “unless” may also signify a disjunction as when we say “You will fail the exams unless you study
hard” (Copi and Cohen 2009, p. 323)

D. Punctuation
Punctuations are used to make statements clear and sensible (Education First n.d.). Without them, there is
a high possibility for sentences to be ambiguous. For example, the comma, which is perhaps the most abused and
misused punctuation (Grammarly 2018) may create a very big difference in a sentence depending on where it is
placed. The statement “The teacher says John is a fool” may be given different interpretations when a punctuations
are put in it: “The teacher” , says John, “is a fool”; or “The teacher says ‘John is a fool’” (Copi and Cohen 2009
p.323). Punctuations are essential in Mathematics. For example, in performing two or more operations in numbers
like 2 x 3 + 5, the answer may either be 11 or 16 – the first when punctuated as (2 x 3) + 5 and the second when
punctuated as 2 x (3 + 5) (Copi and Cohen 2009, p. 323).
In Symbolic language, parentheses, brackets, and braces are also used because ‘logic compound are
themselves often compounded together into more complicated ones’ (Copi and Cohen 2009, p.324). Thus p q ∨ r
might mean the conjunction of p with the disjunction of q with r or p (q ∨ r) or it might mean the disjunction
whose first disjunct is the conjunction of p and q and whose second disjunct is r or (p q) ∨ r. The difference of
these two can be seen by looking, for example, at p as false and q and r as both true. The second punctuated formula
will therefore be true because its second disjunct is true and the first punctuated formula will be false because its
first conjunct is false (Copi and Cohen 20oi, p. 324).
The word “either” may also be used in different ways in the English language. It may serve as a conjunction
as in the statement “There is danger in either sides” but it is often used to introduce a disjunction as in the statement
“Either the blind prisoner has a red hair or the blind prisoner has a white hat.” However, the most important use of
“either” is to punctuate a compound statement (Copi and Cohen 2009, p. 324). For example, the ambiguous sentence
“The organization will meet on Thursday and Anand will be elected or the election will be posted” may be cleared
by putting “either” in the beginning (“Either the organization will meet on Thursday and Anand will be elected or
the election will be posted”) or between “and” and “Anand” (“The organization will meet on Thursday and either
Anand will be elected or the election will be posted”) (Copi and Cohen 2009, p. 324).
In the case of a negation of a disjunction, the statement “Either Fillmore or Harding was the greatest U.S.
president” (F ∨ H) can be contradicted by the statement “Neither Fillmore nor Harding was the greatest U.S.
president” (~(F ∨ H) or (~F) (~H) (Copi and Cohen 2009, p. 324).
The word “both” shall also be regarded carefully. The statement “Both James and Nadine…” will have a
negation of “Neither James nor Nadine…” and the negation will be applied to each of them, thus its symbol would
be ~(J) ~(N). on the other hand, the negation “James and Nadine are not both…” means that we apply the negation

105 | P a g e
to the pair of them taken together thus, the symbol would be ~(J N). Also,to avoid further ambuguity and decrease
the use of parentheses, it was established as a convention that the negation symbol would only apply to the smallest
statement or where it is near rather that to the larger formula. Thus in the formula ~p V q, the negation would only
apply to p (Copi and Cohen 2009, p. 325).
Exclusive disjuctions, like conjunctions, negations and inclusive disjunctions, may also be written in
symbolic language and it may be written as p ∨ q ~(p • q) or by the exclusive disjunction “ ∨ “ (Copi and Cohen
2009, p. 325)
For truth-functional compound statements, we can easily determine the truth value of these statements by
simply looking at the truth value of their components. However, determining such truth value for more complex
compound statement, we must always start with the innermost components going outward. For example, we
calculate the truth value of the compound statement ~[~(A X) (Y ∨ ~B)] where A and B are true and X and Y are
false. The answer is: Because X is false, the conjunction A X is also false and so its negation ~(A X) is true. B is
true so its negation ~B is false, and because why is also false, the disjunction of Y with ~B, Y∨~B, is false. The
bracketed formula [~(A X) (Y ∨ ~B)] is a conjunction of true and false statements which is therefore false. Thus,
the negation of the whole statement would be true (Copi and Cohen 2009, p. 325-326).
There may be some circumstances where we may not know the truth value of one of the component
statements but still be able to know the truth value of the truth-functional compound. To do this, we just have to
calculate the truth value of a given compound statement assuming that one of its components is true, then by
calculating the compound statement assuming that the same component is false. If the truth value of the compound
statements is the same for both calculations, then we were able to know the truth value of the compound statement
even if we do not know the truth value of one of its components. This is so because we know that the truth value of
the components can only be either true or false, and nothing more. Truth tables help us in these cases and even in
cases when there are more than one undetermined truth value (Copi and Cohen 2009, p326)

Bibliography

Copi, Irving and Car Cohen. 2009. Introduction to Logic. 13th Ed. Jurong, Singapore:
Pearson Education, Inc.
Cothran, Martin. 2012. “The Difference Between Traditional and Modern Logic and the
Difference it Makes”. Vital Remnants. https://vereloqui.blogspot.com/2012/12/the-difference-between-
traditional-and.html (accessed April 8, 2018).
N/a. 2018. “Definition of Symbol”. Merriam-Webster Dictionary. https://www.merriam-
webster.com/dictionary/symbol (accessed April 8, 2018).
N/a. 2014. “Logical Connectives”. New World Encyclopedia.
http://www.newworldencyclopedia.org/entry/Logical_connectie(accessed April 8, 2018).

106 | P a g e
N/a. 2004. “Philosophy 103: Introduction to Logic The Language of Symbolic Logic”. Introduction to Logic.
philosophy.lander.edu. 2004. http://philosophy.lander.edu/logic/symbolic.html (accessed April 8, 2018).
N/a. n.d. “Punctuation”. Education First. https://www.ef.com/english-resources/english
grammar/punctuation/(accessed April 8, 2018).

N/a. 2018. “Rules for Comma Usage”. Grammarly Inc.


https://www.grammarly.com/blog/comma/(accessed April 8, 2018).
Slaney, John. 2014. “Negation”. The Logic Notes. The Australian National University.
http://users.cecs.anu.edu.au/~jks/LogicNotes/negation.html (accessed April 8, 2018).
Teachphilosophy. “Part 1: Symbolic Logic (The basics, letters, operators,
connectives”.YoutubeVideo, 14:02. Posted April 2013.
https://www.youtube.com/watch?v=1asxHpewYi8&feature=youtu.be
Truth Functional Connectives. (n.d.). [ebook] people.umass.edu., p.p. 32. Available at:
http://people.umass.edu/phil110h/text/reading03.pdf [Accessed 8 Apr. 2018].

CONDITIONAL STATEMENTS AND MATERIAL IMPLICATION

• Conditional Statements
Conditional statement is a compound statement where two statements are combined by placing the word
“if” before the first and inserting the word “then” between them. Also called a hypothetical, an implication, or an
implicative statement. For example, “If it rains, then the road will be wet.” The component statement that follows
“if” is called antecedent, and the component statement that follows “then” is called consequent. In the previous
example, “If it rains” is the antecedent and “then the road will be wet” is the consequent.
Conditional statement tells us that in any case in which the antecedent is true the consequent is also true. It
does not tell us that the antecedent is true, but only that if its antecedent is true, then its consequent is also true. It
does not tell us that the consequent is true, but only that its consequent is true its antecedent is true. The essential
meaning of a conditional statement is the relationship asserted to hold between the antecedent and the consequent,
in that order. This relationship is called “implication”.
There are four types of implication, namely: logical, definitional, causal, and decisional.
In logical implication, the consequent follows logically from its antecedent. In this conditional statement,
“If all humans are mortal and Timothy is a human, then Timothy is mortal.”, you can assert that its consequent
logically follows its antecedent.
In definitional implication, the consequent follows the antecedent by its very definition. In this conditional
statement, “If Robert is a bachelor, then Robert is unmarried.”, you can observe that the term bachelor in the
antecedent was merely defined in the consequent as unmarried.

107 | P a g e
In causal implication, the connection between antecedent and consequent is discovered empirically. In this
conditional statement, “If this blue litmus paper is placed in acid, then this piece of paper will turn red.”, you can
discover the connection of the antecedent and consequent is causal because it is capable of being verified or
disproved by observation or experiment.
In decisional implication, there is no logical connection, definition between the consequent and the
antecedent, nor is there any causal law involved. This is a decision of the speaker to behave in the specified way
under the specified circumstances. In this conditional statement, “If we lose the game, then I will run naked in the
street.”, you can assert that the decision of the speaker to run naked in the street rest entirely in the event that they
will lose the game.
These four conditional statements are different in that they assert a different type of implication between
its antecedent and its consequent. But they are not completely different; all assert types of implication. But is there
a common meaning among these? One way to find a common meaning is to check when the conditional statement
is false.
“If this blue litmus paper is placed in that acid solution, then this piece of blue litmus paper will turn red”
It is important to realize that this conditional does not assert that any blue litmus paper is actually placed in
the solution, or that any litmus paper actually turns red. It asserts merely that if this piece of blue litmus paper is
placed in the solution, then this piece of blue litmus paper will turn red. It is proved false if this piece of blue litmus
paper is actually placed in the solution and does not turn red. Under what circumstances should we agree that the
conditional statement is false? The acid test. The falsehood of a conditional statement is present when its antecedent
is true, because if its consequent is false while its antecedent is true, the conditional itself is thereby proved false.
“Conditional statement is false if its antecedent is true and consequent is false.”
Which sense of implication do we use? We must try to find a sense that is at least a part of the meaning of
all different types of implication. The “⁓ (p • ⁓ q)” is regarded the common meaning that is part of the meaning of
all four different types of implication symbolized as “If p, then q”. Every conditional statement means to deny that
its antecedent is true and its consequent false, but this need not be the whole of its meaning. No matter what type
of implication is asserted by a conditional statement, part of its meaning is the negation of the conjunction of its
antecedent with the negation of its consequent.
The common meaning of the “if-then” is symbolized as “⁓” called horseshoe. The symbol “⁓” is
completely unambiguous. “If p then q” is symbolized as “p ⁓ q” as an abbreviation for “⁓ (p • ⁓ q)”, whose
meaning is included in the meanings of each of the various kinds of implications considered but does not constitute
the entire meaning of any them.

• Material Implication
The horseshoe “⁓” also represents another kind of implication but it is not the same kind of implication as
any of those mentioned earlier, it is called material implication. Material implication constitutes a fifth type of
implication that may be asserted in ordinary discourse.

108 | P a g e
“No real connection between antecedent and consequent”, this kind of relationship is what is meant by
material implication. For example, “If Robert is a good-looking guy, then the Earth is flat.” For this reason, material
implications have often been thought to be an inadequate way of representing the normal uses of conditional
statements. Rather, it is often used as an emphatic or humorous method of denying its antecedent. The consequent
of such is usually a statement that is obviously false. What does material implication asserts? It asserts that “it is
not the case that the antecedent is true and the consequent is false”.
Material implication symbol is also a truth-functional connective, like the symbols for conjunction and
disjunction. The possible combinations of truth-values for implication is represented as follows:
Where p is true and q is true, p ⸧ q is true.
p q p⸧q
Where p is true and q is false, p ⸧ q is false.
T T T
Where p is false and q is true, p ⸧ q is true.
T F F
Where p is false and q is false, p ⸧ q is true.
F T T
The truth-table for implication is as follows:
F F T

As seen in the truth table, the symbol “⁓” has some features that may appear odd: The assertion that a false
antecedent materially implies a true consequent is true; and the assertion that a false antecedent materially implies
a false consequent is also true. So “If the moon is made of green cheese, then the Earth is round” is true (it has a
false antecedent and true consequent) and “If the moon is made of cheese, then the Earth is flat” is also true (it has
a false antecedent and false consequent).
For further explanation, as illustrated by Copi and Cohen in their book “Introduction to Logic”,
the number 2 being smaller than the number 4 “(2 < 4)”, it follows that any number smaller than 2 is
smaller than 4. The conditional formula “If x < 2, then x < 4” is true for any number whatsoever. If we
focus on the numbers 1, 3, and 4, and replace the number variable x in the preceding conditional formula
by each of them in turn, we can make the following observations. In “If 1 < 2, then 1 < 4” both antecedent
and consequent are true, and of course the conditional is true. In “If 3 < 2, then 3 < 4”, the antecedent is
false and the consequent is true, and of course the conditional is again true. In “If 4 < 2, then 4 < 4 both
the antecedent and the consequent is false, but the conditional remains true. The last two cases correspond
to the third and fourth rows of the truth-table of implication. There is no number that is smaller than 2 but
not smaller than 4; that is why there is no conditional statement with a true antecedent and false consequent.
This is exactly what the truth-table for implication lays down.
This paradox is not a genuine paradox, but merely puts some pressure on the identification of the conditional
form as it is found in natural languages, with material implication.
All types of implication can also be treated as material implication. They can be translated into symbolism
(translate “if-then” phrase into a logical symbol). This is because most conditional statements assert more than that
a merely material implication holds between their antecedent and consequent. Like in the disjunctive syllogism, it
is justified on the ground that many arguments contain conditional statements of various kinds, but the validity of

109 | P a g e
all valid arguments of the general type is preserved even if the additional meanings of their conditional statements
are ignored.

• Some “If” and “then” Indicator Words


“If…” can be replaced by such phrases as:
 “in case…”
 “provided that…”
 “given that…”
 “on condition that…”
Some indicator words for “then…” includes:
 “implies…”
 “entails…”
Conditional statements can be reformulated in different ways. The statement, “If he has a good lawyer, then
he will be acquitted”, can equally well be stated without the use of “then” as “If he has a good lawyer, he will be
acquitted.
The order of the antecedent and consequent can be reversed, provided that the “if” still directly precedes
the antecedent. “He will be acquitted if he has a good lawyer.”, in this example the “if” still precedes the antecedent
“he has a good lawyer”.
A shift from active to passive voice may accompany a reversal of the antecedent and consequent provided
that it has logically equivalent meaning. For example, “His being acquitted is implied by his having a good lawyer.”

• The Concept of Necessary and Sufficient Conditions


Necessary condition is defined as “a condition A is said to be necessary for a condition B, if (and only if)
the falsity (/nonexistence/non-occurrence as the case may be) of A guarantees (or brings about) the falsity
(/nonexistence/non-occurrence) of B. In an “If-then” statement, the clause that follows the “then” states the
necessary condition for the antecedent. In general, “q is a necessary condition for p” is symbolized as p ⁓ q.
For example, we all know that air is necessary for human life, without air, there is no human life.
Thus, “If a human being is alive, then that human being has air.” simply states that human being has air
(to breathe) is a necessary condition for that human being’s being alive
Sufficient condition is defined as “a condition A is said to be sufficient for a condition B, if (and only if)
the truth (/existence/occurrence as the case may be) of A guarantees (or brings about) the truth
/existence/occurrence) of B. In general, “p is a sufficient condition for q” is also symbolized as p ⁓ q.
For example, while air is a necessary condition for human life, it is by no means a sufficient
condition, it does not, by itself alone suffice for human life. While someone may have air to breathe, that
person will still die if he lacks water (for a number of days), has taken poison, is exposed to extreme cold
or heat, etc. There are, in fact, a very great many conditions that are necessary for human life, and no one

110 | P a g e
– or even just few of them – will suffice for human life. But if all of human needs in order to live are present,
then the specified situation will be realized.
If p is a sufficient condition for q, we have p ⁓ q, and q must be a necessary condition p. If p is a necessary
condition for q, we have q ⁓ p, and q must be a sufficient condition for p. Hence, if p is a necessary and sufficient
for q, then q is a sufficient and necessary for p.

• “If” statements that are Not Conditionals


Not every statement containing the word “if” is a conditional. The following statements are not
conditionals:
There is a food in the refrigerator if you want some.
Your table is ready, if you please.
There is a message for you if you’re interested
The meeting will be held even if no permit is obtained.
The presence or absence of particular words is never decisive. In every case, one must understand what is
given sentence means, and then restate that meaning in a symbolic formula.

Bibliography

I. Copi, C. Cohen, and K. MacMahon, 2014. Introduction to Logic. 14th Edition. Edinburg Gate, Harlow, Essex
CM20 2JE, England: Pearson Education Limited
Dr. V. Prabhakaran, 2011. Essentials of Symbolic Logic. School of Distance Education, University of Calicut, India
N/a, 2004. Philosophy 103: Introduction to Logic Conditional Statements and Material Implication. Introduction
to Logic. Philosophy.lander.edu. 2004. http://philosophy.lander.edu/logic/conditional.htl last visited April
19, 2018
M. Clark, 1996. A Comparison Techniques for Introducing Material Implication. University of Natal,
Pietermaritzburg, South Africa. http://www.cs.cornell.edu/Info/People/gries/symposium/clarke.htm last
visited April 19, 2018

P. Suber, 1997. Paradoxes of Material Implication. Philosophy Department, Earlham College, Indiana, U.S.A.
https://legacy.earlham.edu/~peters/courses/log/mat-imp.htm last visited April 19, 2018
D. Brierton, Conditionals, necessary and sufficient conditions. University of Missouri – Kansas City.
http://cas2.umkc.edu/philosophy/vade-mecum/2-4.htm last visited April 19, 2018
S. Norman, 1997. The Concepts of Necessary Conditions and Sufficient Conditions. Department of Philisophy,
Simon Fraser Univeristy, British Columbia, Canada

111 | P a g e
ARGUMENT FORMS AND REFUTATION BY LOGICAL ANALOGY

The central task in refuting logical analogy is to determine between valid arguments from invalid
arguments.
Rules:
1. if the premises of a valid argument are true its conclusion must be true
2. if the conclusion of a valid argument is false, at least one of the premises must be false
If an argument is invalid, then any argument with the same logical form is invalid, thus we represent the formula:

If P then Q
Modus ponens
P

Therefore Q
Example: This is invalid

If Washington was assassinated, then Washington is dead

Washington is dead
----------------------------

Therefore, Washington was assassinated (invalid)

- In this given example we may agree with the premises but disagree with the conclusion. Judging by the
form of the argument is already invalid.
Reason: the argument is of the same form
 An argument is valid if it has a form that can never lead one to affirm a false conclusion on the basis of
true premises.
 If the argument has a form that can lead one from true premises to a false conclusion, then the argument is
invalid.
Other examples:
1. If Leonardo da vinci painted the Sistine chapel, then Leonardo da vinci was a great painter.
Leonardo Da Vinci was a great painter.

Therefore, Leonardo Da vinci painted the Sistine chapel. (invalid)

2. If we can construct a warp engine, then we can travel faster than light.

112 | P a g e
We can travel faster than light
Therefore we can construct a warp engine (invalid )

Proving the invalidity of an argument


1. Has exactly the same form as the first
2. Has true premises and a false conclusion

ARGUMENT FORMS
An array of symbols containing statement variables but no statements resulting to an argument
Statement variables are letters for which a statement may be substituted
1. In example number 1;
B G
If Leonardo da vinci painted the Mona Lisa, then Leonardo da vinci was a great painter.
G
Leonardo Da Vinci was a great painter.
G
Therefore, Leonardo Da vinci painted the Mona Lisa.

B G

... G
SUBSTITUTION INSTANCE
For definiteness:

P Q

... P
This is called substitution instance in which any argument that results from
the substitution of statements for statement variables in an argument form. It is clear that any substitution instance
of an argument form may be said to have this form.
In the process of substitution instance we have what we call a specific form of that argument
Definition: if an argument is produced by substituting “CONSISTENTLY” a different simple statement for each
statement variable in that argument form that argument form is the specific form of that argument

113 | P a g e
For instance in the previous example the argument form may be substituted with the following variables
thus:

B G
Figure 1
G

... G - In figure 1 it can be subject to a substitution instance of each of


the four argument forms thus: it can be either figure 2, figure 3, figure 4,
or figure 5

P Q P Q P Q P

Q R R Q

... P ... P ... S ... R Figure 2


Figure 3
Figure 4 Figure 5

 Of these four argument forms, the first corresponds more closely to the structure of the given argument than
do the others

Group A- arguments Group B- argument forms

1. A B
Specific form
.
.. A p q
.
..p

2. G H Specific form
~H p q
... ~G ~q
... ~q

114 | P a g e
3. [ G (G H)] [H (H G)]
Substitution instance:
... G (G H)
p q

. .. p

Specific form:

[p (p q)] [q (q p)]
4. W X) (Y Z) Substitution instance:
.
... p (p q)
. . ( W X) [(W X) (Y Z)]
p q

. .. p (p q)

Specific form:

[p (p q)] [q (q p)]
8.5 PRECISE MEANING OF VALID AND INVALID
. .. p (p q)
What precisely is meant by saying that an argument form is
invalid or valid?

Refutation by Logical Analogy


An argument form is invalid if and only if it has at least one substitution instance with true premises and a false
conclusion.
- Any argument whose specific form is an invalid argument is an invalid argument

An argument form is valid if and only if it has no substitution instance with true premises and a false conclusion.
- Reason: Any argument is valid provided that its specific form of that argument has a valid form of
argument

Example: ( valid)
A B
Specific form
If it is raining outside then the floor is wet p q
A P
It is raining outside . .. q
B
Therefore the floor is wet

115 | P a g e
(invalid)
A B
If it is raining outside then the floor is wet
Specific form
A p q
It is raining outside P
~B . .. ~ q
Therefore the floor is not wet

TESTING ARGUMENT VALIDITY USING TRUTH TABLES

TRUTH TABLE
- A diagram in rows and columns showing how the truth or falsity of a proposition varies with that of its
component. (Merriam Webster Dictionary)
- The method of using a truth table is a very simple and powerful tool to test the validity of every truthful-
functional argument. (Copi & Cohen)

Statement variables p q p·q pvq p⊃q


T T T T T
GUIDE T F F T F
COLUMNS
F T F T T
F F F F T

HOW TO USE THE TRUTH TABLE:


1. Construct the truth table correctly.
a. There must be a guide column for each statement variable (p,q,r, and so on)
b. The array must exhibit the possible combinations of truth and falsity of all these variables.
c. There must be a number of horizontal rows sufficient to do this: four rows if there are two variables,
eight rows if there are three variables, and so on.
d. There must also be an additional vertical column for each of the premises and the conclusion.
e. There must also be a column for each of the symbolic expressions wherein the premises and conclusions
are built.
2. Be careful in placing the T’s and F’s in the appropriate columns, governed by our understanding of the
truth-functional connectives: the dot, the wedge, and the horseshoe – and the circumstances under which
each truth-functional compound is true and the circumstances under which is false.
3. Examine all possible substitution instances of it to see if any one of them has true premises and a false
conclusion.

116 | P a g e
4. If there is a row which has true premises and a false conclusion, the argument form is invalid. If there is
none, it is valid.

Decide on the validity of these argument forms:

p q p⊃q p q p⊃q
1. p⊃q
T T T T T T
q
T F F T F F
-----------
F T T PREMISES F T T
p
F F T F F T
INVALID

CONCLUSION

 If Bacon wrote the plays attributed to Shakespeare, then Bacon was a great writer. p ⊃ q
 Bacon was a great writer. q
 Therefore Bacon wrote the plays attributed to Shakespeare. p

Steps:
a. Substitute the given statements.
b. Fill in the two guide columns.
c. Fill in the third column by referring back to the guide columns. The third column heading is the first
“premise” of the argument form; the second column is the second “premise”, and the first column is
the “conclusion”.
d. Read backwards.
e. Check each row. If there is a row where there are T’s under both premises and an F under the
conclusion, which indicates that there is at least one substitution instance of this argument form that
has true premises and a false conclusion, then this shows that this argument form is invalid.

2. p v q p q pvq ~p
~p
------- T T T F
q T F T F
F T T T
VALID F F F T

117 | P a g e
 Either you will see Joe in class today or he will oversleep. p v q
 You did not see Joe in class today. ~p
 Therefore, Joe overslept. q

3. (p v q) ⊃ (p · q) p q pvq p·q (p v q) ⊃ (p · q)
pvq
----------
T T T T T
p·q
T F T F F
F T T F F
VALID
F F F F T

 If either the Philippines or Bangladesh accepts industrialization, then Philippines and Chile would become
richer. (p v q) ⊃ (p · q)
 Either the Philippines or Bangladesh accepts industrialization. p v q
 Therefore, Philippines and Chile would become richer. p · q

SOME COMMON ARGUMENT FORMS

A. Common Valid Forms

1. Disjunctive Syllogism
One of the simplest arguments forms relies on the fact that in every true disjunction, at least one of the
disjuncts, must be true. Basically, this argument gives us two options and says that, since one option is
FALSE, the other option must be TRUE.

This is symbolized as:

pvq

118 | P a g e
~p p q pvq ~p
-------
T T T F
q
T F T F
F T T T
VALID
F F F T

 The coin is either in my right hand or my left hand. p v q


 It’s not in my right hand. ~p
 So, it must be in my left hand. q

2. Modus Ponens
The simplest type of intuitively valid argument involving a conditional statement. This is also known as the
“method of affirming the antecedent”. Basically, the argument states that, given a first thing, a second thing
is true. It then AFFIRMS that the first thing is true. So, the second thing must also be true.

This is symbolized as: p q p⊃q

T T T
p⊃q
T F F
p
F T T
-----------
F F T
q

VALID

 If you have a driver’s license, then you must have taken the driver’s test. p ⊃ q
 You do have a driver’s license. p
 So, you must have taken the driver’s test. q

3. Modus Tollens
If a conditional statement is true, then if the consequent is false, the antecedent must also be false. The
argument form that relies on this is very commonly used to establish the falsehood of some proposition
under attack. This is also known as the “method of denying the consequent”.

119 | P a g e
This is symbolized as:
p q p⊃q ~q ~p

p⊃q T T T F F
~q T F F T F
----------- F T T F T
~p F F T T T

VALID

 If you have a driver’s license, then you must have taken the driver’s test. p ⊃ q
 You have not taken the driver’s test. ~q
 So, you must not have a driver’s license. ~p

4. Hypothetical Syllogism
Basically, the argument states a chain of reasons, where the first thing is connected to a second, and the
second to a third, so the first is ultimately also connected to the third. A larger truth-table is required to
demonstrate the validity of this form since it involves three distinct statement variables instead of two, and
we must consider all eight of the possible combinations of their truth-values.

This is symbolized as:

p q r p⊃q q⊃r p⊃r


p⊃q
T T T T T T
q⊃r
T T F T F F
----------
T F T F T T
p⊃r
T F F F T F
F T T T T T
VALID
F T F T F T
F F T T T T
F F F T T T

 If you hit the ball in on this turn, you’ll get a hole in one. p ⊃ q

120 | P a g e
 If you get a hole in one, you’ll win the game. q ⊃ r
 Therefore, if you hit the ball in on this turn, you’ll win the game. p ⊃ r

B. Common Invalid Forms

1. Fallacy of Affirming the Consequent


Although the shape of this form is something like that of modus ponens, this form is invalid. Basically, the
argument states that, given a first thing, a second thing is true. It then AFFIRMS that the second thing is
true, and concludes from this that the first thing must also be true.

This is symbolized as:


p q p⊃q
T T T
p⊃q
T F F
q
F T T
--------
F F T
p

INVALID

 If you were standing out in the rain, then you would be wet now. p ⊃ q
 You are wet now. q
 Therefore, you must have been standing out in the rain. p
This sort of inference is mistaken. For instance, just because you are wet does not guarantee that you were
just standing out in the rain. Perhaps you just jumped into a pool, or into the shower. Or perhaps someone
just poured a bucket of water on you. We cannot conclude for sure from the fact that you are soaking wet
that you were outside in the rain.

2. Fallacy of Denying the Antecedent


This is also an invalid argument whose shape is somewhat like that of modus tollens. Basically, the
argument states that, given a first thing, a second thing is true. It then DENIES that the first thing is true,
and then concludes from this that the second thing must also NOT be true.

This is symbolized as:

p⊃q

121 | P a g e
~p
---------
~q

INVALID

 If you were standing out in the rain, then you would be wet now. p ⊃ q
 You were not standing out in the rain. ~p
 So, you must not be wet now. ~q

p q p⊃q ~q ~p

T T T F F
T F F T F
F T T F T
F F T T T

This sort of inference is mistaken. For instance, just because you were not just standing in the rain does not
guarantee that you are not soaking wet right now. Again, perhaps you just jumped into a pool, or into the
shower. Or perhaps someone just poured a bucket of water on you. We cannot conclude for sure from the
fact that you were not just outside in the rain that you are not soaking wet now.

C. Substitution Instances and Specific Forms

Symbol Substitution instance of the valid Substitution instance of the invalid


argument form argument form

RvW pvq p
~R ~p q
------- -------- -------
W q r

*SPECIFIC FORM

122 | P a g e
 In determining whether any given argument is valid, look to the specific form of the argument in question
because only the specific form will reveal the full logical structure of that argument. If the specific form of
an argument is valid, the argument itself must be valid, as shown in second column above.

 If an argument is not in the specific form, it cannot be used to show that the given argument is invalid, as
shown in the third column above.

 An argument form that is valid can have only valid arguments as substitution instances. All of the
substitution instances of a valid form must be valid, as shown in the truth table. (Copi & Cohen)

Statement Forms and Material Equivalence

A. Statement Forms and Statements

In exactly the same sense that arguments may be substitution instances of general argument forms,
individual compound sentence can be substitution instances of general statement forms. Likewise, truth-tables were
employed to test the validity of statement forms or to exhibit interesting logical features of some statement forms.
Statement form is defines as a “sequence of symbols containing no statements, but containing statement
variables connected in such a way that when statements are consistently substituted for the statement variables, the
result is a statement” (Copi and Cohen, 2014). For instance, “p V q” is a statement form, because statement will
result if the variables p and q will be substituted by statements. Additionally, a statement form is different from
specific form.
To distinguish the specific form of a given statement, Copi and Cohen (2014) states that, “we distinguish
the specific form of a given statement as that statement form from which the statement results by substituting
consistently a different simple statement for each different statement variable”. Meaning the statement form will
become the specific form of a given statement when the variables are already substituted by a simple statement. For
example, p v q is the specific form of the statement “the blind prisoner has a red hat or the blind prisoner has a
white hat”. This is true only when said statement is already the substitution instances of the said general specific
form.
In propositional logic, statement forms have six primary classes namely: 1) simple, 2) conjunctive, 3)
disjunctive, 4) conditional, 5) biconditional, and 6) negation.
Simple statement form expresses one idea. “Lincoln was assassinated” symbolized as L is an example of this
class of statement form.
Conjunctive statement form is a compound statement made up of two statements connected with the word “and”.
The two statements must be both true to infer its validity because if one of the statement is false, then the whole
compound statement is false. The statement form “p . q” is an example of this class.

123 | P a g e
Disjunctive statement form is a compound statement wherein only one of the statements or disjuncts is true. It
is connected with words “either…or” or “or”. An example of this statement form is p v q. A statement form
symbolised as p v q, its substitution instances are disjunctive statements.
Conditional statement form is a compound statement wherein the first statement is called antecedent and the
second statement is the consequences. For example, if it rains then the ground is wet.
Biconditional statement form is a compound statement where each component implies the other. “Myrna will
get mood if and only if Bob will sing” is example of this statement.
Negation statement form is a statement denying that another statement is true. It symbolised as ~p.

B. Tautologous, Contradictory, and Contingent Statement Forms

A statement like argument can also be determined its truth and falsehood. Consider, for example, the
statement, “Lincoln was assassinated” (symbolized as L), and the statement, “Either Lincoln was assassinated or
else he wasn’t (symbolized as L v ~L), are both obviously true. The first statement is historical facts, while the
second statement is a logical truth. These kinds of truth are important in ascertaining the validity of a statement.
Likewise, falsehood has also these two kinds.
There are three kinds of statement forms as to its truth or falsity, namely: 1) tautology, 2) contradiction, and
3) contingent. The substitution instances of statement form can be only true, only false, or either. That’s why
statement forms have three kinds.
1) Tautologous statement form is a statement form that has only true substitution instances (Copi and Cohen,
2004), it is also called tautology. Meaning, this statement form contains only T in its truth table. Consider,
for example, the statement form: p v ~p

p ~p p v ~p
T F T
F T T

The truth-table shows that whether the component statement “p” is true or false makes no difference to the
truth value of the statement form; it yields a true statement in either case. But it entails that any compound statement
which is a substitution instance of this form, no matter what its content, can only make true assertions. Copi and
Cohen (2004) stated that “any statement that is a substitution instance of a tautologous statement form is true in
virtue of its form, and is itself said to be tautologous, or a tautology”.

2) Self-contradictory or contradiction statement form is a statement form that has only false substitution
instances (Copi and Cohen, 2014). This is the complete opposite of tautology. Thus, this statement form
whose column in a truth-table contains only F. Consider, for example, the statement form: p . ~p

124 | P a g e
p ~p p . ~p
T F F
F T F

The truth-table shows that whether the component statement “p” is true or false makes no difference because
in either case it signify that all its substitution instances are false. Compound statements that are substitution
instances of this form can never be used to make true assertions. Thus, it is logically false because this statement
form in virtue of its form.

3) Contingent statement form is being neither tautologous nor self-contradictory. A contingent statement
may be true or false; a contingent statement form has some true and some false substitution instances (Copi
and Cohen, 2014). This statement whose column in a truth table contains either T or F. Consider, for
example, the statement form: p . q

p q p.q
T T T
T F F
F T F
F F F

The truth-table shows that the truth value of this compound statement form contains true and false. In this
statement form, the value of component p and q whether it is false or true makes a difference in the truth value of
the compound statement form. Thus, statements that are substitution-instances of this statement form may be either
true or false, depending upon the truth value of their component statements.

C. Material Equivalence

Material equivalence is one of the four truth-functional connectives which deductive argument commonly
depends. Likewise, the truth value of these truth-functional connectives depends on the truth or falsity of the
statements it connects. Material equivalence is the truth functional connective that asserts that the statements it
connects have the same truth value (Copi and Cohen, 2014). If the two statements are equivalent in truth value are
materially equivalent. Meaning, two statements are materially equivalent when they are both true, or both false.
Just like the other truth-functional connectives have symbol, the symbol for material equivalence is the
three-bar sign “≡” or tribar. The three-bar sign is read as, to say “if and only if”. This connective has also truth-
table, hence:

125 | P a g e
p q p≡q
T T T
T F F
F T F
F F T

The truth-table shows that the truth value of this connective depends upon the truth or falsity of its
components, and the only time it is true is when the two statements (p or q) are either both true, or both false as
shown in the first and last row.
This truth-functional connective infers that any two statements materially imply one another. And any two
false statements also materially imply one another. That’s why if one statement is true and yet the second statement
is false, then the two statements are not materially equivalent. Therefore, any two statements that are materially
equivalent must imply one another, because they are both true or both false. (Copi and Cohen, 2004)
For example A and B are materially equivalent. It may infer that B is true if A is true and also that B is true
only if A is true or B is false if A is false. Copi and Cohen (2004) states that, “every implication is a conditional
statement. Two statements, A and B, that are materially equivalent entail the truth of the conditional A ⊃ B, and
also entail the truth of the conditional B ⊃ A. Because the implication goes both ways when material equivalence
holds, a statement of the form A ≡ B is often called a biconditonal (p360).

LOGICAL EQUIVALENCE

In this chapter Copi and Cohen introduces a new relation, important and very useful, but not a connective, and
somewhat more complicated than any of the truth-functional
connectives just discussed.
MATERIAL EQUIVALENCE

 Both statement have the same truth value


 Either both true, or both false
 They must (materially) imply one another
 A false antecedent (if) materially implies any statement
 A true consequent (then) is (materially) implied by any statement
 Symbol: Three-bar sign (≡), “if and only if”

Example # 1:
126 | P a g e
a. Sheila will buy one if and only if Rollyn will.
b. If Sheila buys one Rollyn will and if Rollyn buys one, Sheila will.

Example # 2:
a. Sheila will try the food if Rollyn does and Rollyn will try the food if Sheila does.
b. Therefore, Sheila will try the food if and only if Rollyn does.

Note: Examples just to show the “if and only if”

However, statements that are materially equivalent most certainly…


 Cannot be substituted for one another (we only know that their truth values are the same)

Examples # 1:
a. “Jupiter is larger than the Earth”
b. “Tokyo is the capital of Japan”
* Both are materially equivalent because they are both true, but we obviously cannot replace one with the other.

Examples # 2:
a. “All spiders are poisonous”
b. “No spiders are poisonous”
* Both are materially equivalent because they are both false, and they certainly cannot replace on another.

p ≡ q
T T T
T F F
F F T
F T F

LOGICAL EQUIVALENCE
Definition: Two statements are logically equivalent if the statement of their material equivalence is a tautology.

Tautology
Definition: A statement form all whose substitution must be true. Thus, in contrast with Material
Equivalence, this relation permits substitution.

127 | P a g e
* A tautology is a logical statement in which the conclusion is equivalent to the premise.
* A formula or assertion that is true in every possible interpretation.
* A tautology is a formula which is "always true"

p ≡ (p • q) > p “and” q is true/false whenever p is true/false


p ≡ (p v q) > p “or” q is true/false whenever p is true/false

 Two statements can be equivalent in a sense much stronger than that of material equivalence; they
may be equivalent in meaning as well as having the same truth value.
 Any proposition that incorporates one of them, can just as well incorporate the other
 Permits mutual replacement
 No case where one statement is true, while the other is false
 Statements that are logically equivalent are materially equivalent as well (same truth value)
 Symbol: Three-bar with a small “T”, immediately above it ( )
o Expresses a tautological biconditional

To make this the relation of Logical Equivalence clearer, we go into the principle of Double Negation as well as
to give an example.

Simple Logical Equivalence


Double Negation
P and ~~p means the same thing
- Double negation switches its truth value back
- Is the same as the original one

Example:
a. “He is aware of that difficulty” (p)
b. “He is not unaware of that difficulty” (~~p)
* Two statements with the same content.
* Either of these expressions may replace the other because they both say the same thing.

Symbolized as p ~~p

p ~p ~~p p ~~p
T F T T

128 | P a g e
F T F T

Difference: between material equivalence and logical equivalence

Material Equivalence Logical Equivalence


 Truth-functional connective  Not a mere connective (Truth-functional
Any logical connective (e.g. conjunction, component)
disjunction, material implication and material Any component of a compound statement
equivalence) between a compound statement whose replacement there by any other
whose truth value is determined wholly by the statement having the same truth value would
truth value of its components. have the truth value of the compound statement
o May be true or false unchanged.
o Depends on the truth/falsity of elements o Expresses relationship between two
 Truth-functional connective statements that is not truth-functional
 Same truth value, no factual connections  Absolutely impossible for them to have
 May not substitute different truth values
 Must have the same meaning (may substitute)

Then we go to the two tautologous biconditionals (or logical equivalence) of great importance because they
express the interrelations among conjunction and disjunction, and their negations, known as…

DE MORGAN’S THEOREMS:
Tautologous Biconditionals

First, what will serve to deny that a disjunct is true?


Disjunction

To deny that a disjunct is true (Affirm by denying)

* Any disjunction p v q asserts no more than at least one of its two disjuncts is true.
* You cannot contradict by saying that at least one is false.
* Must assert (to deny) that both disjuncts are false.

129 | P a g e
* Therefore, the negation of the disjunction (p v q) is logically equivalent to asserting the conjunction of the
negation of p and q.
Biconditional: ~(p v q) ≡ (~p • ~q)

Can be formulated as:


a. The negation of the disjunction of two statements is logically equivalent to the conjunction of the negation of
the two statements.

~(p v q) (~p • ~q)


Both disjuncts are false

Example:
a. Neither Peter nor Quentin are married. - negation of the disjunction of two statements
means the same as, or is logically equivalent to
Peter is not married, and Quentin is not married. – conjunction of the negation of the two statements

Similarly
Conjunction

* Conjunction p v q asserts that both are true and assert that at least one is false.
* Thus, asserting the negation of the conjunction, (p • q), is logically equivalent to asserting the disjunction of the
negations of p and of q.
Biconditional: ~(p • q) (~p v ~q)

Can be formulated as:


b. The negation of the conjunction of two statements is logically equivalent to the disjunction of the negation of
the two statements.

~(p • q) (~p v ~q)

Example:
a. Peter and Quentin are not both married. – negation of the conjunction of two statements
means the same as, or is logically equivalent to
Peter is not married, or Quentin is not married. – disjunction of the negation of the two statements

130 | P a g e
p q pvq ~(p v q) ~p ~q ~p • ~q ~(p v q) ≡ (~p • ~q)
T T T F F F F T
T F T F F T F T
F T T F T F F T
F F F T T T T T

 Whatever the truth values of p an q, this biconditional must always be true. (Tautology)
 Conlusion: ~(p v q) (~p • ~q)

MATERIAL IMPLICATION

 Symbol: ⊃
 Helpful when we seek to manipulate truth-functional connectives
 Defined in 8.3 as an abbreviated way of saying ~(p • ~q). That is, “p materially implies q” simply
means, by definition, that it is not the case that p is true while q is false.
o The definiens, ~(p • ~q), is the denial of conjunction.
o De Morgan’s theorem: ~(p • ~q) is logically equivalent to (~p ~~q)
o And in turn, applying the principle of double negation, is logically equivalent to ~p v q
o Therefore, the original definiens of the horseshoe, ~(p • ~q), may be replaced with no change
of meaning by the simpler expression ~p v q
o Thus, p ⊃ q is logically equivalent to ~p v q
(p ⊃ q) (~p v q)

 Is a valid rule of replacement that allows for a conditional statement to be replaced by a disjunction in
which the antecedent is negated. The rule states that P implies Q is logically equivalent to not-P or Q
and can replace each other in logical proofs.

Example:
a. “If it is a bear, then it can swim.”
b. “Thus, it is not a bear or it can swim.”
*Where p is the statement “it is a bear” and q is the statement “it can swim”.

THE THREE “LAWS OF THOUGHT”

131 | P a g e
Basic laws of thoughts:
 The principle of identity. This principle asserts that if any statement is true, then it is true. Using our
notation, we may rephrase it by saying that the principle of identity asserts that every statement of the form p
⊃ p must be true, that every such statement is a tautology.
The principle of identity has been attacked on the ground that things change,
and are always changing.

Example:
a. “There are only thirteen states in the United States” – true in 1790, fifty states today
…such is incomplete, an elliptical formulation of the statement that…
b. “There were only thirteen states in the United States in 1790” – true today as it was in 1970

When we confine our attention to complete, nonelliptical formulations of propositions, we see that their
truth (or falsity) does not change over time. The principle of identity is true, and it does not interfere with
our recognition of continuing change.

 The principle of noncontradiction. This principle asserts that no statement can be both true and false. Using
our notation, we may rephrase it by saying that the principle of noncontradiction asserts that every statement
of the form p • ~p must be false, that every such statement is self-contradictory.

The principle of noncontradiction has been attacked by Hegelians and Marxists


on the grounds that genuine contradiction is everywhere pervasive, that the world
is replete with the inevitable conflict of contradictory forces.

Example:
a. “It is raining outside”
b. “It is not raining outside”

The principle of noncontradiction, understood in the straightforward sense in


which it is intended by logicians, is unobjectionable and perfectly true.

 The principle of excluded middle. This principle asserts that every statement is either true or false. Using
our notation, we may rephrase it by saying that the principle of excluded middle asserts that every statement
of the form p v ~p must be true, that every such statement is a tautology.

The principle of excluded middle has been the object of much criticism, because

132 | P a g e
it leads to a “two-valued orientation,” which implies that things in the
world must be either “white or black,” and which thereby hinders the realization
of compromise and less-than-absolute gradations.

Example:
a. “This is black”
b. “This is white”
The first statement cannot be jointly true be with the second statement where “this” refers to the same
thing.

The contradictory of the statement “This is white” is the statement “It is not the case that this is white” and
(if “white” is used in precisely the same sense in both of these statements) one of them must be true and
the other false. The principle of excluded middle is inescapable.

o Status as the most fundamental laws of thought is doubtful.


o The first (identity) and the third (excluded middle) are tautologies, but there are many other
tautologous forms whose truth is equally certain. The second (noncontradiction) is by no means the
only self-contradictory form of statement.
o We do use these principles in completing truth tables. Nevertheless, in regarding the entire system of
deductive logic, these three principles are no more important or fruitful than many others.
o All three of these “laws of thought” are unobjectionable—so long as they are applied to statements
containing unambiguous, nonelliptical, and precise terms.
o Of the principle of noncontradiction, Aristotle wrote: “That the same attribute cannot at the same time
belong and not belong to the same subject and in the same respect” is a principle “which everyone must
have who understands anything that is,” and which “everyone must already have when he comes to a
special study.” It is, he concluded, “the most certain of all principles.”
o The “laws of thought” may not deserve the honorific status assigned to them by some philosophers, but
they are indubitably true.

References:
Copi, I.M., & Cohen, C. (2009). Introduction to Logic 13th Edition. Jurong, Singapore: Pearson Education Inc.
<http://www.philosophypages.com/lg/e10b.htm. > Accessed 04 April 2018.
<http://rintintin.colorado.edu/~vancecd/phil1440/forms.pdf> Accessed 04 April 2018.
Copi, I. M., & Cohen, C. (2004). Introduction to logic. Upper Saddle River, N.J: Pearson/
Prentice Hall.:

133 | P a g e
Copi, I. M., & Cohen, C. (2014). Introduction to logic, 14th ed. Upper Saddle River, N.J: Pearson/
Prentice Hall.
Copi, Irving and Car Cohen. 2009. Introduction to Logic. 13th Ed. Jurong, Singapore: Pearson
Education, Inc.
http://selfpace.uconn.edu/class/phil102/u13.pdf

134 | P a g e
CHAPTER 9: METHODS OF DEDUCTION

9.1 Formal Proof of Validity

In general truth tables are adequate to test the validity of arguments, however as the number of component
statement increases the use of truth tables become more inefficient. A method which is highly efficient and is now
used to test the validity of an extended argument. According to Copi and Cohen the new method is to deduce its
conclusion from its premises by the sequence of elementary arguments which is known to be valid.

We consider the following argument as an example:


If Anderson was nominated, then she went to Boston.
If she went to Boston, then she campaigned there.
If she campaigned there, she met Douglas.
Anderson did not meet Douglas
Either Anderson was nominated or someone more eligible was selected
Therefore someone more eligible was selected.

Although the validity of the argument may be obvious we have to consider the matter of proof.
In order to completely understand we have to translate them in their symbolic form.

Let:
 A be Anderson
 B be Boston
 C be Campaigned
 D be Douglas
 E be Eligible

1. A ⊃ B
2. B ⊃ C
3. C ⊃ D
4. ~D
5. A ˅ E
E

135 | P a g e
The validity of this argument is established by means of a truth table that requires thirty-two rows because
the five different statements are involved. We can prove the argument valid by deducing its conclusion using a
sequence of just four valid arguments. In the first two premises we can infer that A ⊃ B and B ⊃ C is the same as
you A ⊃ C. In using a hypothetical syllogism. From A ⊃ C and the third premise which is C ⊃ D we can validly
state that A ⊃ D as another hypothetical syllogism. Then from A ⊃ D and the fourth premise ~D, we can say that
the statement is ~A by Modus Tollens. Now from ~A and the fifth premise which is A ˅ E as a Disjunctive Syllogism
so we validly infer E which is now the conclusion of the original argument.
The conclusion was deduced from the five original arguments by using the four elementary valid argument.
This now proves that the original argument is valid. In the given example above, we used (1) Hypothetical
Syllogisms; (2) Modus Tollens; and (3) Disjuntive Syllogisms. We call them as rules of inference wherein their
application allows conclusions to be validly inferred or deduced from premises.
Rules of inference is defined as the rules that permit valid inferences from statements assumed as premises
(Copi and Cohen, 2014). The method that we applied in the given example is known as a deductive argument. We
use the rules of inference successively to prove the validity of the argument. This is a more efficient way of proving
validity and it is as reliable as the truth-table method. It is considered as efficient because it allows us to follow the
flow of the reasoning process from the premises to the conclusion. This method is known as natural deduction.
Once we use the natural form of deduction we can provide a formal proof of the validity of any argument that is
valid.
The formal proof of validity of an argument as a sequence of statements, each of which either is a premise
of that argument or follows from preceding statements of the sequence by an elementary valid argument or by a
logical equivalence, such that the last statement in the sequence is the conclusion of the argument whose validity is
being proved.
An elementary valid argument on the other hand is any argument that is a substitution instance of
an elementary valid argument form. Any substitution instance of an elementary valid argument form is an
elementary valid argument (Copi and Cohen, 2014).

9.2 The Elementary Valid Argument Forms

The main objective is to create set of logical rules, which we can prove the validity of deductive arguments.
The few elementary argument forms that have been introduced include Modus Ponens and Disjunctive Syllogisms.
In addition to the two, we would need rules that are more powerful. The rules of inference may be thought
as a toolbox, from which tools can be taken as needed to prove validity.
The following are considered to be the essential elementary valid argument forms for the application of
Modern Logic.
 Modus Ponens (M.P)

136 | P a g e
 Modus Tollens (M.T.)
 Hypothetical Syllogisms (H.S.)
 Disjunctive Syllogism (D.S.)
 Constructive Dilemma (C.D.)
 Absorption (Abs.)
 Simplification (Simp.)
 Conjunction (Conj.)
 Addition (Add.)

1. Modus Ponens
Modus Ponens is considered to be one of the simplest type of intuitively valid argument which
involves a conditional statement which is illustrated in the following argument:

If the second native told the truth, then only one native is a politician
The second native told the truth
Therefore only one native is a politician

In considering the statements above, we assign representation and convert them to symbolic form.
Let:
o p be your second native
o q be your politician

The argument is represented as


o p⊃q
o p
 q

Modus Ponens is also known as the method of putting or affirming. It is an argument which, if the
truth of a hypothetical premise is assumed, and the truth of the antecedent of that premise is also assumed,
we may conclude that the consequent of that premise is true (cs.ucsb.edu, n.d.).

2. Modus Tollens
Modus Tollens refers to the situation wherein if the conditional statement is true, then if the
consequent is false, the antecedent must also be false. The argument form that relies on MT is used to
establish falsehood of some proposition under attack. We consider the following argument:

137 | P a g e
A literal reading of Genesis would lead one to conclude that the world is less than 6,000 years
old and that the Grand Canyon could have been carved by the global flood, 4,500 years ago.
Since this is impossible, a literal reading of Genesis must be wrong.

In considering the statements above, we assign representation and convert them to symbolic form.
Let:
o p be your first sentence
o q be your second sentence

The argument is represented as


o p⊃q
o ~q
 ~p

Modus Tollens is the method of taking away or denying. It is an argument which, if the truth of a
hypothetical premise is assumed, and the falsity of the consequent of that premise is assumed, we may
conclude that the antecedent of that premise is false.

3. Hypothetical Syllogism
Hypothetical Syllogism is one form of argument that contains only conditional statements. We
consider the following argument.

If the first native is a politician, then the first native lies.


If the first native lies, then the first native denies being a politician.
Therefore if the first native is a politician, then the first native denies being a politician.

In considering the statements above, we assign representation and convert them to symbolic form.
Let:
o p be your politician
o q be your native lies
o r be your denies being a politician

The argument is represented as


o p⊃q
o q⊃r
 p⊃r

138 | P a g e
Hypothetical Syllogism contains three distinct statement variables. It contains a hypothetical
proposition as a premise. If the syllogism contains hypothetical propositions exclusively, it is called a pure
hypothetical syllogism; if the syllogism contains one conditional and one categorical premise, it is called a
mixed hypothetical syllogism.

4. Disjunctive Syllogism
The simplest of the argument forms is Disjunctive Syllogism. It relies on the fact that in every true
disjunction, at least one of the disjuncts must be true. Therefore, if one the premises is false then the other
must be true. This is the most common form of the argument. We consider the following argument:

In trying to cover up her own illegal alien peccadillo, or stonewall her way out of it, she was driven
either by stupidity or arrogance.
She is obviously not stupid;
Her plight must result then from her arrogance.

In considering the statements above, we assign representation and convert them to symbolic form.
Let:
o p be your stupidity
o q be your arrogance

The argument is represented as


o p˅q
o ~p
 q

A valid argument form in which one premise is a disjunction, another is the denial of one of the
two disjuncts and the conclusion is the truth of the other disjuncts (Copi and Cohen, 2014).

5. Constructive Dilemma
A dilemma is an argument in which one of two alternatives must be chosen. The antecedents of the
two conditional propositions p ⊃ q and r ⊃ s. In Constructive Dilemma, a combination of two argument in
MP form and it is most certainly valid. We consider the following argument:

If I win a million dollars, I will donate it to an orphanage.


If my friend wins a million dollars, he will donate it to a wildlife fund.

139 | P a g e
I win a million dollars or my friend wins a million dollars.
Therefore either an orphanage will get a million dollars or a wildlife fund will get a million dollars.

In considering the statements above, we assign representation and convert them to symbolic form.
Let:
o p be your “I will a million dollars”
o q be your “I will donate it to an orphanage”
o r be your “ My friend wins a million dollars”
o s be your “he will donate it to a wildlife fund”

The argument is represented as


o (p ⊃ q) . (r ⊃ s)
o p˅r
 q˅s

Constructive Dilemma is considered as one of the nine elementary valid argument forms.

6. Absorption
Any proposition p always implies itself. Thus, if we know that p ⊃ q, we may validly infer that p
implies both itself and q. It makes the principle of identity, one of the basic logical principles, always
available for our use. We consider the following argument:

If it will rain, then I will wear my coat.


Therefore, if it will rain, then it will rain and I will wear my coat.

In considering the statements above, we assign representation and convert them to symbolic form.
Let:
o p be your “It will rain”
o q be your “I will wear my coat”
The argument is represented as
o p⊃q
 p ⊃ (p . q)

7. Simplification
Simplification states that if two propositions are true when they are conjoined then we may validly
infer that one of them is true by itself. We consider the followings statement:

140 | P a g e
John has an apple and an orange.

In considering the statements above, we assign representation and convert them to symbolic form.
Let:
o p be your apple
o q be your orange
The argument is represented as
o p.q
 p

Simplification gives us a tool to validly take one true conjunction that can stand on its own away
from the statement.

8. Conjunction
Conjunction is an argument form wherein two propositions are known to be true and we can put
them together into one conjunctive expression. If they are true separately then they are true together. We
consider the following statements:

Bob likes apples.


Bob likes oranges.
Therefore, Bob likes apples and oranges.

In considering the statements above, we assign representation and convert them to symbolic form.
Let:
o p be your apple
o q be your orange
The argument is represented as
o p
o q
 p.q

9. Addition
Any disjunction is considered true if either of its disjuncts are true. That is, p ˅ q is true if p is true,
or if q is true, or if they are both true. If we know that some proposition is true then we also know that either

141 | P a g e
it is true or some other is true. We can construct a disjunction using only one true proposition. In addition
we add any logical proposition. We consider the following statement:

Michigan is north of Florida.


The moon is made of cheese.

In considering the statements above with assign representation and convert them to symbolic form.
Let:
o p be first statement
o q be second statement
The argument is represented as
o p
 p˅q

We know that the first proposition is true and the second proposition is just simply added to make
a disjunction. If we know that the first proposition is true then some other proposition is either false or true.
Addition is merely adding a proposition to make a disjunction.

The nine elementary argument forms contains two features (1) they must be applied with exactitude.
Arguments that a person is trying to prove valid uses modus ponens must have the exact form. Each statement must
be replaced by some statement consistently and accurately. The elementary argument form must be fitted precisely
to the argument; (2) elementary valid argument forms must be applied to the entire lines of the larger argument.

9.3 Formal Proofs of Validity Exhibited

Formal Proofs of Validity are known to be, for an argument, a sequence of statements which is either a
premise of the argument or follows from a preceding statement of the sequence by an elementary valid argument
or by a logical equivalence such that the last statement is deemed to be the conclusion.

Example:
1. A . B
2. (A ˅ C) ⊃ D
 A.D
3. A
4. A ˅ C
5. D

142 | P a g e
6. A . D

In the example above we can infer that the first two lines are premises. The very last line is the conclusion
which is the same as the conclusion in the given. In applying the 9 elementary argument forms we follow the
following:
 In line 3 we can deduce from line 1 by Simplification.
 In line 4 we can infer that from line 3 that if A is true then C is inferred to be true by Addition.
(Addition tells us that if p is true then p ˅ q is also true)
 In line 5 D is located and D appears from line 2 so in the previous step we proved that A ˅ C
is true then we can infer that if A ˅ C then D is also true by Modus Ponens.
 A has been proved true on line 3 and D has been proved in line 5. We may conjoin them leading
now to line 6 which is A . D by conjunction.

REFERENCES:
Copi and Cohen (2014). Introduction to Logic 14th Edition. Edinburgh Gate: Pearson Education Limited
pp. 312-375

Hurley, Patrick (2008). A concise introduction to logic. Wadsworth Publication Co, p 361

Logic (n.d.). Lessons in logic. Retrieved from https://www.cs.ucsb.edu/~pconrad


cs40/lessons.logic/modusPonensModusTollens.html on April 10, 2018

9.4 Constructing Formal Proofs of Validity

One of the central tasks of deductive logic is to prove formally that valid arguments are really valid. This
chapter will focus on arguments whose formal proofs must be constructed. Proofs may be short and simple or long
and complex. They also said that success in constructing formal proofs requires mastery of the rules of inference.
Formal proofs are simply objective methods of presenting reasoning trails, such that anyone who learns this method
can check the steps against their own common sense.
For simple arguments, the only rules needed are the nine elementary valid argument forms which were
discussed previously. With just these rules, many arguments can be proved valid.
For example, we have the argument:
1. A
2. B
∴ (A ∨ C) · B

143 | P a g e
In this argument, the conclusion (A ∨ C) · B is a conjunction. We have B in the second line, so what we
need to prove is (A ∨ C). A is easily derived from the first line of the argument. By using the Addition rule, which
tells us that to any given p with a truth value of true we may add (disjunctively) any q whatever, we simply add
C. Therefore, the third line in the proof of validity shall be shown as: 3. A ∨ C, 1, Add. The final line of the proof
must show the conclusion of the argument. In this case, the conclusion is (A ∨ C) · B. By using the first three lines,
we can conjoin the disjunction found in line 3 with premise B found in line 2. The fourth line shall be shown as: 4.
(A ∨ C) · B, 3,2, Conj.
The proof of validity shall be written as:
1. A
2. B
3. A ∨ C, 1, Add.
4. (A ∨ C) · B, 3,2, Conj.

9.5 Constructing More Extended Formal Proofs


In this chapter, we will now construct formal proofs of validity of more complex arguments using the same
process and the rules of interference we used to construct formal proofs for simple arguments.
For example, we have this argument:
1. A ∨ (B ⊃ A)
2. ~A · C
∴ ~B
Although it may look simple, this argument requires a longer proof of validity compared to what we have
done in the previous chapter.
Using Modus Tollens, we can infer the conclusion ~B from (B ⊃ A) found in the first premise. We can
infer it from this premise if we can establish the hypothetical separately and establish ~A. To establish ~A, we apply
the rule of Simplification to line 2, which contains the premise ~A · C. It will be written as: 3. ~A, 2, Simp
From there, we now apply Disjunctive Syllogism to infer (B ⊃ A). The fourth line will now be: 4. (B ⊃ A),
1,3, D.S.
Finally, we apply Modus Tollens to lines 4 and 3. The fifth line shall read: 5. ~B, 4,3, M.T.
The formal proof is now complete and shall be written as a whole as:
1. A ∨ (B ⊃ A)
2. ~A · C
3. ~A, 2, Simp
4. (B ⊃ A), 1,3, D.S.
5. ~B, 4,3, M.T.

144 | P a g e
In constructing some proofs, coming up with the appropriate strategy will not be so simple. But, it will be
helpful to ask the following questions: What statement(s) will enable one to infer the conclusion? What
statement(s) will enable one to infer that?

Sometimes in constructing formal proofs, a statement is correctly inferred but it would turn out that it is not
needed in the proof. However, retaining the unneeded statements does not render the proof to be incorrect. It may
be more efficient to allow the statements to remain using the more extended numbering which that inclusion makes
necessary.

In the study of Logic, our aim is to evaluate arguments in a natural language such as English. From those
evaluated arguments, we now use abbreviations. Without setting abbreviations for arguments, people would most
likely use different abbreviations which would make discussion difficult.

For example, we have the following statement: If either Gertrude or Herbert wins, then both Jens and
Kenneth lose. Gertrude wins. Therefore, Jens loses.

We could set the following abbreviations based on the statement above:

G – Gertrude wins.
H – Herbert wins.
J – Jens loses.
K – Kenneth loses.

If we were to translate the statement into symbols, it would read as follows:

1. (G ∨ H) ⊃ (J · K)
2. G
∴J

To prove G ∨ H, we use Addition on the second premise, G. The third line shall now read: 3. G ∨ H, 3,
Add.

From the first and third lines, we apply Modus Ponens on both to produce the fourth line in the proof. The
fourth line shall be: 4. J · K, 1,3, M.P.

145 | P a g e
Lastly, from the fourth line, we use Simplification in order to get the conclusion J. The fifth line is: 5. J, 4,
Simp.

The proof shall read as:


1. (G ∨ H) ⊃ (J · K)
2. G
3. G ∨ H, 3, Add.
4. J · K, 1,3, M.P.
5. J, 4, Simp

9.6 Expanding the Rules of Inference: Replacement Rules

Even though we have the nine elementary rules previously discussed, it is not enough that we use only those
because there are arguments which cannot be proven by limiting ourselves to just the nine elementary rules. The
rule of replacement is a rule that permits us to infer from any statement that is logically equivalent to the compound
replaced. They are called replacement rules because they operate through the substitution of equivalent statements
where possible and where needed. They can be applied on either given or previously derived statements; substituting
entire statements or just their components whenever deemed necessary to establish the desired conclusion.
There are ten rules of replacement.

1. DE MORGAN’S THEOREMS (De M.)

There are two kinds under De Morgan’s Theorem. The first kind asserts that when we deny that both
propositions are both true, that is logically equivalent to asserting that either one of them is false, or the other
one is false, or they are both false.
The second kind asserts that when we deny that either of the two propositions is true, that it is logically
equivalent to asserting that both are false.
For example, the statement “Not both Anna and Ben teach Math” can also be stated as “Either Anna or Ben
do not teach Math.”

2. COMMUTATION (Com.)

In Commutation, it states that the order of statement of the elements of a conjunction, or of a disjunction
does not matter because the meaning stays the same.

146 | P a g e
The statement “Uzbekistan and Liechtenstein are doubly landlocked countries in the world.” can also be
stated as “Liechtenstein and Uzbekistan are doubly landlocked countries in the world.”

3. ASSOCIATION (Assoc.)

Association allows us to group statements differently. Knowing that three statements are true, to assert that
P is true along with Q and R clumped is logically equivalent to asserting that P and Q clumped is true along with
R.
The grouping of components of a statement consistently related by way of disjunction, or by way of
conjunction, in just one statement may vary without affecting the basic assertion of the statement.
The statement “Jimin is a dancer while both Jin and Kook are singers.” can also be stated, without any
change in what is being asserted as, “Jimin is a dancer while Jin is a singer, and Kook is also a singer.”

4. DISTRIBUTION (Dist.)

There are two kinds of distribution. The first asserts that the conjunction of one statement with the
disjunction of two other statements is logically equivalent to a disjunction whose first disjunct is the
conjunction of the first statement with the second and whose second disjunct is the conjunction of the first
statement with the third.
The second asserts that the disjunction of one statement with the conjunction of two others is logically
equivalent to the conjunction of the disjunction of the first and the second and the disjunction of the first and the
third.
The statement “Kook is a humble dancer while Dae and Hyun are arrogant dancers.” can also be stated as

5. DOUBLE NEGATION (D.N.)

Double Negation simply asserts that any statement is logically equivalent to the negation of the negation of
that same statement.
The statement “It is false that Namjoon does not want to study Law.” can also be stated as “Namjoon wants
to study Law.

6. TRANSPOSITION (Trans.)

147 | P a g e
Transposition allows us to turn any conditional statement around. If any conditional statement is true, then
if its consequent is false its antecedent must also be false. any conditional statement is logically equivalent to the
conditional statement asserting that the negation of its consequent implies the negation of its antecedent.

Transposition expresses the logical force of Modus Tollens.

7. MATERIAL IMPLICATION (Impl.)

This logical equivalence does no more than formulate the definition of material implication as a replacement
that can serve as a rule of inference. In material implication, p ⊃ q means that either the antecedent (p) is false, or
the consequent (q) is true.
It is often easier to manipulate or combine two statements if they have the same basic form if they are both
in disjunctive form, or if they are both in implicative form.

8. MATERIAL EQUIVALENCE (Equiv.)

There are two kinds of material equivalence. The first kind tells us that the assertion of the statements’
material equivalence is logically equivalent to asserting that they are both true or they are both false. Meanwhile,
the second kind says that the statement that they are materially equivalent is logically equivalent to the statement
that they imply one another.

9. EXPORTATION (Exp.)

If one asserts that two propositions conjoined are known to imply a third, that is logically equivalent to
asserting that if one of those two propositions is known to be true, then the truth of the other must imply the truth
of the third.
The statement “If you always sleep and eat fatty foods, then you will get fat.” can also be stated as “If you
always sleep, then you will get fat if you always eat fatty foods.”

10. TAUTOLOGY (Taut.)

Any statement is logically equivalent to the disjunction of itself with itself, and that any statement is
logically equivalent to the conjunction of itself with itself. Tautology is understood as “saying the same thing itself.”

148 | P a g e
A choice between one thing and itself would yield the same thing, and likewise, combining the same
description o one person or thing is the same as saying the same thing.

REFERENCES:
Abordo, A., Adora, M., Matay-eo, F., Taggaoa, R., & Villanueva, R. (2012). Logic Manual. Baguio City: Saint
Louis University.
Copi, I., Cohen, C., & McMahon, K. (2014). Introduction to Logic (14th ed.). Edinburgh Gate, Essex: Pearson
Educational Limited.
Pine, R. C. (n.d.). Essential Logic. Retrieved April 25, 2018, from
http://coursecontent1.honolulu.hawaii.edu/~pine/EL/chapt9-EL-am.pdf.....

9.7 The System of Natural Deduction

According to Copi and Cohen, there is an important difference between the first nine and the last ten rules
of the inference.

The first nine rules can be applied only to whole lines of a proof serving as premises.

Illustration:
Statement A cannot be inferred from the statement A B by simplification only if A B constitutes a whole line.
Statement A cannot be inferred validly either from (A ⊃ B) C or from C ⊃ (A B). This is because the latter two
statements can be true while A is false.
Likewise, the statement A ⊃ C does not follow from the statement (A B) ⊃ C by simplification of by any other
rule of inference.

Applying the simplification or any other rule of inference it does not follow at all.
If A is true and B and C are both false then (A B) ⊃ C is true but A ⊃ C is false.
Additional equivalences or additional valid argument forms can be added to our set of rules but it would make it
more difficult to command. Although A v B follows from A by Addition, we cannot infer (A v B) ⊃ C from A ⊃
C by addition of by any other rule of inference also.
For if A and C are both false and B is true, A ⊃ C is true but (A v B) C is false.

The last ten rules can be applied either to whole lines or to parts of lines. By replacement, logically equivalent
expressions can replace each other wherever they occur, even where they do not constitute whole lines of a proof.

149 | P a g e
Illustration:
Statement A ⊃ ( B ⊃ C ) can be inferred not only from the whole line of ( A B ) ⊃C by exportation but from the
line [ ( A B ) ⊃C ] v D we can infer [ A ⊃ ( B ⊃ C ) ] v D by exportation.

With the notion of formal proof that it can be decided mechanically in a finite number of steps where no thinking
is required. Only two things are required:
1. The ability to see that a statement occurring in one place is the same as a statement occurring another.
2. The ability to see whether a given statement has a certain pattern to see if it is a substitution instance
of a given statement form.
Thus, the formal proof of validity of the numbered sequence of statements below can be easily settled.

1. A V B /.·.A
2. ˜ B 1, Com.
3. B V A 3,2, D.S.
4. A

Lines 1 and 2 are the premises.


Line 4 is the conclusion of the given argument.
Line 3 follows from preceding lines by one of the given rules of inference can be decided in a finite number of steps
even where the notation “1, Com.” is not written.
The explanatory notation in the 2nd column should always be included but is not necessary part of the proof itself.
It can be verified by inspection and comparison of shapes that 3 does not follow from 1 and 2 by Modus
Ponens or by Modus Tollens, or by a Hypothetical Syllogism and so on.
The legitimacy of any statement in any formal proof can be tested in a finite number of steps. To preserve
this effectiveness one step should be taken at a time. However, constructing a formal proof is not an effective
procedure. Formal proofs differ from truth tables. The making of truth tables is completely mechanical and we can
always construct a truth table to test its validity by following the simple rules of procedure set but we have no
mechanical rules for the construction of formal proofs.
In the construction of formal proofs, some rules of thumb of hints are suggested. The following are the rules of
thumb or hints:
a) Deducing conclusions from the given premises by the given rules of inference.
b) Eliminating statements that occur in the premises but not in the conclusion.
- Elimination can proceed only in accordance with the rules of inference. But these rules contain
many techniques for eliminating statements such as:
i. Simplification- the right- hand conjunct can be dropped from a whole line that is a
conjunction.
150 | P a g e
ii. Commutation- permits switching the left-hand conjunct of a conjunction over to the right-
hand side, from which it can be dropped by Simplification.
 The middle term q can be eliminated by a Hypothetical Syllogism given two
statements of the patterns p ⊃ q and q ⊃ r.
iii. Distribution- a useful rule for transforming a disjunction of the pattern p v (q · r) into the
conjunction (p v q) (p v r) whose right- hand conjunct can then be eliminated by
Simplification.
c) Working backward from the conclusion by looking for statement/statements from which it can be deduced
and then try to deduce those intermediate statements from the premises.

9.8 Constructing Formal Proofs Using the Nineteen Rules of Inference

Elementary Valid Argument Forms


1. Modus Ponens (M.P.)
2. Modus Tollens (M.T.)
3. Hypothetical Syllogism (H.S.)
4. Disjunctive Syllogism (D.S.)
5. Constructive Dilemma (C.D.)
6. Absorption (Abs.)
7. Simplification (Simp.)
8. Conjunction (Conj.)
9. Addition (Add.)
Logically Equivalent Expressions
10. De Morgan’s theorems (De M.)
11. Commutation (Com.)
12. Association (Assoc.)
13. Distribution (Dist.)
14. Double Negation (D.N.)
15. Transposition (Trans.)
16. Material Implication (Impl.)
17. Material Equivalence (Equiv.)
18. Exportation (Exp.)
19. Tautology (Taut.)

We begin with simple arguments whose proofs require only two statements added to the premises.

151 | P a g e
A ‫~ ﬤ‬A
∴~A
ANSWER:
1. A ‫~ ﬤ‬A
∴~A
2. ~A ∨ ~A 1, Impl.
3. ~A 2, Taut.
The Material Implication was used as the first statement and the Tautology was applied in the conclusion and as the
second statement.

Formal proofs with three lines added to the premises


~A ‫ ﬤ‬A
∴A

ANSWER:
1. ~A ‫ ﬤ‬A
∴A
2. ~~A ∨ A 1, Impl.
3. A∨A 2, D.N.
4. A 3, Taut.

The Material Implication was used as the first statement, the Double Negation in the second, and Tautology was
used in the third statement.

Formal Proofs with pattern of Inference


~A
∴A‫ﬤ‬B

ANSWER:
1. ~A
∴A‫ﬤ‬B
2. ~A ∨ B 1, Add.
3. A‫ﬤ‬B 2, Impl.

SOLUTION (Copi & Cohen):

152 | P a g e
If is known to be true, as here, then A must be false. A false statement materially implies any other statement. So
must be true, whatever B may assert, if we know that is true. In this case, is given as premise; we only need to add
the desired B and then apply Implication. The proof of the argument (or the proof segment, when it is a part of some
longer proof)

More Convoluted construction of formal proof..


A ‫~ ﬤ‬B
~(C•~A)
∴ C ‫~ ﬤ‬B

ANSWER:
1. A ‫~ ﬤ‬B
2. ~(C•~A)
∴ C ‫~ ﬤ‬B
3. ~C ∨ ~~A 2, De M.
4. C ‫~~ ﬤ‬A 3, Impl.
5. C‫ﬤ‬A 4, D.N.
6. C ‫~ ﬤ‬B 5, 1,H.S.
SOLUTION (Copi &Cohen):
In this argument the conclusion unites a statement that appears in the second premise, C, with a statement that
appears in the first premise, How shall we effect that unification? The first premise is a conditional whose
consequent, , is also the consequent of the conclusion. The second premise contains the negation of the antecedent
of the first premise, . If we can manipulate the second premise to emerge with, we can achieve the needed unification
with H.S. We can do that. If we apply De M. to the second premise we will get a disjunction that, when replaced
by a conditional using Impl., will be one short step away from the conditional needed.

REFERENCES:
Copi, I., Cohen, C., & McMahon, K. (2014). Introduction to Logic (14th ed.). Edinburgh Gate, Essex:
Pearson Educational Limited

9.9. PROOF OF INVALIDITY

In an invalid argument there is no formal proof of validity. However, failure to discover formal proof of
validity does not mean that the argument is invalid, we just haven’t tried enough. Our inability to find a proof of
validity may be caused by the fact that the argument is not valid, but it may be caused instead by our own lack of

153 | P a g e
ingenuity—as a consequence of the non-effective character of the process of proof construction. Thus, we ask:
what does constitute a proof that a given instrument is invalid?
The method to be described is closely related to the truth-table method but it is obviously shorter. We all
know that an argument is invalid when all the premises are true but the conclusion is false. Now, if we can make
an assignment of truth values to the simple component statements of an argument that will make its premises true
and its conclusion false, then making that assignment will suffice to prove that the argument is invalid. Thus, if
we can make such assignment of truth values without actually contracting the whole truth table, much work will
be eliminated.

Consider this argument:


If the governor favors public housing, then she is in favor of restricting the scope of private
enterprise.
If the governor were a socialist, then she would be in favor of restricting the scope of private
enterprise.
Therefore, if the governor favors public housing, then she is a socialist.

F⊃R
S⊃R
∴F⊃S

We can prove it invalid without having to construct a complete truth table. First, we have to determine
what assignment of truth values is needed to make the conclusion false. Thus, we start in the conclusion to
determine the truth value of F and S. Based on the example, a conditional is false only if its antecedent is true and
the consequent false. Thus assigning the truth value TRUE to F and FALSE to S will make the conclusion F ⊃ S
false. Furthermore, if the truth value TRUE is assigned to R, both premises are true.
This method of proving invalidity is an alternative to the truth-table method of proof. what we did when
we made the indicated assignment of truth values was to construct one row of the given argument’s truth table.
The relationship can perhaps be seen more clearly when the truth-value assignments are written out horizontally:

F R S F⊃R S⊃R F⊃S


T T F T T F

An argument is proved invalid by displaying at least one row of its truth table in which all its premises are
true but its conclusion is false. Consequently, we need not examine all rows of its truth table to discover an
argument’s invalidity: discovering a single row in which its premises are all true and its conclusion false will
suffice. This method of proving invalidity is a method of constructing such a row without having to construct the

154 | P a g e
entire truth table. The present method is shorter than writing out an entire truth table. However, this method has
uncertainties so a certain amount of trial and error is likely to be needed.

Examples:
1.) A⊃B
C⊃D
AvD
∴BvC

ANSWER:
A B C D A⊃B C⊃D AvD BvC
F F F T T T T F

2.) S ⊃ (T⊃U)
V ⊃(W⊃X)
T ⊃ (V • W)
~ (T • X)
∴S≡U

ANSWER:
S T U V W X S ⊃ (T⊃U) V ⊃(W⊃X) T ⊃ (V • W) ~ (T • X) S≡U
T F F T T T T T T T F

9.10. INCONSISTENCY

An argument is proved invalid if truth values can be assigned to make all of its premises true and its
conclusion false. If a deductive argument is not invalid, it must be valid. So, if truth values cannot be assigned to
make the premises true and the conclusion false, then the argument must be valid. This follows from the definition
of validity, but it has this curious consequence: Any argument whose premises are inconsistent must be valid.
In the following argument, for example, the premises appear to be totally irrelevant to the conclusion:
If the airplane had engine trouble, it would have landed at Bend.

If the airplane did not have engine trouble, it would have landed at Creswell.
The airplane did not land at either Bend or Creswell.

Therefore, the airplane must have landed in Denver.
155 | P a g e
Here is its symbolic translation:
A⊃B
~A⊃C
~ (B v C)
∴D

No truth-value assignment can make them all true because they are inconsistent with one another. Their
conjunction is self-contradictory, being a substitution instance of a self-contradictory statement form. If we were to
construct a truth table for this argument, we would find that in every row at least one of the premises is false.
Because there is no row in which the premises are all true, there is no row in which the premises are all true and the
conclusion false. Hence the truth table for this argument would establish that it is in fact valid. Of course, we can
also provide a formal proof of its validity:

1. A ⊃ B
2. ~ A ⊃ C
3. ~ (B v C) / ∴ D
4. ~ B • ~ C 3, De M.
5. ~ B 4, Simp.
6. ~ A 1, 5, M.T.
7. C 2, 6, M.P.
8. ~ C • ~ B 4, Com.
9. ~ C 8, Simp.
10. C v D 7, Add.
11. D 10, 9, D.S.

In this proof, lines 1 through 9 are devoted to making explicit the inconsistency that is implicitly contained
in the premises. That inconsistency emerges clearly in line 7 (which asserts C) and line 9 (which asserts ~C). Once
this explicit contradiction has been expressed, the conclusion follows swiftly using Add. and D.S.
Thus, we see that if a set of premises is inconsistent, those premises will validly yield any conclusion, no
matter how irrelevant. The essence of the matter is more simply shown with the following outrageous argument,
whose openly inconsistent premises allow us to infer validly an irrelevant and absurd conclusion.
Notice that if the premises of an argument are inconsistent—that is, if they directly contradict each other—
then there will be no line in the truth-table for that argument on which all of the premises are true. In that case, the
argument is valid no matter what its conclusion happens to be! It is impossible for the premises to be true while the
conclusion is false simply because it is impossible for the premises to be true.

156 | P a g e
It's even possible to construct a valid proof for any conclusion once you accept contradictory premises:
1. A
2. ~A
3. A v X 1, Add.
4. X 3, 2 D.S.

This is what's wrong with contradicting yourself, that once you endorse contradictory propositions, you
can prove anything. The point here is, although arguments with inconsistent premises are all valid, they cannot be
sound, since at least one of their premises must be false. Thus, any argument with inconsistent premises is valid,
regardless of what its conclusion may be.

9. 11. INDIRECT PROOF OF VALIDITY

Indirect proofs are also known as a “reductio ad absurdum” (i.e. “reduction to the absurd”). An indirect
proof of validity is written out by stating as an additional assumed premise the negation of the conclusion. If we
can derive an explicit contradiction from the set of premises thus augmented, the argument with which we began
must be valid.
This is so because, contradictory statements cannot both be true. Therefore, a statement added to the
premises that makes it possible to deduce a contradiction must entail a falsehood. This gives rise to another
method of proving validity. Suppose we assume (for the purposes of the proof only) the denial of what is to be
proved. Suppose, using that assumption, we can derive a contradiction. That contradiction will show that when we
denied what was to be proved we were brought to absurdity. We will have established the desired conclusion
indirectly, with a proof by reductio ad absurdum.

Let us take for example the following argument:


1. A ⊃ (B • C)
2. (B v D) ⊃ E
3. D v A
∴E

In the very next line we make our assumption (for the purpose of the indirect proof) of the denial of the
conclusion.

4. ~ E I.P. (Indirect Proof)

157 | P a g e
With the now enlarged set of premises we can, using the established rules of inference, bring out an
explicit contradiction, thus:
5. ~ (B v D) 2, 4, M.T.

6. ~ B • ~ D 5, De M.

7. ~ D • ~ B 6, Com.

8. ~ D 7, Simp.

9. A 3, 8, D.S.

10. B • C 1, 9, M.P.
11. B 10, Simp.

12. ~ B 6, Simp.

13. B • ~ B 11, 12, Conj.

It is obvious in the last line of the proof an explicit contradiction, which is the demonstration of the
absurdity of the to which we were led by assuming ~ E in line 4. This contradiction, formally and explicitly
expressed in the last line, exhibits the absurdity and completes the proof.
We can illustrate this also by first constructing a direct formal proof of an argument, and then
demonstrating the validity of that same argument using an indirect proof. When illustrating, the proof without
reduction ad absurdum is written on the left side while the proof with reduction ad absurdum is written on the
right side with an exclamation point (!) before each number to indicate that it has been derived through reduction
ad absurdum.

Take for example:


1. (H ⊃ I) • (J ⊃ K)
2. (I v K) ⊃L
3. ~ L
∴ ~ (H v J)
4. ~ (I v K) 2, 3, M.T. !4 ~ ~ (H v J) I.P. (Indirect Proof)
5. ~ I • ~K 4, De M. !5 H v J 4, D.N.
6. ~ I 5, Simp. !6
I v K 1, 5, C.D.

7. H ⊃ I 1, Simp. !7
L 2, 6, M.P.
8. ~ H 7, 6, M.T. !8 L • ~ L 7, 3, Conj.
9. (J ⊃ K) • (H ⊃ I) 1, Com.

10. J ⊃ K 9, Simp.

11. ~ K • ~ I 5, Com.

12. ~ K 11, Simp.
158 | P a g e
13. ~ J 10, 12, M.T.
14. ~ H • ~ J 8, 13, Conj.
15. ~ (H v J) 14, De M.

Examples:

1.) 1. D ⊃ (Z ⊃ Y)
2. Z ⊃ (Y ⊃ ~ Z)
∴~Dv~Z
3. ~ (~ D v ~ Z) I.P. (Indirect Proof)
4. ~ ~ D • ~ ~ Z 3, De M.

5. D • ~ ~ Z 4, D.N.

6. D • Z 5, D.N.
7. D 6, Simp.
8. Z ⊃ Y 1, 7, M.P.

9.Z • D 6, Com.

10. Z 9, Simp.

11. Y ⊃ ~ Z 2, 10, M.P.
12. Y 8, 10, M.P.
13. ~ Z 11, 12, M.P.
14. Z • ~ Z 10, 13, Conj.

9.12. SHORTER TRUTH-TABLE TECHNIQUE


Another method of testing the validity of arguments.
So, we can prove the validity of an argument by showing that no such set of truth values can be
assigned.
We do this by showing that its premises can be made true, and its conclusion false, only by
assigning truth values inconsistently—that is, only with an assignment of values such that some component
statement is assigned both a T and an F.
Proving the validity of an argument with this “shorter truth-table technique” is one version of the
use of reductio ad absurdum, reducing to the absurd—but instead of using the rules of inference, it uses truth-value
159 | P a g e
assignments.
Its easiest application is when F is assigned to a disjunction (in which case both of the disjuncts
must be assigned F) or T to a conjunction (in which case both of the conjuncts must be assigned T). When
assignments to simple statements are thus forced, the absurdity (if there is one) is quickly exposed.
But where the method calls for T to be assigned to a disjunction, we cannot be sure which disjunct
is true; and where F must be assigned to a conjunction, we can- not be sure which conjunct is false; in such cases
we must make various “trial assignments,” which slows the process and diminishes the advantage of this method.

REDUCTIO AD ABSURDUM
A mode of argumentation or a form of argument in which a proposition is disproven by following
its implications logically to an absurd conclusion. Arguments that use universals such as, “always”, “never”,
“everyone”, “nobody”, etc., are prone to being reduced to absurd conclusions. The fallacy is in the argument that
could be reduced to absurdity -- so in essence, reductio ad absurdum is a technique to expose the fallacy.
The proof without the reductio ad absurdum is on the left and requires fifteen steps; the proof using the
reductio ad absurdum is on the right and requires only eight steps. An exclamation point (!) is used to indicate that
a given step is derived after the assumption advancing the indirect proof had been made.
Take for example:
1. (H ⊃ I) • (J ⊃ K)
2. (I v K) ⊃L
3. ~ L
∴ ~ (H v J)
4. ~ (I v K) 2, 3, M.T. !4 ~ ~ (H v J) I.P. (Indirect Proof)
5. ~ I • ~K 4, De M. !5 H v J 4, D.N.
6. ~ I 5, Simp. !6
I v K 1, 5, C.D.

7. H ⊃ I 1, Simp. !7
L 2, 6, M.P.
8. ~ H 7, 6, M.T. !8 L • ~ L 7, 3, Conj.
9. (J ⊃ K) • (H ⊃ I) 1, Com.

10. J ⊃ K 9, Simp.

11. ~ K • ~ I 5, Com.

12. ~ K 11, Simp.
13. ~ J 10, 12, M.T.
14. ~ H • ~ J 8, 13, Conj.
15. ~ (H v J) 14, De M.

Example:
Assume P is true.
160 | P a g e
From this assumption, deduce that Q is true.
Also, deduce that Q is false.
Thus, P implies both Q and not Q (a contradiction, which is necessarily false).
Therefore, P itself must be false.
Source: https://www.logicallyfallacious.com/tools/lp/Bo/LogicalFallacies/151/Reductio-ad-Absurdum

161 | P a g e
CHAPTER 11: ANALOGICAL REASONING
11.1. INDUCTION AND DEDUCTION REVISITED
Arguments are built on premises that are believed, or assumed, to be true.
Our reasoning process usually begins with the accepted truth of some “matters of fact,” in David
Hume’s phrase. To establish matters of fact we must rely on reasoning that is inductive.
Induction thus provides the starting points—the foundation—for the reasoning that concerns us
most. We reason to establish truths in our everyday lives, to learn facts about our society, to understand the natural
world.
Deduction is certainly powerful in enabling us to move from known (or assumed) propositions to
other propositions that those premises entail, but in the search for truths with which our reasoning must begin, it is
insufficient.
In deductive arguments, the claim is made that conclusions follow with certainty from their
premises
That claim is appropriate because any deductive argument, if it is good, brings to light in its
conclusion what was already buried in its premises.
The relation between premises and conclusion, in deduction, is one of logical necessity. In every
deductive argument, if it is valid and if its premises are true, its conclusion must be true.
In inductive arguments—the concern of this chapter—the relation between premises and
conclusion is not one of logical necessity
The claim of certainty is not made. The terms valid and invalid simply do not apply.
This does not mean that inductive arguments are always weak; sometimes they are very strong
indeed, and fully deserve our confidence.

Example:
Scientists now assert without reservation (for example) that smoking is a cause of cancer. This is true, but
it is a truth that cannot be known with the demonstrative certainty of a valid syllogism.
If p or q is true, and not p is true, we may conclude that q must be the case, beyond all doubt.
In the realm of induction, as we seek new knowledge of facts about the world, nothing is beyond
all doubt.
We must rely on arguments that support their conclusions only as probable, or probably true.
Arguments grounded on analogies, aiming to establish particular conclusions, are examined first, in this
chapter. Arguments that go beyond particulars, aiming to establish generally applicable causal laws, are also
examined in this chapter. We shall also explore the uses of hypotheses and their confirmation in developing
scientific theories, and we will analyse the concept of probability itself, the conceptual instrument with which
inductive conclusions are commonly expressed.

162 | P a g e
11.2. ARGUMENT BY ANALOGY

Analogy- A parallel drawn between two (or more) entities by indicating one or more respects in
which they are similar.
-To give an analogy is to claim that two distinct things are alike or similar in some
respect.

Here are examples :


Capitalists are like vampires.
Like the Earth, Europa has an atmosphere containing oxygen.
Note: The most common type of inductive relies on analogy.

Analogical Argument- A kind of inductive argument in which, from the fact that two entities
are alike in some respect(s), it is concluded that they are also alike in
some other respects.
- To argue by analogy is to argue that because two things are similar, what
is true of one is also true of the other.
- According Copi and Cohen, Arguments by analogy are not to be classified as either valid or
invalid; it is only probability that is claimed for them. In all such arguments, it is plainly-
logically possible that although the premises are true, the conclusions are false.
- Analogy is also used as explanation, when something that may not be familiar to the reader,
they may compare it to something else, presumably more familiar, to which it has certain
similarities.

Example:
A. If I tell you that I got very good service from a computer of a certain make and model, you may infer
that a new computer of the same make and model will serve you well.
B. When a new book is called to my attention and I infer that I will enjoy reading it because I have read
and enjoyed other books by the same author.
C. There might be life on Europa because it has an atmosphere that contains oxygen just like the Earth.
D. This novel is supposed to have a similar plot like the other one we have read, so probably it is also
very boring.

So in general, when we make use of analogical arguments, it is important to make clear in what ways are two
things supposed to be similar..

163 | P a g e
Non-argumentative Analogy- are commonly encountered in the writing of high school students.
- They do not infer any argument.
Examples:
A.) Thanos fell 12 stories, hitting the pavement like a sloppy bag filled with vegetable soup.
B.) Her hair glistened in the rain, like a nose hair after a sneeze.
C.) He was deeply in love. When she spoke he thought he heard bells, like a garbage truck
backing up.

Form
Argumentative analogies are arguments that employ analogies as premises. Typically, arguments by analogies
contain three parts:
1. The analogy (as a premise) which contends that two different cases (A and B) are analogous, i.e. they
share some relevant feature x.
2. A statement that says from case A, some proposition P follows.
3. The conclusion that says since case A and case B are analogous, and P follows from A, then it also
follows from B.
Here is the basic structure of an analogy:
Analogy: a,b,c,d all have the attributes P and Q
Statement: a,b,c all have the attribute R
Conclusion: Therefore d probably has the attribute R.

11.3. APPRAISING ANALOGICAL ARGUMENTS

Here are some criteria we can use in evaluating the relative strength of analogical arguments.
1.) Number of Entities

 The larger the comparison set, the stronger the argument. The more things we can name that are like the
target subject that also have the target attribute, the more convincing our argument will be.
Example:
“If we can name several other Nicholas Sparks novels that became block-buster movies, the more
probable it will be that his latest novel will also become a block-buster movie. “
If we change our premise to:

164 | P a g e
"Nicholas Spark's novels The Notebook, The Vow, A Walk To Remember, Dear John, and Safe
Haven were all made into block-buster movies."
This strengthens the argument.

2.) Number of Similar Aspects

 The more comparison attributes, the stronger the argument. The more ways we know the target subject is
like the members of the comparison set, the more probable it will be that it is also like the members of the
comparison set in having the target attribute.
Example:
If we change our premise to:
"Nicholas Spark's novels The Notebook, The Vow, A Walk To Remember, Dear John, and Safe
Haven were all made into block-buster movies."

3.) Variety of the instances in the premises


 The more differences there are between the members of the comparison set, the stronger the argument.
 Suppose the members of the comparison set are quite different except that they all have the comparison
attributes. Then any ways the target subject may be different from the members of the comparison set are
less likely to be important in determining whether the target subject will share the target attribute with the
members of the comparison set.
Example:
If we change our premise to;
"Nicholas Spark's novels The Notebook, The Vow, A Walk To Remember, Dear John, and Safe
Haven were all made into block-buster movies."
The first was about a rich girl and a poor guy falling in love, the second was about a couple rekindling their
marriage, and the third was about young love.", this strengthens the argument. “Dear John” will not
be exactly like Nicholas’ other novels that were made into block-buster movies. The more variety there is in the
stories in those other novels, the less important it will be for our conclusion that the story in the latest novel is
different from those in the earlier books.

4.) Disanalogies weaken an analogical argument.

 A disanalogy is a way that the target subject is different from the members of the comparison set.
 Disanalogies are most damaging when the difference is one that is particularly relevant to the comparison
attribute.
Example:

165 | P a g e
Suppose that someone were to point out that The Notebook, The Vow, Dear John, and Safe Haven were all
best-sellers, but that A Walk To Remember was not selling well. How well a book sells is certainly relevant
to the decision whether to make a movie of it.
So this would be an important disanalogy between the Nicholas Sparks novels in the comparison set and A
Walk To Remember.
 Looking for disanalogies is an important part of evaluating an analogical argument.
Of course, no two things are exactly alike and it will always be possible to find some ways that the target
subject is different from the members of the comparison set in an analogical argument.
Example:
It might turn out that there was a picture of the author on the back of all four of these novels, and that
Nicholas Sparks was wearing a coat and tie in the photo on The Notebook, The Vow, Dear John, and Safe
Haven but he was wearing a sweater in the photo on A Walk To Remember. But this is hardly relevant to
whether the book will be made into a movie and does not weaken the argument.

5.) Relevance
 The more relevant the comparison attributes are to the target attribute, the stronger the argument. And
the less relevant the comparison attributes are, the weaker the argument.
Example:
If we change our premise to:
"The novels The Notebook, The Vow, Dear John, and Safe Haven were all sold at National Bookstore,
and they were all made into block-buster movies"
This weakens the argument. A large book seller like National Bookstore tries to carry the broadest selection of
books possible. That they carry a particular book is not very relevant to whether that book is made into a movie.
Example, National Bookstore certainly carry Revised Penal Code Annotated by Reyes and Statutory Construction
Compendium Book by Atty. Mondok, but it's pretty unlikely either of these will be made into a movie! That all
the books mentioned in the premise of our analogical argument were written by the same author seems much
more relevant when we think about whether another book will be made into a movie.

6.) Claims that the conclusion makes

 The stronger the target attribute, the weaker the argument. A strong target attribute means that the
conclusion is making a strong claim. The stronger claim we make in our conclusion in any argument, the
stronger evidence we need to support our claim.
 So if the premises are the same, then they support a weaker conclusion better than a stronger one.

166 | P a g e
In our example, the claim that Safe Haven will become a block-buster movie is a pretty strong claim. We could
weaken it if we just claimed that A Walk To Remember will become a movie. By weakening the target attribute,
and thus the conclusion, we would produce a stronger argument.

11.4. REFUTATION BY LOGICAL ANALOGY

This method of refutation by logical analogy points the way to an excellent general technique for testing
arguments. To prove the invalidity of an argument, it suffices to formulate another argument that:
(1) has exactly the same form as the first; and
(2) has true premises and a false conclusion.

What is the rationale for this procedure?


- The first thing to understand is that validity is a formal or structural property of an argument.
- To say that an argument is valid is to say nothing about whether its constituent propositions
are true. What it says is that if the premises are true, then the conclusion is true. A valid
argument is such that it is impossible for its premise(s) to be true while its conclusion is false.
- Validity is a relation between premise(s) and conclusion, not a property of premise(s) or
conclusion. A valid argument is one that preserves truth. We value validity because—and
only because—we value truth. It is a means to our end. Since validity concerns only the form
of an argument, if two arguments have the same form and one of them is invalid, then the
other is invalid as well. (In other words, two arguments with the same form are either both
valid or both invalid.)
Suppose we are wondering whether a particular argument, X, is valid. One way to find out is to try to construct
another argument of the same form as X that has true premises and a false conclusion. Suppose we manage to do
this. Then, given the definition of “valid argument,” we may infer that the second (constructed) argument is
invalid, for (by definition) no valid argument has true premises and a false conclusion. But if the second argument
is invalid and if the second argument has the same form as X, then X, too, is invalid.
Steps of refuting arguments:
Step 1: Our first task is to set up the syllogism in standard form and order and label our terms
Step 2: Invent classes so that the conclusion is clearly false.
Step 3: Substitute the appropriate minor and major terms in the major and minor premisses.
Step 4: Choose a middle term which will make the premisses clearly true.

Example:

167 | P a g e
Since no conservatives are liberals and all religious people are conservatives, all religious people are
liberals.
Argument A1:
1. No C are L.
2. All R are C.
Therefore,
3. All R are L.
Let us substitute terms for the letters “C,” “L,” and “R” in such a way as to make 1 and 2 true and 3 false.
Argument A2:
4. No triangles are squares.
5. All three-sided figures are triangles.
Therefore,
6. All three-sided figures are squares.

Premises 4 and 5 are true, but 6 is false. What this shows is that A2 is invalid, for, by definition, no valid
argument has true premises and a false conclusion. But if A2 is invalid and if A2 has the same form as A1, then
A1 is invalid. The refutation is complete.

Websites:
http://philosophy.hku.hk/think/arg/analogy.php
http://donald.nute.ws/PHIL1500/Analogies/analogy_notes.htm
https://www.scribd.com/doc/255205739/Copi-and-Cohen-s-Introduction-to-Logic
https://www.uta.edu/philosophy/faculty/burgessjackson/Refutation%20by%20Logical%20Analogy.pdf
Youtube Video:
https://www.youtube.com/Analogical_reasoning
Books:
Systematic Approaches to Argument by Analogy- by Henrique Jales Ribeiro

168 | P a g e

Das könnte Ihnen auch gefallen